Anda di halaman 1dari 164

DAFTAR ISI

Tugas Mandiri – 1 ................................................................................................................... 1


Tugas Mandiri – 2 .................................................................................................................... 7
Tugas Mandiri – 3 .................................................................................................................... 14
Tugas Mandiri – 4 .................................................................................................................... 21
Tugas Mandiri – 5 .................................................................................................................... 29
Tugas Mandiri – 6 .................................................................................................................... 34
Tugas Mandiri – 7 .................................................................................................................... 41
Tugas Mandiri – 8 .................................................................................................................... 48
Tugas Mandiri – 9 .................................................................................................................... 55
Tugas Mandiri – 10 .................................................................................................................. 63
Tugas Mandiri – 11 .................................................................................................................. 68
Tugas Mandiri – 12 .................................................................................................................. 75
Tugas Mandiri – 13 .................................................................................................................. 81
Tugas Mandiri – 14 .................................................................................................................. 87
Tugas Mandiri – 15 .................................................................................................................. 94
Tugas Mandiri – 16 ..................................................................................................................101
SNMPTN Tahun 2008 Kode Soal 301 .......................................................................................107
SNMPTN Tahun 2009 Kode Soal 183 .......................................................................................113
SNMPTN Tahun 2009 Kode Soal 383 .......................................................................................121
SNMPTN Tahun 2010 Kode Soal 326 .......................................................................................128
SNMPTN Tahun 2010 Kode Soal 336 .......................................................................................135
SNMPTN Tahun 2011 Kode Soal 123 .......................................................................................143
SNMPTN Tahun 2012 Kode Soal 321 .......................................................................................150
SBMPTN Tahun 2013 Kode Soal 123 .......................................................................................157

i
1. Tugas Mandiri Bahasa Indonesia

1. Kalimat-kalimat berikut ini tidak dapat 2. Kalimat berikut ini termasuk kalimat
digunakan dalam karya ilmiah karena baku adalah…
penulisan tanda bacanya tidak mengikuti (A) Siswa yang gemar membaca yang
EYD, KECUALI memiliki ilmu pengetahuan yang
(A) Penggunaan Obat tradisional lebih luas dengan siswa yang belum
termasuk jamu yang makin marak gemar membaca
dewasa ini adalah kenyataan yang (B) Masyarakat yang belum siap
patut disyukuri. menerima kenaikan harga BBM.
(B) Kendati pemakaian jamu cukup (C) Mengetahui ada tidaknya hubungan
marak dan industri jamu juga antara aktivitas pergaulan dengan
berkembang, sungguh mengheran prestasi belajar siswa.
kalau ternyata produksi tanaman (D) Pertentangan-pertentangan yang
obat beberapa tahun belakangan menimbulkan ketegangan dunia
stagnan. yang terjadi karena nafsu berkuasa.
(C) Dalam catatan Kepala Badan POM (E) Untuk memperkaya pemahaman
Indonesia memiliki 30.00 jenis dan pengetahuannya, seorang
tumbuhan. penulis harus mengumpulkan data,
(D) Penurunan angka produksi menjadi informasi, atau pengetahuan
ironi, kalau kita bandingkan dengan tambahan yang berkaitan dengan
pasar yang sesungguhnya tema karangan.
membentang luas.
(E) Yang cukup menggembirakan, 3. Meskipun gejala flu burung sama
adalah kenyataan bahwa beberapa dengan gejala flu biasa menyerang
rumah sakit sudah memasukkan manusia, tetapi gejala flu burung
obat tradisional dalam terapi kepada cenderung lebih sering dan cepat
pasien. menjadi parah disbanding flu biasa.
Kalimat di atas dapat diperbaiki dengan
menghilangkan kata
(A) meskipun (D) lebih
(B) tetapi (E) sama
(C) dengan

Sony Sugema College 1


4. Kala keputusan Persebaya untuk mundur 6. Memelihara tanaman hias merupakan
dan tidak memainkan pertandingan modal dasar… mencintai lingkungan.
terakhir Delapan Besar Liga Djarun Tanaman hias tersebut selain sebagai
Indonesia, Persebaya dijatuhi skorsing penghias lingkungan, juga membuat
dua tahun tidak boleh mengikuti lingkungan menjadi teduh, asri, dan
kompetisi serta harus neniti kembali nyaman. …, banyak tanaman hias yang
langkah dari Devisi dua. digunakan sebagai penghias … peneduh
Inti kalimat panjang tersebut adalah… lingkungan.
(A) Persebaya mundur dan tidak Kata penghubung yang tepat untuk
memainkan pertandingan. melengkapi teks di atas adalah…
(B) Persebaya diskorsing selama dua (A) dari, hal ini, atau
tahun. (B) sehingga, ketika, dan
(C) Persebaya tidak boleh mengikuti (C) untuk, sekarang ini, serta
kompetisi (D) untuk, kala, maupun
(D) Keputusan Persebaya mundur dan (E) untuk, saat ini, dan
tidak memainkan pertandingan.
(E) Keputusan Persebaya 7. Di antara kalimat berikut ini terdapat
mengakibatkan mereka tidak boleh kalimat yang tidak memiliki kesejajaran
bertanding. bentuk, yaitu
(A) Pemakaman jenazah mantan
5. Sikap berbahasa yang positif dan pejabat itu akan dilakukan besok
kebiasaan berbahasa Indonesia dengan dan diberangkatkan besok dari
baik dan benar perlu …. dan …. di rumah duka sekitar pukul 10.00
kalangan masyarakat. WIB.
Pasangan kata yang sesuai untuk mengisi (B) Pemikir mengatur strategi yang
tempat kosong di atas adalah … . harus ditempuh dan pelaksana
(A) ditingkatkan – dikembangkan mengerjakan tugas dengan baik
(B) meningkat – berkembang serta penyandang dana
(C) meningkat – dikembangkan menyediakan biaya.
(D) dikembangkan – bertingkat (C) Petani itu tetap mengayunkan
(E) dikembangkan – meningkat cangkulnya walaupun tubuhnya
dibasahi oleh keringat.
(D) Tito luluis SNMPTN karena kerajinan
dan ketekunannya.
(E) Program kerja yang telah disusun
meliputi pengonsolidasian
organisasi, penertiban administrasi
dan keuangan, serta pembentukan
kader mubalig.

2 Sony Sugema College


Sony Sugema College
8. Urutkanlah enam kalimat berikut ini 10. …. Untuk tarif pelanggan rumah tangga,
sehingga menjadi paragraf yang baik ! harga yang harus dibayar oleh
1. Dalam hal ini, pemandu harus pelanggan Indonesia adalah sekitar 3,72
dilengkapi dengan kemampuan hingga 9,97sen dolar per kilowatt
memberi penjelasan secara rinci jamnya. Di Vietnam harganya jatuh di
kepada wisatawan. bawah angka itu, yaitu 1,62 hingga 5,48
2. dalam menjual pariwisata, terutama sen dolar. Sementara itu, harga listrik di
ketika masuk sentra-sentra Thailand hampir sama dengan di
kerajinan, peran pemandu sangat Vietnam. Yang paling murah adalah
besar. harga listrik di Laos, yaitu antara 0,13
3. mereka perlu penjelasan yang sampai 1,29 sen dolar. Tingginya harga
terperinci dan lugas tentang apa listrik di Indonesia ini perlu dikaji.
yang dilihatnya. Mengapa Indonesia tidak dapat menjual
4. dengan demikian, mereka akan listrik dengan harga murah seperti
merasa mendapatkan suatu negara-negara ASEAN lainnya, padahal
pengalaman yang luar biasa, jumlah pelanggan di Indonesia jauh lebih
seakan-akan ikut mengalami sendiri. banyak ?
5. wisatawan tidak hanya dibiarkan Kalimat yang tepat digunakan sebagai
melihat demontrasi proses produksi kalimat pertama paragraf di atas adalah
kerajinan tanpa penjelasan. ….
6. misalnya, ketika menyaksikan (A) Belum lama ini Bank Dunia
demontrasi proses pemintalan sutra, melaporkan harga listrik di beberapa
mereka harus diberi penjelasan negara ASEAN temasuk Indonesia,
tentang proses ini mulai dari Vietnam, Thailand, dan Laos.
pengadaan bahan hingga maksud (B) Menurut laporan Bank Dunia, harga
permintalan. jual listrik di Indonesia ternyata lebih
Urutan yang baik adalah … . mahal daripada tarif listrik di
(A) 3 – 4 – 5 – 1 – 2 – 6 beberapa negara ASEAN seperti
(B) 2 – 1 – 5 – 3 – 6 – 4 Vietnam, Thailand, dan Laos.
(C) 1 – 2 – 4 – 5 – 6 – 3 (C) Menurut Laporan Bank Dunia
(D) 4 – 5 – 3 – 2 – 1 – 6 disebutkan harga listrik di beberapa
(E) 2 – 1 – 4 – 3 – 5 – 6 negara ASEAN seperti Indonesia,
Vietnam, Thailand, dan Laos.
9. Kalimat yang mengandung cara (D) Menurut laporan Bank Dunia, Laos
melakukan perbuatan adalah … . merupakan negara anggota ASEAN
(A) Pemerintah mengumumkan harga yang harga listriknya paling murah
BBM baru itu tadi malam. dibandingkan dengan Vietnam,
(B) Perundingan itu akan berlangsung di Thailand, dan Indonesia.
Jakarta. (E) Dalam laporan Bank Dunia
(C) Dengan tegas, mereka menolak usul mengenai harga listrik di beberapa
itu. negara ASEAN disebutkan bahwa
(D) Mereka bersedia mengalah demi harga listrik di Vietnam dan Thailand
keutuhan organisasi. hampir sama.
(E) Tanpa dukungan anggota, program
ini tidak akan berhasil.
Sony Sugema College 3
Karena terseret Perang Dunia ke-2 12. Penulisan kata depan yang salah
dan Perang Pasifik, Indonesia diduduki Bala terdapat pada kalimat
Tentara Jepang dari tahun 1942 sampai (A) Banyak karya sastra Indonesia yang
1945. Pada tanggal 9 Maret 1942, tidak dipublikasi berupa buku, tetapi
Pemerintah Hindia Belanda menyerah tanpa di dunia maya (internet)baik yang
syarat kepada Jepang. Seluruh bekas daerah dikelola resmi oleh pemerintah,
Hindia Belanda berada di bawah kekuasaan organisasi non-profit maupun situs
Pemerintah Bala Tentara Dai Nipon. pribadi.
Gubernur Hindia Belanda terakhir, Tjarda van (B) Naiknya permukaan air laut telah
Starkenborgh, ditawan Jepang dan diangkut mendorong 100 orang penduduk
ke Formusa. sebuah pulau di Samudera Pasifik
Dengan didahului oleh ledakan bom berpindah ke lokasi yang lebih tinggi
atom di Hiroshima dan Nagasaki, Jepang (C) Mencairnya gletser dan gunung es di
menyerah kalah kepada Sekutu pada tanggal Kutub Utara menyebabkan banjir di
14 Agustus 1945. Pada tanggal 17 Agustus sebagian besar kawasan Samudra
1945, atas nama Bangsa Indonesia, Soekarno Pasifik
dan Hatta memproklamirkan kemerdekaan (D) Jumlah penduduk Jakarta lebih
Indonesia. Undang-Undang Dasar Negara besar dari pada jumlah penduduk
Republik Indonesia ditetapkan pada tanggal Bandung.
18 Agustus 1945. Sementara itu, tanggal 29 (E) Di antara ilmuwan yang mendukung
September 1945 tentara Sekutu yang teori heliosentris adalah Galileo.
pertama mendarat di Pulau Jawa dipimpin
oleh Letnan Jenderal Philip Christison. 13. Judul karya tulis: AKTIVITAS PERGAULAN
DAN PRESTASI BELAJAR SISWA ( Studi
11. Apabila kalimat ke-2 dan ke-3 alinea ke-1 Deskripsi tentang Kecerdasan Emosi dan
bacaan di atas dijadikan kalimat Intelektual Siswa SMA N 75 Solo Tahun
majemuk, kata penghubung yang Ajaran 2006-2007)
diperlukan adalah ... Masalah yang diteliti yang sesuai dengan
(A) dan judul tersebut adalah…
(B) kemudian (A) aktivitas pergaulan dan prestasi
(C) bahkan belajar siswa
(D) maka (B) kecerdasan emosi dan intelektual
(E) sehingga (C) kecerdasan emosi dan pergaulan
(D) kecerdasan emosi dan intelektual
siswa
(E) emosi dan intelektual SMA N 75 Solo

4 Sony Sugema College


Sony Sugema College
14. Dua hari yang lalu, telah digagalkan (E) Sebelum Von Koenigswald datang ke
penyelundupan satwa liar ke Jepang. Sangiran daerah Sangiran sudah
Belum lagi usai itu, menyusul dua orang terkenal di dunia ilmu pengetahuan.
Kuwait menyelundupkan 246 satwa liar
juga dari bandara Soekarno – Hatta 16. Dalam pemilihan ketua partai, Budi
pada pukul 23.30. mereka ditangkap dan Santosa memperoleh tujuh belas atau
diperiksa oleh yang berwajib. terpaut tujuh suara dari perolehan ketua
Kedua kalimat di atas dapat diungkapkan terpilih, sedangkan satu calon lainnya
kembali dengan kalimat … . hanya mendapat dua suara. Sementara
(A) Pemerintah harus membuat itu, dua suara menyatakan abstain dan
undang-undang tentang satu dinyatakan gugur.
penyelundupan. Berdasarkan isi bacaan di atas,
(B) Tidak ada manfaatnya melepas pernyataan berikut yang salah adalah …
penyelundup. (A) Dalam pemilihan itu, terdapat tiga
(C) Nasib malang bisa menimpa orang kandidat ketua partai.
yang tidak mau tahu aturan (B) Pemenang dalam pemilihan itu
tertentu. mendapatkan dua puluh empat
(D) Para penyelundup tidak harus suara.
ditahan karena ketidaktahuannya. (C) Jumlah suara yang diperoleh
(E) Penyelundup disiapkan menjadi pemenang lebih banyak daripada
orang yang tahu peraturan setelah gabungan jumlah suara yang
diperiksa. diperoleh calon yang gagal.
(D) Jumlah suara yang diperoleh
15. Seandainya Von Koenigswald tahun 1934 pemenang lebih sedikit daripada
tidak menginjakkan kakinya di bumi gabungan jumlah suara yang
Sangiran, situs manusia purba ini diperoleh calon yang gagal.
mungkin tidak akan setenar sekarang. (E) Pemenang dalam pemilihan itu
Mengapa ? Karena sejak kunjungan itu, mendapatkan suara lebih dari
nama Sangiran muncul dalam ranah ilmu separo jumlah pemilih.
pengetahuan sebagai situs penemuan
alat batu. 17. Awalnya, Pulau Jawa terapung-apung di
Pernyataan berikut ini sesuai dengan samudra. Seperti sepotong gabus, ia
maksud yang terkandung dalam kutipan dihempas gelombang, terlempar ke
di atas KECUALI … . sana-sini. Agar pulau itu tetap bertahan
(A) Sejak kedatangan Von Koenigswald di tempatnya, dibutuhkan sebuah paku
ke Sangiran, daerah Sangiran untuk menancapkannya ke bumi. Paku
menjadi terkenal. itu berupa Bukit Tidar yang terletak di
(B) Sangiran terkenal karena situs Magelang.
manusia purba. Bacaan di atas merupakan contoh … .
(C) Sejak tahun 1934, Sangiran dikenal (A) narasi
sebagai daerah situs manusia purba. (B) deskripsi
(D) Von Koenigswald mengunjungi (C) eksposisi
Sangiran tahun 1934 ketika daerah (D) argumentasi
itu belum terkenal. (E) persuasi

Sony Sugema College 5


18. “Kasus korupsi yang melibatkan mantan
pejabat itu dipetieskan oleh penyidik.”
Istilah dipetieskan bermakna …
(A) tidak dilanjutkan kasusnya
(B) tidak diumumkan
(C) tidak ditanggapi
(D) diproses
(E) dimasukkan ke dalam peti

19. ‘Sepeda motor tua keluaran tahun 70-an


itu terseok-seok dan terengah-engah
menelusuri jalan berbatu dan
menanjak.”
Majas yang terkandung dalam kalimat di
atas adalah …
(A) litotes
(B) metaphor
(C) pleonasme
(D) simile
(E) personifikasi

20. Kalimat yang mengandung kata


bermakna konotatif terdapat dalam
(A) Keluarga itu sepertinya mendapat
durian runtuh. Kehidupan mereka
berubah.
(B) Para pembeli menyukai durian yang
matang pohon.
(C) Durian bangkok selalu menjadi
unggulan di kelasnya.
(D) Harga durian mengalami fluktuasi.
(E) Kebanyakan makan durian, tidak
baik bagi kesehatan.

6 Sony Sugema College


Sony Sugema College
2. Tugas Mandiri Bahasa Indonesia

Ada kecenderungan semakin (D) Pada saatnya nanti, sampah


memasuki kehidupan modern sampah yang anorganik akan mendominasi,
dibuang lebih banyak sampah yang tergolong selaras dengan perubahan pola
sampah anorganik atau sampah yang tidak hidup masyarakat kota.
membusuk. Di kota-kota di Amerika Serikat (E) Volume sampah warga Jakarta jauh
yang sudah lebih maju daripada kota di lebih banyak daripada kota-kota di
Indonesia, misalnya 5 persen sampahnya Amerika Serikat.
adalah kertas. Sekitar 22 persen adalah
sampah sisa makanan, 10 persen kayu dan 2. Berdasarkan teknik penyajiannya,
daun-daunan dari taman, 10 persen sampah wacana di atas berbentuk karangan
kulit, plastik, dan karet, sekitar 9 persen adalah
logam, dan 9 persen lainnya beling. (A) narasi
Bandingkan dengan komposisi (B) deskripsi
sampah Jakarta. Menurut data BPS sampah (C) eksposisi
kertas Jakarta tahun 1989 – 1990 sekitar 8, (D) persuasi
28 persen, sampah kayu hanya 3,77 persen, (E) argumentasi
sampah kain, karet, kulit tiruan, dan plastik
9,16 persen, logam hanya 2,08 persen, dan 3. Pola kalimat kedua paragraf satu teks
gelas atau kaca hanya 1,77 persen. Jumlah bacaan di atas sama dengan
tertinggi masih merupakan bahan organik, (A) Bunga raya sangat bagus untuk
termasuk didalamnya sisa makanan yang dirangkai.
bisa membusuk, yaitu 73,99 persen. Lalu apa (B) Pil anti malaria sangat dibutuhkan di
yang bisa dikerjakan warga Jakarta untuk daerah Irian Jaya.
mengurangi volume sampah? (C) Hukum adalah peraturan yang
1. Pernyataan berikut ini sesuai dengan dibuat oleh suatu kekuasaan yang
teks tersebut, KECUALI berlaku untuk orang banyak serta
(A) Diperkirakan sampah organik di bersifat mengikat dan memaksa.
Jakarta akan lebih pada masa (D) Pasukan anti-AS bersiaga penuh di
mendatang. perbatasan.
(B) Pada umumnya, sampah organik (E) Pemerintah mengecam pengunjuk
menduduki peringkat teratas dalam rasa yang berdemontrasi.
hal persentase.
(C) Ada korelasi antara gaya hidup dan
jenis sampah yang dihasilkan.

Sony Sugema College 7


4. Menurut data BPS sampah kertas (E) Khodijah, Lili, Dasril dan Wido
Jakarta tahun 1989 – 1990 sekitar 8, 28 besahabat sejak dibangku SMA.
persen, sampah kayu hanya 3,77 persen,
sampah kain, karet, kulit tiruan, dan 7. Penggunaan tanda baca yang mengikuti
plastik 9,16 persen, logam hanya 2,08 EYD terdapat dalam kalimat
persen, dan gelas, atau kaca hanya 1,77 (A) Dr. Ir. Tjokorda Raka Sukawati,
persen. berhasil menemukan teknik baru
Kalimat di atas akan menjadi kalimat landasan putar untuk menggeser
baku jika ejaannya diperbaiki sbb beton ratusan ton.
(A) setelah kata Jakarta diberi tanda (B) Indonesia adalah negara yang cinta
koma (,) damai, dan tidak menyukai
(B) penulisan persen seharusnya prosen peperangan.
(C) setelah kata dan tidak perlu (C) Dalam era serba tanpa kabel seperti
memakai tanda koma(,) saat ini suatu kawasan berteknologi
(D) setelah kata BPS harus memakai tinggi menjadi hal mutlak untuk
tanda koma (,) dikembangkan.
(E) setelah kata persen tidak perlu (D) Terumbu karang merupakan
tanda koma ekosistem vital, dinamis, bertingkat
biodiversitas dan berproduktivitas
5. Judul yang tepat untuk teks di atas tinggi, serta memiliki peran yang
adalah cukup nyata.
(A) Sampah di Kota Besar (E) Apakah faktor utama, penyebab
(B) Masalah Sampah di Kota Besar terjadinya pencemaran laut di
(C) Persentase Sampah Jakarta Indonesia?
(D) Sampah Organik dan Anorganik
(E) Sampah Anorganik Kota Jakarta 8. Pencemaran laut membawa dampak
yang hebat bagi kelangsungan dan
6. Pemakaian tanda baca yang benar kelestarian … atau keanekaragaman
terdapat pada kalimat hayati laut. Sebagai contoh adalah
(A) Menurut Kamus Besar Bahasa pencemaran akibat polusi kimiawi. Polusi
Indonesia, edit adalah kimiawi yang dibawa dan diproses …
mempersiapkan naskah yang siap berlangsung begitu cepat dan tidak
cetak atau terbit dengan terkendal. Polusi kimiawi ini
memperhatikan ejaan, diksi, dan menyebabkan kemunduran kuantitas
struktur kalimat. jenis … tertentu.
(B) Jika tujuan penulisan sudah jelas Kata serapan yang baku untuk
Anda dapat menentukan bahan melengkapi paragraf di atas adalah
penulisan, jenis, dan luasnya. (A) ekosistem, industrialisasi, biota
(C) Perkembangan produksi gula (B) ekosistem, industri, biota
Indonesia dari tahun 2000 s / d 2006 (C) habitat, industrialisasi, spesies
terus meningkat. (D) habitat, industri, spesies
(D) Dengan langkah baru, seperti yang (E) habitat, produksi, biota
diumumkan oleh Presiden Bush
para koruptor akan disiarkan ke
publik dan dihukum.

8 Sony Sugema College


Sony Sugema College
9. Daerah Istimewa Yogyakarta merupakan (D) Sebagian karyawan SSC mendapat
provinsi dengan luas wilayah yang penghargaan dari Pak Sony Sugema.
sempit di Indonesia, yaini 3.200 km2. (E) Pemukiman penduduk di daerah
Jumlah penduduk pada tahun 1988 Bandung Timur mulai tergenang air.
sebanyak 3.081.249, dengan
pertumbuhan penduduk 1,25%. Besarnya 11. Ekspor kopi Lampung ke Timur Tengah
minat belajar dan mahasiswa dari kini menghadapi hambatan serius akibat
berbagai provinsi untuk belajar di DIY adanya kebijakan baru tentang dokumen
menyebabkan jumlah penduduk DIY ekspor yang tidak lazim dalam
terus bertambah dari tahun ke tahun. perdagangan internasional.
Pada tahun 1997, jumlah penduduk DIY Kalimat di atas merupakan perluasan
melonjak tinggi hingga mencapai jumlah dari kalimat inti
tidak kurang dari 3,5 juta orang. Yang (A) Ekspor kopi Lampung tidak lazim
menyebabkan jumlah penduduk DIY dalam perdagangan internasional.
terus bertambah dari tahun ke tahun (B) Ekspor kopi Lampung menghadapi
pada kutipan adalah hambatan.
(A) minat pelajar dan mahasiswa dari (C) Ekspor kopi ke Timur Tengah dari
berbagai provinsi untuk mendapat Lampung.
pelajaran di DIY (D) Ekspor kopi Lampung terhambat
(B) keadaan Yogyakarta yang karena ada kebijakan baru tentang
merupakan kota pelajar dan kota dokumen ekspor.
mahasiswa (E) Ekspor kopi ke Lampung
(C) hidup di Yogyakarta murah dan menghadapi hambatan.
masyarakat Yogyakarta ramah
(D) besarnya minat pelajar dan 12. Data yang digunakan untuk menjawab
mahasiswa semua permasalahan yang ada dalam
(E) besarnya minat pelajar dan penelitian ini dapat dipilah menjadi dua,
mahasiswa dari berbagai provinsi yaitu data utama dan data penunjang.
untuk belajar di Yogyakarta Agar menjadi kalimat efektif, di atas
dapat diperbaiki menjadi
10. Imbuhan peN-an pada kalimat (A) Data penelitian ini dipilah menjadi
Pencemaran perairan sungai atau dua dua, yaitu data utama dan data
limbah akan menurunkan kualitas air penunjang.
sungai. Bermakna tidak sama dengan (B) Data untuk menjawab
peN-an dalam kalimat permasalahan dalam penelitian ini
(A) Menurut pendengaran saya, dapat dipilah menjadi dua, yaitu
Robinson termasuk pekerja yang data utama dan data penunjang.
rajin. (C) Data yang digunakan dalam
(B) Sebagai pemimpin yang baik ia penelitian ini dapat dipilah menjadi
terkenal dengan pendiriannya yang dua, yaitu data utama dan data
kuat. penunjang.
(C) Pemandangan puncuk Gunung (D) Data ada dua kelompok, yaitu data
Merbabu sangat menarik. utama dan data penunjang.
(E) Data terdapat dua jenis,utama dan
penunjang.
Sony Sugema College 9
13. Informasi yang tersiar dalam … ini (C) Pada dasarnya berita yang
memuat pula nilai-nilai budaya yang sesungguhnya berawal dari isu yang
tidak seluruhnya sesuai dengan … berkembang di masyarakat.
Pancasila.. (D) Tidak ada jeleknya kita
Kata yang tepat untuk mengisi bagian mempercayai isu yang berkembang
yang kosong kalimat di atas adalah di masyarakat.
(A) umum, falsafah (E) Informasi yang berkembang di
(B) globalisasi, idelogi masyarakat ada baiknya dicermati
(C) era teknologi, ide agar tidak merugikan pihak-pihak
(D) glabalisasi, karakter tertentu.
(E) internet, falsafah
16. Sebuah pabrik kue mencoba
14. Stasiun-stasiun televisi swasta yang memproduksi jenis kue yang agak lain
menampilkan acara infotainment dari biasanya, yakni kue yang rasanya
memang telah menjadikannya sebuah sangat enak, aromanya wangi,
paket hiburan yang selalu sarat dengan bentuknya artistik, dan dibungkus dalam
peristiwa-peristiwa spektakuler. kemasan yang memikat. Kue ini dijual
Makna kata spektakuler pada kalimat di dengan harga mahal, tetapi anehnya
atas adalah sangat laku dijual di Bandung,
(A) menarik perhatian Semarang, Surabaya sehingga
(B) bersifat menggemparkan perusahaan mempunyai untung
(C) bersifat menyedihkan besar.Berdasarkan pengalaman ini
(D) membuat bahagia pemilik kue berkesimpulan bahwa untuk
(E) membuat perasaan terharu konsumsi orang-orang yang tinggal di
kota-kota besar lainnya, seperti Jakarta
15. Beberapa tahun yang lalu isu makanan dan Medan perlu diproduksi kue sejenis
yang berformalin sempat menjadi karena selera mereka tidak akan
pembicaraan masyarakat. Sempat pula berbeda jauh.
menjadi berita-berita hangat di media- Hal utama yang dibicarakan dalam teks
media. Masyarakat untuk beberapa saat di atas adalah
menghindari makanan yang dianggapp (A) Produksi pabrik kue yang unik.
mengandung formalin seperti tahu, (B) Pemilik kue beranggapan bahwa kue
tempe, ikan asin, ataupun ikan basah. yang dijual harus bervariasi.
Akibatnya, beberapa produsen tahu (C) Pabrik kue memproduksi macam-
mengalami keterpurukan usaha. macam kue.
Pendapat yang sesuai dengan isi tajuk (D) Kue yang dijual dengan harga mahal.
tersebut adalah (E) Selera orang-orang yang tinggal di
(A) Informasi yang menyangkut kota besar pada dasarnya sama.
makanan berformalin tidak perlu
kita percayai agar tidak merugikan
pihak-pihak tertentu.
(B) Kita perlu menyikapi isu itu dengan
sungguh-sungguh.

10 Sony Sugema College


Sony Sugema College
17. Kecepatan pertambahan penduduk 2,3% 19. Di antara kalimat berikut ini yang paling
per tahun, menunjukan jumlah penduduk tepat untuk memorandum berupa
akan berganda dua kali lipat sesudah 30 pemberian informasi adalah
tahun, apabila angka pertambahan (A) Anggota OSIS segera berkumpul di
penduduk tidak berubah. Memang aula pengarahan kepala sekolah
Indonesia masih mempunyai wilayah- dalam persiapan pelepasan kelas III.
wilayah yang kurang penghuninya. (B) Anggota OSIS jangan pulang dulu
Meskipun demikian, untuk memudahkan ada pengarahan kepala sekolah
penduduk dari wilayah yang padat ke berhubung persiapan pelepasan
wilayah yang kurang padat kelas III.
membutuhkan waktu, biaya, dan sarana (C) Dengan surat ini, saya beri tahukan
Inti paragraf di atas adalah bahwa anak OSIS akan diberi
(A) Wilayah Indonesia masih kurang pengarahan oleh kepala sekolah
penduduk. sehubungan dengan persiapan
(B) Penduduk Indonesia bertambah pelepasan kelas III.
2,3% per tahun. (D) Kepala sekolah akan memberi
(C) Kecepatan pertambahan penduduk pengarahan kepada anggota OSIS
Indonesia akan berganda. dalam rangka persiapan pelepasan
(D) Pertambahan penduduk Indonesia kelas III.
akan berganda setiap tahun. (E) Harap Saudara kumpulkan anggota
(E) Penduduk Indonesia bertambah OSIS tanggal 10 Juli 2007, pukul
2,3% per tahun dan akan menjadi 15.00, rapat persiapan pelepasan
berlipat ganda . kelas III, di Aula. Terima kasih.
20. Membaca dimulai dengan melihat.
18. Kata serapan yang berasal dari bahasa Stimulus masuk lewat indera
asing digungakan secara tepat dalam penglihatan, mata. Pada tingkat awal,
kalimat di bawah ini, KECUALI anak menunjukan kemampuan yang
(A) Karena sentimentalitas yang tidak disebut membaca. Pada saat permulaan
terkendali, timbul perselisihan itu anak mulai sadar bahwa tanda
antara kedua sahabat itu. lambang tertentu menunjukan nama
(B) Penurunan harga salam hanya atau benda. Kemudian, mereka belajar
disebabkan oleh setimen pasar. bahwa jika lambing-lambang itu
(C) Saat ini sedang diusahkan sentralitas dirangkai akan tersusun pembicaraan.
tanaman tebu di sekitar pabrik gula. Kapan anak-anak itu siap membaca
(D) Barang-barang plastik cocok sebagai buku?
substitusi aneka barang dari logam. Informasi penting yang terdapat dalam
(E) Segala sesuatu hendaknya dibahas paragraf di atas adalah
secara objektif jangan secara (A) Stimulus penglihatan adalah mata.
subjektif (B) Membaca itu dimulai dengan
melihat.
(C) Belajar membaca sebaiknya pada
waktu kecil.
(D) Membaca adalah merangkaikan
kata-kata.
(E) Anak-anak siap untuk membaca.
Sony Sugema College 11
21. Urutan lima kalimat berikut sehingga (C) Karena itu, janganlah berbicara
menjadi paragraf yang utuh. sembarangan agar orang lain tidak
1. Bangunan rumah yang padat dan tersinggung.
jalan masuk hanya satu meter (D) Jadi, mengenal seseorang sama
2. Kondisi warga juga kurang sehat halnya dengan mengenal sebuah
dan acuh dengan kondisi kumuh itu. buku.
3. Sampah rumah tangga dibiarkan (E) Sebaiknya, kita teliti terlebih dahulu
begitu saja. sebelum memutuskan untuk
4. Jemuran pakaian dipasang tanpa membeli buku tsb .
aturan.
5. selokan digenangi air. 23. ... jatah minyak tanah satu orang
6. Kondisi permukiman kumuh di sebanyak 5 liter dengan harga Rp.
Jakarta sangat memprihatinkan 2.400,00 per liter. Setiap pangkalan
7. bak-bak sampah dipenuhi barang minyak tanah mendapatkan jatah 5000
yang tidak terpakai. liter. Namun, setiap harinya ada 500
Rangkaian yang tepat dari lima kalimat di orang yang membeli di kami.
atas adalah Makna dari kalimat ini pada wacana di
(A) 3-1-4-5-6-7-2 atas adalah ...
(B) 3-2-1-4-5-6-7 (A) Kelurahan Citeureup dan Kelurahan
(C) 6-1-3-4-5-7-2 Puspanegara mengalami kekurangan
(D) 6-1-3-4-2-5-7 minyak tanah
(E) 3-4-2-5-1-7-6 (B) Kelurahan Citeureup dan Kelurahan
Puspanegara sebetulnya tidak
22. Bisa saja kita tidak suka sebuah buku kekurangan minyak tanah
sewaktu melihatnya. Namun, cobalah (C) Kecamatan Citeureup pada umumnya
baca dulu dengan cermat dan analis isi tidak kekurangan minyak tanah
yang terkandung di dalam nya. Mungkin (D) Kecamatan Citeureup mengalami
rasa tidak suka akan berubah menjadi kekurangan jatah minyak tanah
suka. Begitu pula dengan manusia, kita (E) Kecamatan Citeureup melakukan
jangan cepat mengambil kesimpulan pengurangan jatah minyak tanah
bahwa orang itu tidak menyenangkan.
Mungkin setelah kita ajak “ngobrol”,
berandai-andai, rasa antipati akan
berubah menjadi simpati.
Kalimat berikut ini yang tepat untuk
mengakhiri paragraf analogi di atas
adalah
(A) Oleh karena itu, kita harus hati-hati
berbicara sebelum mengenal orang
tersebut.
(B) Dengan demikian, kita dapat
membedakan mengenal seseorang
dengan sebuah buku.

12 Sony Sugema College


Sony Sugema College
24. Kemiskinan yang melanda Indonesia Kalimat pertama pada bacaan diatas
dewasa ini karena krisis moneter dan adalah ...
ekonomi yang terjadi pada (A) kalimat inti
pertengangan Juli 1997, akibat (B) kalimat pembuka
berkurangnya kepercayaan masyarakat (C) kalimat topik
kepada pemerintah (kegagalan (D) kalimat transformasi
pemerintah) membawa konsekuensi (E) kalimat permulaan
perlunya dilanjutkan pembangunan
nasional 25. Judul yang tepat untuk bacaan di atas
Pembangunan yang dilaksanakan ialah ...
oleh Orde Baru yang menggantikan Orde (A) Pembiayaan Pembangunan Swasta
Lama (1945-1965), sebenarnya dapat Dan Pemerintah
dikatakan cukup berhasil membawa (B) Kimiskinan Di Indonesia
Indonesia ke peringkat Negara (C) Peningkatan Pendapatan Per Kapita
berpenghasilan menengah dengan Masyarakat
penghasilan per kapita $1155 pada (D) Upaya Pembangunan Pedesaan
tahun 1996 dari $60 pada tahun 1966 (E) Pembangunan Untuk Mengatasi
dan inflasi dari 600 persen per tahun ke 5 Kemiskinan
persen per tahun, serta masyarakat
miskin hanya 11 persen penduduk dari 26. Kata investasi pada alinea ketiga dalam
sekitar 60 persen. Namun karena krisis, arti ekonomi mengandung makna ...
Indonesia terpuruk menjadi negara (A) pembiayaan pembangunan
berpenghasilan rendah lagi dengan (B) kebijakan moneter
penghasilan per kapita $ 400 per tahun, (C) anggaran pendapatan dan belanja
inflasi sampai 40 -60 persen, dan negara
masyarakat miskin 24 persen dari (D) penanaman modal
penduduk atau sekitar 49,5 juta jiwa (E) tabungan negara
(17,6 juta jiwa di perkotaan dan 31,9
juta jiwa di pedesaan).
Selama ini pembangunan di
Indonesia dilaksanakan oleh swasta dan
pemerintah. Pada Pelita VI direncanakan
investasi sebesar Rp 815 triliun dengan
77 persen ditangani swasta dan 23
persen ditangani pemerintah.
Pembiayaan pembangunan yang
dilakukan oleh pemerintah biasanya
direncanakan melalui Rencana Anggaran
Penerimaan dan Belanja Negara
(RAPBN). Prinsip dasar di dalam
pembiayaan pembangunan itu bertalian
dengan konsep dasar financial, proses
financial dan mekanisme financial.

Sony Sugema College 13


3. Tugas Mandiri Bahasa Indonesia

Realitas kemiskinan tidaklah (C) Kemiskinan tidak identik dengan


sederhana. Apakah kriteria yang tepat untuk tingkat biaya konsumsi.
kemiskinan dan siapakah yang termasuk di (D) Tingkat biaya konsumsi rendah
dalamnya? Hal itu tidak dapat dipahami mencerminkan tingkat kemiskinan.
hanya dari sederet angka tentang seberapa (E) Jumlah penduduk miskin belum
besar jumlah orang yang tingkat biaya menggambarkan kemiskinan itu
konsumsinya tidak mencapai garis sendiri
kemiskinan. Kenyataan yang ada justru jauh
lebih sulit. 2. Pernyataan berikut yang sesuai dengan
Biro Pusat Statistik DKI sendiri isi bacaan di atas adalah
memberikan indikator penduduk miskin, (A) Menurut Badan Pusat Statistik
yakni penduduk dengan rumah berlantai Nasional, orang yang belanjanya
tanah, luas rumah kurang dari delapan meter setara dengan 2.100 kalori sehari
persegi, pola makanan tidak ganti-ganti, dan termasuk berada di garis
tidak mampu membeli pakaian baru. Badan kemiskinan.
Pusat Statistik (nasional) mendefinisikan (B) Program Pembangunan Perserikatan
garis kemiskinan dari besarnya rupiah yang Bangsa-Bangsa memberikan batasan
dibelanjakan untuk memenuhi kebutuhan bahwa penduduk miskin adalah
konsumsi setara dengan 2.100 kalori per orang yang penghasilannya dua
kapita per hari, di tambah kebutuhan pokok doler per hari.
lainnya, seperti sandang, perumahan, (C) Badan Pusat Statistik Nasional dan
kesehatan, dan pendidikan. Sementara itu, UNDP memberikana batasan yang
Progran Pembangunan Perserikatan Bangsa- sama tentang penduduk miskin,
Bangsa (UNDP) mendefinisikannya sebagai yakni dari segi pendapatan.
orang yang bekerja dengan pendapatan dua (D) Biro Pusat Statistik DKI menjelaskan
dolar AS atau sekitar Rp 17.000,00 per hari. bahwa penduduk miskin adalah
orang yang tidak memiliki
1. Yang dimaksud dengan pernyataan Hal penghasilan sehingga tidak dapat
itu tidak dapat dipahami hanya dari membeli baju baru.
sederet angka tentang seberapa besar (E) Kondisi kehidupan penduduk miskin
jumlah orang yang tingkat biaya digambarkan secara jelas oleh
konsumsinya tidak mencapai garis Badan Pusat Statistik Nasional.
kemiskinan
(paragraf I, kalimat 3) adalah
(A) Kemiskinan harus dipahami secara
lebih mendalam.
(B) Deretan angka tentang kemiskinan
sulit sekali dipahami.
14 Sony Sugema College
Sony Sugema College
3. Masalah pokok yang dibahas dalam 6. Pola kalimat saat ini perhatian
bacaan di atas adalah masyarakat terhadap perawatan kulit,
(A) indikator kemiskinan terutama kaum wanita, semakin
(B) batasan kemiskinan meningkat sama dengan
(C) tanda-tanda kemiskinan (A) Meskipun terkesan sentimental,
(D) ciri-ciri kemiskinan Irwan memiliki ciri khas dalam
(E) istilah kemiskinan karangannya.
(B) Sekarang merokok merupakan
4. Kenyataan yang ada justru jauh lebih kebutuhab seorang baik laki-laki
sulit (kalimat 4 paragraf 1). Pernyataan maupun wanita.
tersebut memiliki arti bahwa (C) Dulu perempuan dianggap tidak
(A) Menghitung jumlah orang yang memiliki sopan santun jika mereka
berpenghasilan rendah sangat sulit. merokok di depan umum.
(B) Jumlah orang yang berpenghasilan (D) Untuk meningkatkan pendapatan,
tidak mencapai garis kemiskinan kita harus pandai mencari peluang.
lebih banyak daripada jumlah angka (E) Lima tahun ke depan kondisi sumber
yang tercatat pada data staristik. daya hutan Indonesia sangat
(C) Angka statistik tentang jumlah mengkhawatirkan.
penduduk yang berpenghasilan
kurang belum cukup untuk 7. Semua kata ditulis secara benar terdapat
menggambarkan kriteria dalam kalimat
kemiskinan. (A) Di Iran, sepak bola perempuan
(D) Orang yang berpenghasilan dari hanya boleh dilakukan dilapangan
biaya konsumsinya setiap hari tertutup.
belum tentu termasuk orang yang (B) Penonton sepakbola perempuan di
berada di bawah kemiskinan. Iranpun hanya kaum perempuan.
(E) Jumlah orang yang tingkat biaya (C) Penada tanganan perjanjian kerja
konsumsinya tidak mencapai garis sama akan dilakukan minggu ini.
kemiskinan dapat dilihat pada (D) Pendidikan prasekolah harus lebih
sederetan angka statistik diperhatikan.
(E) Pengurus PSSI belum dapat
5. Simpulan bacaan di atas adalah menindak lanjuti perubahan jadwal
(A) Jumlah penduduk miskin di pertandingan.
Indonesia sangat banyak.
(B) Penduduk miskin di Indonesia
menimbulkan masalah baru.
(C) Dalam kehidupan penduduk miskin,
tersimpan misteri yang sukit
dipecahkan.
(D) Menentukan kriteria kemiskinan
bukanlah masalah yang mudah.
(E) Menentukan ktriteria kemiskinan
menjadi tanggung jawab Biro
Statistik.

Sony Sugema College 15


8. Kalau bukan agen, pihak maskapai tidak 10. PU : semua negara yang cinta damai
akan melayani … tiket. Semua maskapai tidakmenyukai peperangan
membantah bahwa … terjadi di PK : Indonesia adalah Negara yang cinta
maskapainya sebab mereka memiliki damai
sistem yang tidak memungkinkan K ::…
adanya tiket kosong untuk …. Pernyataan yang tepat untuk melengkapi
Kata yang tepat untuk mengisi kalimat di K (simpulan) di atas adalah… adalah
atas adalah (A) Indonesia termasuk Negara yang
(A) pembelian, percaloan, tidak menyukai peperangan.
diperjualbelikan (B) Indonesia sangat tidak menyukai
(B) perjualbelian, pencaloan, peperangan.
memperjualbelikan (C) Indonesia tidak menyukai
(C) pembelian, pencaloan, peperangan.
diperjualbelikan (D) Indonesia cinta damai dan tidak
(D) penjualbelian, percaloan, menyukai peperangan.
diperjualkan (E) Indonesia adalah Negara yang cinta
(E) pembelian, percaloan, damai dan tidakl menyukai
diperjualbelikan peperangan.

9. Makanan utama rayap adalah kayu atau 11. 1. Dia mengemudi mobil
bahan yang kandungan utamanya terdiri 2. mobil itu dibuat tahun 1980
atas selulosa. Dari perilaku makanan 3. suasana begitu indah dalam kenangan
yang demikian dapat ditarik kesimpulan Penggabungan kalimat tunggal tersebut
bahwa …. menjadi kalimat majemuk yang tepat
Pernyataan yang lengkap untuk adalah
melengkapi kalimat di atas adalah (A) Dia mengemudi mobil, dibuat pada
(A) rayap adalah makhluk hidup yang tahun 1980 dan suasana indah
bermanfaat dalam kehidupan kita. dalam kenangan.
(B) rayap makhluk hidup yang (B) Dia mengemudi mobil dan mobil itu
mengurangi kebutuhan selulosa kita. dibuat tahun 1980 pada saat
(C) rayap adalah makhluk hidup yang suasana indah dalam kenangan.
terdapat dalam setiap bangunan (C) Dia yang mengemudikan mobil yang
rumah. dibuat tahun 1980 saat suasana
(D) rayap adalah makhluk hidup yang indah dalam kenangan.
mengganti kayu menjadi selulosa. (D) Dia mengemudikan mobil yang
(E) rayap adalah makhluk hidup dibuat tahun 1980 saat suasana
pemakan bahan yang penting dalam begitu indah dalam kenangan.
kehidupan kita. (E) Dia mengemudikan mobil yang
dibuat tahun 1980 sehingga suasana
terasa begitu indah dalam
kenangan.

16 Sony Sugema College


Sony Sugema College
12. 1. Kampanye pemilihan presiden adalah 15. Kami sangat … generasi muda yang
proses penyampaian pesan tentang banyak … perjuangan para pahlawan.
sosok calon presiden. Kata yang tepat untuk melengkapi
2. Pesan itu diharapkan publik akan kalimat tersebut adalah
memiliki gambaran siapa dan (A) bangga – mengetahui tentang
bagaimana calon presiden yang menjadi (B) membanggakan – mengetahui pada
pilihannya. (C) bangga pada – mengetahui akan
Kata penghubung yang paling tepat (D) membanggakan pada – tahu bahwa
untuk menggabungkan kedua kalimat di (E) membanggakan – tahu akan
atas adalah
16. Bila dilihat dari karier tinjunya,
(A) sehingga (D) dengan demikian
kehebatan Mike
(B) tetapi (E) berhubung dengan
Tyson yang kini berganti nama menjadi
(C) dengan
Malik Abdul Aziz setelah masuk Islam,
rasanya tidak diragukan lagi. Dari
13. Kata berimbuhan bercetak miring
catatan prestasinya diperoleh data
berikut yang digunakan secara tidak
sebagai berikut. Selama 45 kali
tepat terdapat pada kalimat
bertanding, Mike Tyson memenangkan
(A) Industri otomotif terus memasarkan
pertandingan 79 persen, dengan menang
model dan jenis-jenis baru.
KO 86 persen. Jumlah ronde
(B) Dari penjualan mobil baru, dia
pertandingan 164 ronde rata-rata ronde
mendapatkan laba sebesar tiga
pertandingan 3,64 persen. Jumlah
puluh persen.
pendapatan per detik di partai terakhir
(C) Bisnis otomotif roda empat memang
Rp 168 juta. Mike Tyson meraih gelar
masih menggiurkan.
WBC, WBA, dan IBF.
(D) Para pengusaha menargetkan
Simpulan generalisasi di atas yang tepat
penjualan 50 – 60 unit per bulan.
adalah
(E) Perbankan memberikan suku bunga
(A) Dari catatan di atas, rasanya tidak
kredit di kisaran tujuh persen.
berlebihan kalau Mike Tyson pasti
menang di dalam semua
14. Dia selalu … ketika akan menghadapi
pertandingan
lomba. Apalagi kalau sudah melihat
(B) Dari catatan di atas, rasanya tidak
peserta lain walaupun para pelatih
berlebihan kalau Mike Tyson
sudah memberi arahaan menang atau
menjadi orang yang paling terkenal
kalah dalam berlomba adalah sesuatu
di dunia.
yang biasa.
(C) Dari catatan di atas, rasanya tidak
Ungkapan yang tepat untuk mengisi
berlebihan kalau Mike Tyson
bagian yang rumpang pada teks di atas
bergelar petinju terbaik dan sejati.
adalah…
(D) Dari catatan di atas, rasanya tidak
(A) tinggi hati (D) sempit hati
berlebihan kalau Mike Tyson
(B) besar hati (E) kecil hati
menjadi orang sombong karena
(C) lemah hati
terkaya di dunia.
(E) Dari catatan di atas, rasanya tidak
berlebihan kalau Mike Tyson
menjadi petinju yang berani dan
handal.
Sony Sugema College 17
17. Kedisiplinan para siswa dapat 19. Selain melakukan operasi karcis, PT
ditingkatkan Kereta Api Divisi Jabotabek juga
melalui beberapa upaya, antara lain menertibkan pedagang asongan di
melalui kegiatan ekstrakurikuler, seperti dalam kereta serta pedagang kaki lima
upacara bendera, latihan baris-berbaris, yang berjualan di stasiun kereta api,
kepramukaan, dan kegiatan paskibra. pada hari Rabu pagi dan malam.
Tanggapan yang baik sesuai dengan Sebanyak 40 pedagang asongan dan
pernyataan di atas adalah belasan penumpang tanpa karcis
(A) Disiplin sangat diperlukan untuk terjaring dalam operasi di stasiun
membina mentalitas yang tangguh. Isi pokok bacaan di atas adalah
(B) Dengan jiwa yang disiplin, maka (A) operasi penertiban oleh PT Kereta
terbentuklah jiwa yang tangguh. Api Divisi Jabotabek
(C) Kaum remaja khususnya pelajar (B) pedagang kaki lima dan asongan
umumnya mempunyai semangat terjaring
dan disiplin yang tinggi dalam (C) penumpang yang tidak membayar
berbagai kegiatan ekstrakurikuler. terkena operasi
(D) Kedisiplinan sebenarnya akan (D) hanya 40 penjual dan segelintir
terbentuk langsung secara alami penumpang tanpa kercis yang
(E) Melalui kegiatan ekstrakurikuler terkena operasi
itulah akan terbina kedisiplinan yang (E) PT Kereta Api Divisi Jabotabek
matang dan semangat saling menertibkan penumpang, pedagang
menghargai. kaki lima, dan asongan

18. Kata serapan yang berasal dari bahasa 20. Puluhan mahasiswa yang tergabung
asing digunakan secara tepat dalam dalam Kesatuan Aksi Mahasiswa Muslim
kalimat Indonesia (KAMMI) dan Ikatan
(A) Para wakil rakyat yang duduk di DPR Mahasiswa Muhammadiyah (IMM)
harus dapat menyampaikan inspirasi Surabaya menggelar unjuk rasa di depan
masyarakat kepada pemerintah. gedung Grahadi. Mereka menggelar
(B) Tampaknya isi pasal-pasal dalam spanduk dan poster yang berisi imbauan
perundangan itu tidak efektif untuk untuk meninggalkan kemaksiatan.
menanggulangi kejahatan. Setelah puas menggelar orasi. Massa
(C) Kami tidak suka membeli barang- bergerak menuju depan Plaza Tunjungan
banrang yang kuantitasnya kurang (PT) ...
baik. Agar berita tersebut menjadi lengkap,
(D) Kita dapat mengjangkau desa kalimat lanjutan yang cocok adalah
terpencil itu dengan menggunakan (A) Demonstrasi dilakukan agar Pemkot
sarana komunikasi sepeda motor. dan masyarakat dapat menciptakan
(E) Orang itu dapat memanfaatkan suasana kondusif untuk beribadah
waktu secara selektif singga sukses selama bulan Puasa.
dalam usahanya. (B) Para demostran terdiri atas para
aktivis keagamaan di kampus-
kampus sekitar Surabaya.

18 Sony Sugema College


Sony Sugema College
(C) Selain menggelar poster dan 22. Mengatur waktu secara efektif tidak
spanduk, puluhan mahasiswa ini masalah. Kita perlu tekun melatih dan
secara bergantian menggelar orasi. mencoba memanfaatkannya dengan
(D) Setelah dari Plaza Tunjungan massa baik. Agar tercapai kesinambungan
bergerak ke arah gedung DPR. persiapan upaya tersebut diperlukan
(E) Orasi oleh para mahasiswa tersebut berbagai keterampilan manajemen.
dilakukan secara berapi-api. Dalam manajemen waktu yang
diperlukanadalag matematika prioritas
21. Susunlah kelima kalimat di bawah ini kegiatan. Setelah itu yang sangat
sehingga menjadi sebuah paragraf yang penting adalag sejalan dalam
baik mengerjakan kegiatan sesuai dengan
1. Di hulu hutan di sekitar Sungai jadual yang telah direncanakan.
Mahakam terus dibabat. Gagasan utama paragraph yang tepat
2. Sayangnya, harmoni itu terancam adalahkalimat di atas adalah
rusak akibat eksploitasi hutan di (A) Melatih diri
sekitar Mahakam. (B) Prioritas kegiatan
3. Ribuan kubik kayu mengapung di air (C) Disiplin kerja
sungai, atau ditumpuk di bantaran (D) Mengatur waktu dengan tepat
sungai. (E) Memanfaatkan waktu
4. Air, sungai, bumi menciptakan
23. (1) Upaya penipuan melalui SMS terus
sebuah harmoni keindahan di
berlangsung. (2 )Modalnya, sebagian
Sungai Mahakam.
besar menginformasikan bahwa si
5. Selain itu, limbah puluhan pabrik
penerima pesan menjadi pemenang
berpotensi mencemari sungai
undian berhadiah. (3) Calon korban yang
Urutan yang benar adalah
nomor telepon gengamnya dipilih secara
(A) 1, 3, 4, 2, 5
acak diminta menghubungi sebuah
(B) 1, 4, 2, 3, 5
nomor telepon genggam lain. (4) Tahap
(C) 4, 2, 1, 3, 5
selanjutnya, korban diminta datang ke
(D) 4, 2, 3, 1, 5
lokasi Anjungan Tunak Mandiri. (5) ATM
(E) 4, 2, 3, 5, 1
kini tersebar di hampir seluruh pelosok
kota. (6) Saat itulah pelaku mendikte
perintah untuk mengirim uang ATM-nya
ke rekening pelaku. (7) Selama
berkomunikasi dengan korban, pelaku
mengaku sebagai pejabat dari
perusahaan yang mengadakan undian
Salah satu kalimat yang mengganggu
kepaduan teks di atas seningga harus
dihilangkan adalah
(A) kalimat 3
(B) kalimat 4
(C) kalimat 5
(D) kalimat 6
(E) kalimat 7

Sony Sugema College 19


24. Karena melanggar peraturan, siswa itu
terkena sanksi dari sekolah. Siswa itu
masih sangsi akan sanksi yang
ditimpakan kepadanya.
Penggunaan kata sangsi dan sanksi
dalam kedua kalimat di atas
menunjukkan adanya hubungan makna

(A) polisemi
(B) sinonim
(C) homonim
(D) homofon
(E) homograf

25. Kalimat berobjek di bawah ini adalah


(A) Pak Al Amin, ketua RT kami,
berdagang buah-buahan di Pasar
Baru.
(B) Ayahnya menjadi guru di SMAN
Palembang.
(C) Mereka berjalan-jalan berkeliling
kota
(D) Ibu memberi uang kepada pengemis
itu.
(E) Ledakan yang sangat mengejutkan
itu terdengar sampai radius satu
kilometer.

20 Sony Sugema College


Sony Sugema College
4. Tugas Mandiri Bahasa Indonesia

Pada tahun 1984 Indonesia telah dapat mempengaruhi pemilihan pangan


berhasil mencapai swasembada beras. Hal ini yang dikonsumsi, termasuk pangan pokok,
berarti harga beras relatif terjangkau oleh sehingga terbentuk suatu pola pangan
masyarakat. Meningkatnya ketersediaan pokok.
beras, pendapatan penduduk, dan sistem
nilai sosial pada beras telah mengarahkan 1. Judul yang paling tepat untuk teks di atas
pola konsumsi pangan pada beras. Karena adalah
itu, diperlukan upaya-upaya (A) Keberhasilan Indonesia dalam
penganekaragaman pangan pokok untuk Mencapai Swasembada Beras
mengurangi ketergantungan pada beras, (B) Penaggulangan Ketergantungan
sekaligus mempertahankan swasembada pada Konsumsi Beras
beras, serta meningkatkan mutu gizi pangan. (C) Masalah Pangan bagi Indonesia
Upaya penganekaragaman pangan Belum Terpecahkan
telah dimulai sejak Pelita II, yang disertai (D) Dampak Keberhasilan Swasebada
dengan dikeluarkannya Inpres No. 14/1974 Beras pada Masyarakat
dan disempurnakan menjadi Inpres No. (E) Peningkatan Mutu Gizi Melalui
20/1979 tentang “Perbaikan Menu Makanan Penganekaragaman Pangan Pokok
Rakyat”. Upaya mengurangi ketergantungan
pada satu jenis pangan pokok dilakukan, 2. Berikut ini merupakan tujuan
selain untuk melestarikan swasembada pemerintah memperbaiki menu
pangan, khususnya beras, juga untuk makanan rakyat Indonesia KECUALI
membuat keberagaman pangan khas daerah, (A) melestarikan swasembada beras
sehingga setiap daerah memiliki pola (B) memberagamkan pola konsumsi
konsumsi khas, yakni suatu susunan beragam khas daerah
makanan yang biasa dikonsumsi seseorang (C) mengurangi ketergantungan pada
atau kelompok orang. beras
Pangan pokok dikenal sebagai satu (D) meningkatkan pendapatan
atau lebih jenis bahan pangan yang dimakan penduduk dari pangan
secara teratur oleh suatu kelompok (E) meningkatkan kesehatan gizi
penduduk dalam jumlah cukup besar untuk keluarga
menyediakan bagian terbesar dari konsumsi
energi total yang dihasilkan oleh makanan.
Sekelompok penduduk suatu daerah tertentu,
terutama di pedesaan, akan bertumpu pada
pangan pokok penghasil energi dalam hal
pola konsumsinya. Faktor produksi, distribusi,
ketersediaan, pengetahuan gizi, daya beli,
sosial psikologis, daya terima, serta selera
Sony Sugema College 21
3. Pertanyaan berikut yang sesuai dengan (C) Apakah yang dimaksud dengan pola
isi paragraf I teks di atas adalah konsumsi pangan pokok?
(A) Keberhasilan Indonesia dalam (D) Bagaimana cara menyediakan pola
mencapai swasembada beras dapat pangan suatu masyarakat?
meningkatkan mutu pangan (E) Mengapa penduduk suatu daerah di
penduduk. pedesaan bertumpu pada pola
(B) Peningkatan kecenderungan pangan pokok?
penduduk beralih ke pola konsumsi
6. Pemakian tanda baca yang benar
pangan pokok beras perlu diimbangi
terdapat pada kalimat
dengan penganekaragaman pangan
(A) Teknik bernyanyi Titi DJ, diva musik
pokok.
Indonesia, selama konser tunggalnya
(C) Karena swasembada beras dapat
sangat memukau penonton.
meningkatkan mutu gizi keluarga,
(B) Data mahasiswa baru yang diterima
pemerintah perlu mengusahakan
di perguruan tinggi tahun 2005 s/d
pola konsumsi pangan pokok yang
2007 adalah…
lain.
(C) Merry, Wulan, dan Intan bersahabat
(D) Penganekaragaman pola konsumsi
sejak mereka masih duduk dibangku
pangan pokok penduduk merupakan
SMP.
pekerjaan besar yang mendukung
(D) Jangan khawatir, tidak semua siswa
program pemerintah.
berminat untuk masuk ke ITB.
(E) Keberhasilan swasembada beras
(E) Sahabat saya, yang tinggal di Plaju,
ditunjang oleh pemerintah dengan
sekarang sudah pindah ke Bandung.
upaya perbaikan menu makanan
rakyat. 7. Kalimat berikut yang seluruhnya ditulis
dengan menggunakan ejaan yang benar
4. Pola konsumsi pangan pokok suatu adalah
keluarga dipengaruhi oleh beberapa hal (A) Sastrawan selaku subyek sangat
berikut KECUALI penting peranannya di dalam
(A) pemahaman tentang gizi makanan perkembangan kesusastraan.
(B) kondisi sosial dan pola pikir anggota (B) Pernyataan diatas menunjukkan
keluarga bahwa Kesusastraan Indonesia
(C) tingkat pendidikan anggota keluarga memiliki perkembamngan yang
(D) selera anggota keluarga terhadap sangat menggembirakan.
makanan (C) Perbedaan pendapat Sastrawan
(E) kondisi keuangan keluarga akan jelas terlihat keberadaannya
dari segi apapun dalam
5. Pertanyaan berikut yang sesuai dengan kesusastraan.
gagasan pokok paragraf III adalah (D) Karena itu, setiap pelajar yang
(A) Bagaimanakah upaaya pemerintah belajar menganalisis karya sastra
dalam penganekaragaman pola harus selalu memperhatikan latar
pangan pokok? belakang budaya sastrawannya.
(B) Faktor-faktor apakah yang (E) Perbedaan pendapat antar
mempengaruhi pola konsumsi sastrawan Indonesia tidak menutup
pangan pokok masyarakat? kemungkinan menjadi penyebab
terjadinya perselisihan.

22 Sony Sugema College


Sony Sugema College
8. Kata yang bercetak miring berikut yang 10. Bersama dengan ini kami mengundang
digunakan secara benar sesuai dengan seluruh pengurus OSIS untuk mengikuti
konteks kalimat sebelumnya adalah acara rapat pada…
(A) Jika tidak ada permintaan dari Kalimat pembuka surat undangan di atas
atasan untuk mengubah butir-butir sebaiknya diubah menjadi
anggaran, kita tidak perlu (A) Melalui surat ini kami mengundang
mengadakan perubahan butir-butir seluruh pengurus OSIS untuk
tersebut. mengikuti acara rapat pada…
(B) Meskipun berlatih di arena yang (B) Bersama ini kami mengundang
telah disiapkan sebelumnya, ia seluruh pengurus OSIS untuk
selalu tidak mendapatkan hasil dari mengikuti acara rapat pada…
kegiatan perlatihannya itu. (C) Dengan ini kami mengundang
(C) Keputusan itu dirumuskan oleh para seluruh pengurus OSIS untuk
ahli dari berbagai bidang ilmu mengikuti acara rapat pada…
sehingga perumusannya benar- (D) Berhubung ada hal yang harus
benar dapat dibicarakan, dengan ini kami
dipertanggungjawabkan. mengundang seluruh pengurus OSIS
(D) Pemandu diskusi telah untuk mengikuti acara rapat pada…
menyimpulkan hasil pembahasan. (E) Dengan ini kami bermaksud
Kesimpulan tersebut disusun mengundang seluruh pengurus OSIS
berdasarkan hasil presentasi dan untuk mengikuti acara rapat pada…
tanggapan peserta.
(E) Perserta lomba lukis harap 11. Sejak lahirnya konsep pemikiran baru
menyediakan sendiri alat-alat dalam ilmu kedokteran, yang dicetuskan
lukisnya, karena persediaan panitia oleh Profesor Linus Pauling, yakni
sangat terbatas. tentang orthomoleculernya sebagai sains
dasar, penelitian medis diarahkan pada
9. Gadis cantik itu … masa kecilnya di Kota molekul-molekul yang secara normal
Bukit Tinggi. biologis-fisiologis ada dalam tubuh
Kata yang tepat untuk mengisi bagian manusia.
yang kosong adalah Inti kalimat panjang tersebut adalah
(A) bercerita (A) konsep pemikiran baru dicetuskan
(B) menceritakan tentang oleh Profesor Linus Pauling
(C) bercerita tentang (B) orthomolecular medicine adalah
(D) bercerita perihal sains dasar
(E) menceritakan mengenai (C) orthomolecular medicine dasarnya
adalah studi biologi
(D) penelitian medis diarahkan pada
molekul
(E) biologi milekuler merupakan sains
dasar

Sony Sugema College 23


12. Kalimat yang paling tepat susunannya (D) Rumah hunian yang kuat dan kokoh
adalah ditunjang oleh kontruksi beton
(A) Jika Anda memerlukan bantuan bertulang.
dapat menghubungi kami di kantor. (E) Rancangan tersebut dapat
(B) Bagi yang berminat mengikuti direkayasa dengan alat secanggih
perlatihan musik dapat komputer dan sistim digital.
mendaftarkan diri di sekretariat. 15. Kalimat-kalimat berikut menggunakan
(C) Dari hasil rapat kemarin pilihankata yang tepat
mengisyaratkan bahwa proposal (A) Meskipun hanya bermodal
yang diajukan disetujui. paspasan, tetapi Aminah dapat juga
(D) Jika hal itu diperhatikan oleh para menyelesaikan kuliahnya.
pengguna jalan, tidak akan (B) Karena tidak diizinkan meninggalkan
menimbulkan kemacetan. kelas, maka Andre terpaksa tidak
(E) Dalam buku petunjuk teknis, telah bisa mengikuti rapat.
dijelaskan langkah-langkah kerja (C) Berdasarkan hasil analisa data, maka
secara operasional. dapat disimpulkan bahwa
13. Makna imbuhan pe-an dan k e-an yang kesantunan berperan penting dalam
menyatakan tempat terdapat dalam komunikasi.
kalimat... (D) Meskipun uang menjadi modal
(A) Pemakaman jenazah itu akan utama suatu usaha, namun banyak
dilakukan besok dan diberangkatkan juga usaha yang dapat dilakukan
dari kelurahan. tanpa uang.
(B) Pemandangan di pantai itu (E) Agar memperoleh pendapatan yang
menimbulkan keindahan yang lebih banyak lagi, tidak jarang orang
hakiki. yang kreatif memanfaatkan waktu
(C) Pemandian umum di seluruh luangnya untuk bekerja sambilan.
kecamatan Cililin akan diperbaiki.
(D) Penculikan para aktivis itu disertai 16. Para penumpang pesawat itu belum
dengan kekerasan. tahu … penerbangan ditunda karena
(E) Kita harus terus menggalang cuaca buruk.
persatuan dan kesatuan. Konjungsi yang tepat untuk
menggabungkan kedua klausa itu adalah
14. Pola kalimat di bawah ini sama, KECUALI

(A) pelaku penimbunan BBM yang
(A) jika (D) sebab
menari di atas penderitaan rakyat
(B) kalau (E) bahwa
banyak harus dihukum berat.
(C) jangan-jangan
(B) Melalui proses sedimentasi disiplin,
sedikit demi sedikit nilai-nilai
kedisiplinan diendapkan ke dalam
masyarakat.
(C) Sebenarnya soal BBM di negara kita
tidak menjadi masalah karena
negara kita kaya akan kandungan
minyak.

24 Sony Sugema College


Sony Sugema College
17. Semua bentuk kehidupan di muka bumi (D) Yth. Direktur Personalia
menyandang nilai intrinsik, baik PT Intan Jaya Abadi
kehidupan manusiawi maupun non- Jalan Bakaran 352 B
manusiawi. Nilai kehidupan non- Palembang
manusiawi tidak bergantung pada (E) Kepada Yth. Direktur Personalia
kegunaan bagi manusia. Kekayaan dan PT Intan Jaya Abadi
keberagaman hayati bernilai pada Jalan Bakaran 352 B
dirinya sendiri dan pada gilirannya Palembang
memberikan sumbangan berarti bagi
19. Tak selamanya tanah longsor harus
kehidupan di muka bumi. Manusia tidak
dimaknai sebagai nasib. Sebagai
berhak mereduksi kekayaan berbagai
geologis, sebelum terjadi longsor akan
bentuk kehidupan kecuali untuk
muncul retakan tanah memanjang
memenuhi kebutuhan yang sangat vital.
sejajar dengan sisi lereng seperti yang
Simpulan teks di atas adalah
biasa terlihat di jalan raya di tepian
(A) kekayaan bumi dirusak oleh manusia
jurang. Ada pencegahan yang dapat
(B) makhluk di muka bumi ini
dilakukan untuk menghindari, mulai dari
mempunyai kegunaan masing-
yang sederhana seperti menghentikan
masing
eksploitasi lahan hingga menambal
(C) manusia tidak berhak bertindak
retakan atau membuat saluran
sewenang-wenang terhadap
pembuangan air di kawasan lereng.
kehidupan non-manusia
Saluran pembuangan air sederhana
(D) kekayaan alan tidak dibutuhkan oleh
dapat dibuat dari potongan-potongan
makhluk non-manusia
bamboo yang dilubangi kedua ujungnya
(E) kehidupan manusia tidak
lalu ditancapkan di lereng. Tujuannya
bergantung pada kehidupan non-
adalah mengeluarkan air yang terserap
manusia
dapat mengalir keluar sehingga tidak
ada kejenuhan massa tanah.
18. Penulisan alamat surat yang benar
Dalam paragraf tersebut kalimat yang
adalah
tidak berhubungan dengan isi
(A) Yth. Direktur Personalia
keseluruhan paragraph adalah kalimat
PT Intan Jaya Abadi
(A) pertama (D) keempat
Jln. Bakaran 352 B
(B) kedua (E) kelima
Palembang
(C) ketiga
(B) Kepada Yth. Direktur Personalia
PT Intan Jaya Abadi
Jln. Bakaran 352 B Palembang
(C) Yth. Direktur Personalia
PT Intan Jaya Abadi
Jln. Bakaran 352 B, Palembang

Sony Sugema College 25


20. Virus flu burung kemungkinan besar 21. Dalam kehidupan bermasyarakat, apa
terbang ke seluruh pelosok dunia yang dibutuhkan oleh seseorang belum
menumpang pesawat terbang, bukan tentu sama dengan apa yang dibutuhkan
bersama burung migran. Demikian orang lain. Di samping itu, suatu
kesimpulan sementara para pakar kebutuhan yang dapat dicapai oleh
ekologi burung liar. Merka tidak bisa seseorang, belum tentu dapat dicapai
menghubungkan penyebarab virus orang lain. Kenyataan seperti itu dari
mematikan itudengan pola migrasi. waktu ke waktu akan selalu ada
Meskipun demikian, para pejabat Kalimat berikut yang cocok untuk
kesehatan masih meresahkan melanjutkan paragraf di atas adalah
kemungkinan burung migran (A) Oleh karena itu, kita harus siap
menyebarkan virus ke seluruh dunia. menghadapi kebutuhan-kebutuhan
Memang banyak bukti yang menunjukan tersebut.
bahwa burung migrant liar memainkan (B) Sebaiknya siapa pun harus selalu
peran dalam penularan virus, namun mewaspadai kemungkinan
mereka bukanlah actor utamanya. Bila terjadinya konflik tersebut.
hanya berpegang pada bukti bahwa (C) Oleh karena itu, kemungkinan
burung liar adalah penular utama terjadinya konflik akibat perbedaan
penyakit itu. Wabah yang terjadi di itu pun akan selalu ada.
seluruh dunia seharusnya lebih dramatis. (D) Selanjutnya, kemungkinan terjadi
Pokok pikiran oaragraf tersebut adalah…. konflik akibat perbedaan itu pun
(A) Virus flu burung terbang akan selalu ada.
menumpang pesawat terbang bukan (E) Sehingga dengan demikian kita
bersama burung migran. harus dapat memenuhi kebutuhan-
(B) Para pakar ekologi tidak bisa kebutuhan tersebut.
menghubungkan penyebaran virus
mematikan itu dengan pola migrasi.
(C) Para pejabat kesehatan masih
meresahkan kemungkinan burung
migrant menyebarkan virus ke
seluruh dunia.
(D) Burung migran liar memainkan
peran dalam penularan virus, namun
mereka bukanlah actor utamanya.
(E) Wabah yang terjadi lebih dramatis,

26 Sony Sugema College


Sony Sugema College
22. Seiring dengan lajunya pertumbuhan 24. Susunlah keenam kalimat berikut
dan perkembangan kota, dituntut menjadi sebuah paragraf yang utuh dan
adanya sarana transportasi yang padu!.
mamadai. Di kota-kota besar di 1. Padahal, potensi produksi kakao
Indonesia, sarana transportasi kota tidak adalah 400.000 ton.
cukup mengandalkan bus kota, angkutan 2. Selain penurunan volume produksi,
kota, dan kereta api, tetapi juaga jenis kualitas buah yang dipanen juga
jasa angkutan bergengsi, yaitu taksi. merosot tajam.
Kalimat iklan yang sesuai dengan ilustrasi 3. Serangan hama penggerek buah
di atas adalah kakao semakin meluas.
(A) Sukses melaju bersama kepak sayap 4. Keadaan ini mengakibatkan
bus antarkota Garuda. produksi kakao nasional setiap
(B) PO Nusantara melayani transportasi tahunnya akan menyusut minimal
dalam kota. 20 persen.
(C) Wujudkan mimpi Anda dengan naik 5. Hampir seluruh pelosok Indonesia
kereta api ekspres. mengalaminya.
(D) Keselamatan barang Anda terjamin 6. Dapat dipastikan, sulit bagi petani
bila naik bus Mandala. kakao untuk bersaing di pasar
(E) Dengan taksi eksekutif, keamanan internasional.
dan kenyamanan Anda terjamin. Urutan yang benar adalah
(A) 3, 5, 4, 1, 2, 6
23. Tatkala bulan meredupkan cahayanya, (B) 3, 5, 4, 1, 6, 2
burung-burung malam beterbangan (C) 3, 5, 4, 2, 1, 6
menuju sarangnya. (D) 3, 5, 6, 4, 1, 2
Klausa yang berpola S-P-O dalam kalimat (E) 3, 5, 6, 1, 4, 2
di atas adalah …
(A) burung-burung malam bernyanyi
(B) menuju sarangnya
(C) bulan bercahaya
(D) bulan meredupkan cahayanya
(E) tatkala bulan merdup

Sony Sugema College 27


25. Tengoklah pada luka tubuh suku demi
suku
Tengoklah empat lima lobang mata
bayonet
Dua tiga mata peluru di tubuh-tubuh
bergeletakan
Sejarah terus mencatat menghitung
jumlahnya
Balantentara telah menjadi pilar
kegaulan tanah air
Republik tinggal papan nama berlumut
Yang melapuk menyertai kepercayaan
(”Aku Tak Mendustaimu, Indonesia”,
oleh J J Kusni dalam Media Kerja Budaya,
Agustus 2002)
Penggunaan majas personifikasi pada
kutipan puisi di atas terdapat pada baris
ke-...
(A) satu
(B) tiga
(C) empat
(D) lima
(E) enam

28 Sony Sugema College


Sony Sugema College
5. Tugas Mandiri Bahasa Indonesia

Keragaman atau kebhinikaan atau (D) tenggang rasa dalam masyarakat


multikulturalisme merupakan salah satu dan berbangsa
realitas utama yang dialami masyarakat dan (E) strategi untuk memperkenalkan
kebudayaan di masa silam, kini dan di waktu- budaya bangsa
waktu mendatang. Multilulturalisme secara
sederhana dapat dipahami sebagai 2. Bhinneka Tunggal Ika sebagai salah satu
pengakuan, bahkan sebuah Negara atau cirri masyarakat Indonesia dapat dilihat
masyarakat tidak mengandunghanya dari…
kebudayaan nasional tunggal. Sebalikanya, (A) Adanya fondasi yang kuat sebagai
Negara tidak mengandung hanya dasar kehidupan masyarakat.
kebudayaan nasional tunggal. (B) Kepedulian terhadap pendidikan
Keragaman itu hendaklah tidak masyarakat Indonesia.
diinterprestasikan secara tunggal. Komitmen (C) Adanya rasa saling menghormati
untuk mengakui keragaman sebagai salah dalam berbagai hak dan kewajiban.
satu ciri dan karakter utama masyarakat dan (D) Perhatian terhadap nilai-nilai moral
negara-negara tidaklah berarti relativisme bangsa.
cultural, disrupsi sosial atau konflik (E) Komitmen untuk mensejahtrakan
berkepanjangan. Sebab pada saat yang sama kehidupan masyarakat Indonesia.
terdapat simbol-simbol, nilai-nilai, struktur-
struktur dan lembaga-lembaga dalam 3. Arti kata “eksklusivisme” dalam bacaan
kehidupan bersama. Kesemuanya ini memiliki di atas adalah
fokus terhadap kolaborasi, kerja sama, (A) aliran yang mengutamakan
mediasi,dan negoisasi perbedaan-perbedaan kepentingan masyarakat banyak
dan, menyelesaikan konflik. Dengan (B) paham yang cenderung untuk
demikian, mereka menekankan kehidupan memisahkan diri dari masyarakat
bersama, saling mendukung, dan (C) konsep dasar kehidupan dalam
menghormati satu sama lain dalam berbagai masyarakat dan berbangsa
hak serta kewajiban personal maupun (D) paham yang mengutamakan
komunal. tanggung jawab komunal daripada
personal
1. Pokok pokiran penulis yang dapat (E) konsep kehidupan bersama yang
disimpulkan dari bacaan di atas saling mendukung dan menghormati
(A) peninjauan kembali
multikulturalisme 4. Pada bacaan di atas, kata “kolaborasi”
(B) multikulturalisme sebagai landasan berarti…
budaya (A) komitmen (D) perjanjian
(C) keragaman merupakan realitas di (B) mediasi (E) dukungan
NKRI (C) kerjasama
Sony Sugema College 29
5. Proses pembentukan frase tidak semua 7. Kata bercetak miring dalam kalimat
penyakit menular yang mengerikan berikut ini termasuk kata baku, KECUALI
dalam kalimat tidak semua penyakit (A) Gaun pengantin yang dikenakannya
menular yang mengerikan membawa pada malam itu terbuat dari sutera
kematian kematian bagi penderitanya. (B) Dari tahun ke tahun karirnya
Adalah… meningkat dengan pesat.
(A) penyakit > penyakit menular > tidak (C) Ia tidak pernah frustrasi meskipun
semua penyakit menular > tidak sering mengalami kegagalan dalam
semua penyakit menular yang kehidupan.
mengerikan (D) Ia mengirim foto terakhirnya ke
(B) tidak > semua penyakit > penyakit redaksi majalah remaja.
menular yang mengerikan > tidak (E) Manajemen perusahaan terkemuka
semua penyakit menular yang di Jakarta pada umumnya sangat
mengerikan baik.
(C) penyakit > penyakit menular >
penyakit menular yang 8. Keadaan memang sudah mulai aman …,
mengerikan> tidak semua penyakit Kita harus tetap waspada.
menular yang mengerikan Konjungsi antarkalimat yang tepat untuk
(D) penyakit > penyakit menular > tidak mengisi bagian yang rumpang dalam
semua penyakit menular > tidak kalimat itu adalah…
semua penyakit menular yang (A) akan tetapi (D) oleh karena itu
mengerikan (B) dengan demikian (E) karena itu
(E) tidak semua > penyakit menular > (C) jadi
penyakit menular > tidak semua
penyakit menular yang mengerikan 9. Seperti tidak mau kalah dengan IMF,
Bang Dunia dalam beberapa bulan
6. Sama halnya dengan kota-kota besar terakhir ini telah mem-blow up adanya
lainnya di Indonesia, kota Bandung juga praktik korupsi di Indonesia atas dana
tidak lepas dari sejumlah pesoalan, di bantuannya di dua projek, yakni Projek
antaranya kemacetan, pemukiman Transportasi Daerah untuk Indonesia
kumuh, sampah, pencemaran, dan Timur dan ProjekI Infrastruktur Jalan
dagangan asongan. Problem lingkungan Strategis.
hidup di Kota Bandung, jauh lebih parah Inti kalimat di atas adalah…
dan komplex disbanding keadaan (A) IMF mem-blow up adanya korupsi di
Bandung tahun 1980-an. Indonesia.
Dalam kutipan di atas terdapat bentukan (B) Bank Dunia dan IMF mem-blow up
kata yang salah, yakni… korupsi di Indonesia.
(A) persoalan – kemacetan – (C) Bank Dunia mem-blow up adanya
pencemaran praktik korupsi.
(B) pemukiman – dagangan – komplex (D) Bank Dunia mem-blow up adanya
(C) sejumlah – dagangan – di banding dana bantuan IMF.
(D) pencemaran – permukiman – (E) Bank Dunia memberi bantuan di dua
komplex projek, yaitu transfortasi daerah dan
(E) pemukiman – diantaranya – projek infrastruktur jalan strategis.
keadaan

30 Sony Sugema College


Sony Sugema College
10. Dilarang masuk kecuali mahasiswa. (E) Meja hijau di sudut ruang itu sudah
Kalimat ini tidak efektif karena tidak terpakai lagi.
(A) terlalu pendek
(B) tidak perlu memakai kata kecuali 14. Dengan memfokuskan pada tubuh,
(C) tidak bersubjek fotografi mode menjadi bertentangan
(D) harus ditambah kata tidak di depan dengan mode itu sendiri karena mode
kata dilarang justru tentang pakaian yang membuat
(E) kata dilarang seharusnya melarang tubuh menjadi lebih menarik atau
berubah tampilannya.
11. Kata yang berpolisemi terdapat pada Gagasan utama kalimat di atas adalah…
kalimat (A) Fotografi mode memfokuskan pada
(A) kepala sekolah kami memberi tubuh.
contoh menulis alamat di kepala (B) Fotografi mode bertentangan
surat dengan mode.
(B) bang Sahala selalu menabung di (C) Mode membuat tubuh menjadi
Bank Mandiri lebih menarik.
(C) kalau saya beruang saya akan (D) Pakaian membuat tubuh menjadi
membeli boneka beruang sebab lebih menarik.
kamar saya beruang luas (E) Pakaian mengubah penampilan
(D) kaca jendela itu buram dan seseorang
dibeersihkannya dengan kertas
buram 15. Penulisan huruf kapital yang tepat
(E) sepasang merpati terjerat dikala air terdapat pada kalimat…
pasang (A) Pada hari Idul Fitri tahun ini, Ananda
ingin sekali berkumpul dengan ayah
12. Kata-kata yang memiliki hubungan dan Bunda, tetapi keinginan
vertical/ hiponim dengan kata lili adalah tersebut tidak dapat terkabulkan.
(A) melati (D) wangi Oleh karena itu, Ananda mohon
(B) bunga (E) nama orang maaf.
(C) wanita (B) Dari Penelitian LPEM, impor
kendaraan bekas dapat
13. Penggunaan ungkapan yang tepat mengakibatkan Devisa yang hilang
terdapat pada… mencapai 180 s.d. 450 juta dollar AS.
(A) Semua anak-anak menutup mata (C) Kami menyelusuri jalan-jalan desa
ketika mereka melihat pertunjukan naik turun Pegunungan di Daerah
yang mengerikan itu. Kabupaten Gunung Kidul.
(B) Karena banyak tamu, adik masuk (D) Kami juga sempat menelusuri rute
lewat pintu belakang. yang pernah dilalui Jenderal
(C) Tiket Festival Paduan Suara di Aula Sudirman dalam bergerilya
Barat ITB banyak dijual lewat pintu melawan Belanda. Padahal, ketika
belakang. itu Pak Dirman dalam keadaan sakit.
(D) Menjelang hari Idul Adha ayah (E) Hukum yang berlaku di negara kita
membeli kambing hitam untuk tertera dalam Kitab Undang-Undang
dijadikan kurban. perdata.

Sony Sugema College 31


16. Pemakaian huruf miring yang tepat 19. Penulisan nama jabatan yang tepat
terdapat pada kalimat… terdapat pada kalimat…
(A) Artikel S.P. Purba yang berjudul (A) Ayah adalah seorang guru.
Letusan Papandayan dimuat dalam (B) Siapa Bupati yang baru dilantik itu.
koran terkenal. (C) Seminggu yang lalu Kolonel
(B) Roman terkenal Armyn Pane ialah M.Pringgayuda, S.H. meninggal
Belenggu dunia.
(C) Masih relevankah julukan kota (D) Brigjen Yesaya baru dilantik menjadi
kembang bagi Bandung. Mayor Jenderal.
(D) Kebanyakan penduduk Jawa Barat (E) Presiden baru saja melantik
ialah suku bangsa Sunda. beberapa Menteri.
(E) Sajak yang dimuat dalam antologi
berjudul Serpih-serpih Cinta. 20. Imbuhan –an yang tidak tepat
penggunaanya terdapat dalam kalimat
17. Penulisan kata yang sesuai dengan EYD berikut
dalam kalimat di bawah ini adalah… (A) Temuan tim arkeolog itu bermanfaat
(A) Setiap hari Idul Fitri keluarga besar sekali untuk ilmu pengetahuan.
kami selalu mengadakan acara (B) Kita akan mendapat arahan dari
halalbihalal. pembimbing kita.
(B) Jika ingin membaca, ayah harus (C) Saya menerima kiriman dri orang
mengenakan kaca mata tua.
(C) Adik saya selalu membawa sapu (D) Anak kecil dilarang bermain di
tangan. jalanan.
(D) Mereka memberikan bantuan secara (E) Setelah mendapat latihan yang
suka rela kepada korban bencana cukup selama seminggu, saya mulai
alam itu. percaya diri.
(E) Kami turut berduka cita atas
meninggalnya salah seorang teman 21. Keberangkatanya tahun ini ditunda
mereka. karena ada kerusuhan di daerah yang
ditujunya. Fungsi ke-an pada kata
18. Pemakaian tanta baca yang tepat kerusuhan dalam kalimat di atas sama
terdapat dalam kalimat dengan fungsi –nya pada kata dalam
(A) Berbagai persoalan yang harus kalimat
diselesaikan meliputi persoalan di (A) Kami memandangnya dari sudut
bidang politik, sosial dan ekonomi. pandang yang berbeda.
(B) Ia sudah melamar pekerjaan (B) Anaknya yang sudah lama
kemana-mana tetapi belum juga menghilang tidak dikenal oleh
berhasil mendapatkan pekerjaan. ibunya lagi.
(C) Anda harus belajar dengan giat, agar (C) Duduknya berpindah-pindah saja.
dapat diterima di perguruan tinggi (D) Penyanyi itu mengalunkan suaranya
yang diharapkan. dengan sangat merdunya.
(D) Kita memerlukan alat-alat kantor: (E) Rupanya hari ini hari akan turun
meja, kursi, lemari dan computer. hujan.
(E) Dengan demikian, mereka harus kita
perhatikan sedemikian rupa.

32 Sony Sugema College


Sony Sugema College
22. Capres itu tetap menyatakan 24. Pola frase konsep musik garda depan
kebanggaannya meskipun meraih 15 sama dengan
juta suara dalam pemilu itu (A) sosok musik Indonesia baru
Pola kalimat di atas sama dengan pola (B) tolak ukur musik Indonesia
kalimat di bawah ini, KECUALI (C) musik Indonesia saat ini
(A) Partisipasi masyarakat dalam (D) kehidupan seni tradisi lama
menanggulangi bahaya narkoba (E) jenis-jenis musik etnik lain
meningkat setelah mereka
menyadari risiko kecanduan. 25. Keindahan dan keunikan ibu Kota
(B) Mereka memasuki ruang kelas satu Vietnam ini diyakini mampu menyatukan
per satu dengan tertpb setiap hari. peradaban tua dengan cerita-cerita
(C) Ketua DPR menugasi anggota MPR masa lalunya yang penuh misteri hingga
untuk menelaah lagi perundang- membuat kita seolah terlibat dalam
undangan yang berlaku. tradisi serta legendanya.
(D) Pak Nababan mengontrak rumah Kata misteri, tradisi, dan legenda dalam
mewah setiap tiga tahun sekali. teks di atas mengandung makna…
(E) Tito membeli sepatu untuk Intan (A) hal-hal yang gaib, kebiasaan, cerita
adiknya tersayang. (B) yang tidak masuk akal, budaya,
dongeng
23. Banyak orang Jawa bermukim di daerah (C) rahasia, adapt istiadat, cerita rakyat
transmigrasi, khususnya di daerah (D) penuh teka-teki, kebiasaan,
Sumatera Selatan. Lahan pertanian di kepercayaan
wilayah … mereka sangat subur. (E) sulit dijelaskan, budaya, dongeng
Kata yang tepat untuk melengkapi rakyat
kalimat tersebut adalah…
(A) permukiman 26. Penulisan akronim yang benar adalah
(B) pemukiman (A) Pengendara kendaraan bermotor
(C) bermukimnya harus membawa SIM.
(D) mukiman (B) Simon melanjutkan kuliahnya di
(E) mukim fakultas teknik UNDIP Semarang.
(C) Abri tetap menjadi pilihan terbaik
bagi pemuda Indonesia.
(D) Setiap PUSKESMAS seharusnya
membuka perawatan inap
(E) Peningkatan SISKAMLING di desa-
desa berarti meningkatkan iklim
sejuk.

Sony Sugema College 33


6. Tugas Mandiri Bahasa Indonesia

Model bangunan SD 02 Marunda 2. Terumbu karang juga merupakan


tidak seperti kebanyakan seklah lain. habitat sejumlah biota. Ikan kerabu,
Bangunan berbentuk panggung dengan kakap merah, udang, dan ikan hias
lantai dan dinding terbuat dari papan. banyak berlindung di sana. Perusakan
Sekolah itu terletak di jalan Marunda Pulo RT terumbu karang oleh orang-orang yang
03/ 07, Kelurahan Marunda, Kecamatan tidak bertanggung jawab
Cilincing, Jakarta Utara. Gedung sekolah mengakibatkan biota-biota itu terancam
yang mulai berdiri tahun 1988 ini merupakan punah.
gedung sekolah panggung satu-satunya di Kalimat tanggapan yang sesuai dengan isi
wilayah Jakarta Utara, bahkan mungkin di paragraf tersebut untuk menyelamatkan
Jakarta. terumbu karang adalah…
(A) Sebaiknya yang merusak diberi
1. Bacaan di atas menginformasikan bahwa pekerjaan baru yang mereka
(A) sekolah SD satu-satunya di Jakarta inginkan.
Utara (B) Perusak terumbu karang harus
(B) gedung sekolah dasar yang didirikan diberantas dengan cara diberi sangsi
tahun 1988 yang berat
(C) di Marunda terdapat SD 02 pagi (C) .Boleh saja mereka mengambil
(D) SD yang terletak di jalan Marunda terumbu karang itu asal untuk
Pulo, RT 03/ 07 membantu perekonomian mereka.
(E) bangunan SD 02 pagi Marunda, (D) Kalau mereka merusak terubu
Cilincing, Jakarta Utara karang, mereka berkewajiban untuk
memulihkan kembali.
(E) Yang bertanggung jawab terhadap
perusakan itu adalah orang atau
nelayan yang tinggal disekitar laut.

34 Sony Sugema College


Sony Sugema College
3. Saudara-saudara ada ungkapan yang 4. Membudayakan kegemaran membaca
menhyatakan bahwa penulis tidak bukanlah hal yang mudah. Banyak
pernah dilahirkan, tetapi diciptakan. tantangnan yang melatari
Ungkapan tersebut mengandung makna pembudayaan kegemaran membaca.
bahwa penulis sebenarnya bukan Pertama kurangnya pemahaman
persoalan bakat seseorang sejak lahir, masyarakat sendiri terhadap pentingnya
melainkan soal minat dan motivasi, buku. Buku masih dianggap kebutuhan
kegigihan, serta berlatih dengan tekun. nomor sekian. Kenyataan ini terlihat
Kegagaglan berkali-kali dalam menulis ketika kebutuhan pokok sudah
bukanlah alas an yang tepat untuk terpenuhi, orang yang menyisihkan uang
mengubur keinginan menjadi penulis. untuk untuk membeli buku. Sulit sekali
Simpulan yang tepat dari isi pidato menjadikan sebuah buku sebagai
tersebut adalah … kebutuhan utama. Akan tetapi, untuk
(A) keberhasilan seseorang menjadi mendengarkan sebuah kaset atau
seorang penulis ditentukan oleh menonton film, banyak orang yang tak
motivasi sungkan mengeluarkan uang
(B) untuk menjadi penulis yang baik, Pernyataan yang merupakan alas an
seseorang tidak perlu memiliki bakat dalam paragraf tersebut adalah…
alam yang dibawanya sejak lahir. (A) kebudayaan gemar membaca
(C) semua orang akan dapat dengan dikalangan masyarakat tertentu
mudah menjadi penulis jika ia (B) banyak tantangan yang melatari
menginginkannya pembudayaan gemar membaca
(D) kegagalan yang terjadiberkali-kali (C) kenyataan ini terlibat ketika
merupakan motivasi menjadi kebutuhan pokok sudah terpenuhi
penulis (D) buku masih dianggap kebutuhan
(E) tidak ada seorang pun yang nomor sekian
dilahirkan untuk menjadi penulis (E) untuk mendengarkan sebuah kaset
atau menontion film, banyak orang
tak sungkan mengeluarkan uang

5. perusahaan otomootif kami


mengutamakan kepuasan pelanggan.
Kata kepuasan dalam kalimat di atas
termasuk kelas kata …
(A) numeralia (D) nomina
(B) verba (E) ajektiva
(C) adverbial

Sony Sugema College 35


6. Pada masa yang lalu, acara radio penuh 8. PU : semua pelajar yang ingin maju tidak
dengan acara pementasan drama dan mengonsumsi narkoba.
pergelaran musik yang disiarkan secara PK : Gilang seorang pelajar yang ingi
langsung. Akan tetapi sejak sejak TV maju.
makin dikenal. Acara ini terpaksa diubah. S:….
Orang lebih senang melihat kedua acara Kalimat simpulan yang tepat untuk
tersebut ditayangkan di layar TV melengkapi silogisme tersebut adalah…
daripada mendengarkannya di radio. (A) Gilang seorang pelajar yang ingin
Sekarang sebagian besar penduduk tetap sehat
Indonesia akrab dengan tayangan TV (B) Semua pelajar tidak
dan berusaha untuk memiliki pesaw menyalahgunakan narkoba.
Kalimat yang tepat untuk melengkapi (C) Gilang tidak mengonsumsi narkoba.
paragraf tersebut adalah… (D) Pelajar yang ingin tetap sehat tidak
(A) Tak aneh lagi media massa sangat mengonsumsi narkoba.
diminati sebagian besar penduduk (E) Gilang ingin tetap sehat, tidak
Indonesia. mengonsumsi narkoba.
(B) TV merupakan sarana komunikasi
yang sudah tidak asing lagi bagi 9. Raja itu memerintah dengan… Perilaku
rakyat Indonesia. kejam itu menurun kepada para … raja
(C) Hamper sebagian rakyat Indonesia tersebut. Setiap mereka menagih pajak,
telah memiliki pesawat TV. tidak pernah kembali dengan … karena
(D) Dalam segi pengadaan dana TVRI nmereka selalu menagih dengan
mempunyai hak untuk memungut m,emaksa.
iuran TV. Ungkapan yang tepat untuk melengkapi
(E) Sayang sekali acara yang paragraf tersebut adalah …
ditayangkan di TV lebih banyak (A) besar kepala, pembantu-
berupa iklan dari pada acara pembantunya, tangan kosong
pendidikan. (B) tangan besi, tangan kanan, tangan
hampa
7. Ibu guru selalu bilang, semua siswa tidak (C) manis mulut, bawahan mereka,
boleh mensontek pekerjaan teman. Jika tanngan kosong
ketahuan, ibu gura akan memberikan (D) keras kepala, kaki tangan, tangan
sangsi kepada siswa tersebut. besi
Perbaikan kata tidak baku yang tercetak (E) tangan besi, bawahan, keras kepala
tebal dalam paragraph tersebut adalah…
(A) mengomentari, mencontek, sanksi
(B) mengatakan, menyontek, sanksi
(C) memberitahukan, mencontek,
hukuman
(D) mengatakan, mencontek, sanksi
(E) memberitahukan, mencontek,
sanksi

36 Sony Sugema College


Sony Sugema College
10. .Judul karya tulis : Kiat menumbuhkan 13. Opini terdapat dalam kalimat
kesehatan dalam kehidupan sehari-hari (A) Mempertahankan prestasi di bidang
Penulisan judul karya tulis tsb yang tepat Fisika tingkat dunia lebih silit
adalah daripada meraih prestasi itu sendiri.
(A) Kiat Menumbuhkan kesehatan (B) Keyakinan Indonesia untuk
dalam kehidupan sehari-hari mengalahkan Cina di Olimpiade
(B) Kiat Menumbuhkan Kesehatan Fisika tahun depan akan terlaksana.
Dalam Kehidupan Sehari-hari (C) Prestasi yang tertinggi telah diraih
(C) Kiat menumbuhkan Kesehatan Fisika melalui proses penggodokan
Dalam kehidupan Sehari-hari. yang panjang dan serius.
(D) Kiat Menumbuhkan kesehatan (D) Indonesia merebut empat medali
Dalam Kehidupan sehari-hari emas dan dan satu medali perak di
(E) Kiat Menumbuhkan Kesehatan Salamanca Spanyol tahun lalu.
dalam Kehidupan Sehari-hari (E) Meraih gelar juara dunia di bidang
sains adalah prestasi yang sangat
11. Tanaman obat biasa dikenal dengan
hebat di mata dunia.
sebutan … hidup seperti kumis kucing,
temu lawak, dan banyak lagi tanaman
14. Kalimat berikut ini yang termasuk
yang bisa dijadikan obat. Terutama di
kalimat baku adalah
desa-desa, orang lebih percaya dan
(A) Pada setiap pernyataan yang
menganggap tanaman obat lebih … dan
diucapkannya selalu mengandung
… tinggi, disbanding dengan obat dokter.
kritikan yang tajam pada
Kata serapan yang tepat untuk
pemerintah.
melengkapi
(B) Dalam menghadapi konflik sosial
paragraph tsb adalah …
dan krisis ekonomi yang
(A) apotik, efektif, berkualitas
berkepanjanganini, tidak ada cara
(B) apotek, epektif, berkualitas
lain kecuali berdoa dan berusaha.
(C) apotek, efektif, berkualitas
(C) Jika persoalan ini dirundingkan dan
(D) apotik, efektif, berkwalitas
ditanggung bersama, maka akan
(E) apotik, efektiv, berkualitas
dapat segera diselesaikan dan terasa
12. Tema karya tulis : Persamaan hak antara ringan.
laki-laki dan perempuan (D) Ketika berada di pompa bensin
Latar belakang untuk tema tersebut untuk mengisi BBM, untuk tidak
adalah menghidupkan HP.
(A) kedudukan laki-laki di atas segalanya (E) Setiap orang memiliki kekuasaan
adalah benar bawaan entah kepribadian menarik
(B) kaum perempuan tugasnya hanya entah berkarisma tinggi, untuk
mengurus rumah tangga saja memberikan pengaruh.
(C) masih dijumpai perbedaan hak
antara laki-laki dan perempuan
(D) hak kaum laki-laki adalah hanya
untuk mencari nafkah
(E) hak mengurus rumah tangga adalah
suami dan istri itu sendiri

Sony Sugema College 37


15. Kalimat berikut yang memenuhi (B) Temperatur Bumi sama dengan
syaratkepaduan adalah Temperatur Mars.
(A) Apabila Jaksa Agung ingin (C) Bumi dan Mars sama-sama beredar
menggunakan hak hukum yang mengelilingi Bumi.
tersedia, tetap bisa dilakukan. (D) Di Mars mungkin ada makhluk hidup
(B) Kalau kita tidak memperhatikan seperti di Bumi.
secara cermat, akan sulit (E) Persamaan Mars dan Bumi.
menunjukan salahnya.
(C) Dalam menyusun skripsi, 18. Menurut penjelasan gubernur, para
mengumpulkan data merupakan pengusaha kecil dan menengah akan
aktivitas yang harus dilakukan. memperoleh kredit ringan untuk
(D) Untuk menmgembangkan naskah mengembangkan usahanya.
pidato yang baik, mempersyaratkan Kalimat intinya adalah
seseorang memiliki wawasan yang (A) Pengusaha menerima kredit.
luas. (B) Pengusaha akan menerima kredit.
(E) Karena alasannya tidak masuk akal, (C) Pengusaha mengembangkan usaha.
pihak jaksa sebagai penuntut umum (D) Pengusaha menerima penjelasan
tidak menerima. gubernur.
(E) Pengusaha kecil dan menengah
16. Jakarta yang merupakan kota memperoleh kredit ringan.
metropolitan dan ibu kota RI memiliki
potensi pariwisata budaya yang tidak 19. Kalimat berikut efektif, KECUALI
sedikit (A) Mereka sedang membicarakan
Kalimat di atas perluasan dari kalimat inti tentang masalah lingkungan.
(A) Jakarta adalah kota metropolitan. (B) Pak Edi sedang berbicara tentang
(B) Jakarta merupakan ibu kota RI masalah lingkungan.
(C) Jakarta adalah kota budaya (C) Dari penyelidikan itu terbukti bahwa
(D) Jakarta memiliki potensi pariwisata ia tidak bersalah.
(E) Jakarta memiliki potensi pariwisata (D) Dalam obat itu banyak mengandung
budaya yang tidak sedikit. zat yang banyak merusak sel tubuh.
(E) Di lingkungan belajar siswa dilarang
17. Kita tertarik pada planet Mars, karena tidak boleh merokok.
banyak persamaan dengan bumi kita.
Mars dan Bumi menjadi anggota tata 20. Sebanyak 60 personel militer Jerman
surya yang sama. Unsur oksigen dan air rata-rata memiliki keahlian di bidang
agaknya ada juga. Caranya beredar kontaminasi biologi, kimiawi dan nuklir
mengelilingi matahari menyebabkan menggunakan seragam pelindung dan
timbulnya musim seperti di bumi. Jika di kendaraan khusus untuk memasuki zona
Bumi ada makhluk hidup, tidakkah flu burung, Pulau Baltik di Reugen.
mungkin pula ada makhluk hidup di Untuk membentuk kalimat yang baku,
Planet Mars ? tanda baca koma (,) perlu dituliskan
Ide pokok paragraph di atas adalah setelah kata berikut, KECUALI
(A) Bumi dan Mars sama-sama anggota (A) Jerman (D) khusus
tata surya. (B) kimiawi (E) Baltik
(C) nuklir

38 Sony Sugema College


Sony Sugema College
21. Imbuhan pe-an berhubungan dengan 23. Organisasi Dunia mengingatkan bahwa
imbuhan me- seperti dalam pola pandemi flu burung akan terjadi jika
pembentukan kata dengar > mendengar virus flu burung bermutasi menjadi virus
> pendengaran. baru yang mematikan dan dapat
Pernyataan berikut ini yang benar adalah menular dari manusia ke manusia.
… Makna kata pandemi dalam kalimat ini
(A) Pola pembentukan kata pemerincian adalah
dalam kalimat pemerincian barang (A) penularan penyakit dari manusia ke
harus segera diselesaikan sama manusia.
dengan pendengaran. (B) penularan penyakit dari binatang ke
(B) Pola pembentukan kata binatang
pemelintiran dalam kalimat (C) wabah yang dapat menyerang
pemelintiran pernyataan pejabat manusia dan binatang
dilakukan wartawan.berbeda (D) wabah yang berjangkit serempak di
dengan pendengaran. mana-mana
(C) Pola pembentukan kata (E) wabah yang dapat mematikan
pemertahanan dalam kalimat dalam waktu yang singkat
pemertahanan bahasa ibu orang
Madura kuat sama dengan 24. Di antara kalimat-kalimat berikut ini
pendengaran. terdapat kalimat yang tidak memiliki
(D) Pola pembentukan kata kesejajaran bentuk, yakni
pemerolehan dalam kalimat (A) Tahap terakhir penyelesaian gedung
pemerolehan bahasa dimulai sejak itu adalah kegiatan pengecetan
bayi sama dengan pendengaran tembok, pemasangan penerangan,
(E) Pola pembentukan kata pengujian sistim pembagian air, dan
pemerkayaan dalam kalimat pengaturan tata ruang.
pemerkayaan diri hendaknya (B) Petani berusaha meningkatkan hasil
dilakukan secara wajar sama dengan panen dengan menggunakan bibit
pendengaran. unggul, pemupukan, pemberantasan
hama, dan penerapan teknologi
22. Kalimat berikut ini yang ditulis sesuai pascapanen yang tepat.
dengan aturan EYD adalah (C) Yeremia mahasiswa semester lima
(A) Saya tidak akan membeli mobil Fakultas Kedokteran UNPAD itu
mewah, karena tidak punya uang. sedang menolong Qurata Ayuni
(B) Oleh karena itu saya harus rajin dengan dipapahnya ke pinggir jalan.
menabung. (D) Simon memutuskan untuk
(C) Atas bantuan Saudara saya menyelidiki pembunuhan anjing
mengucapkan terima kasih. kesayangannya ketika ia
(D) Malam makin larut ; ayah belum menemukan anjing bernama Dinggo
juga pulang. mati di halaman rumah
(E) Semua siswa baik laki-laki maupun tetangganya.
perempuan, mengikuti ujian. (E) Tutup radiator unik bertermometer
ini akan memberi peringatan saat
suhu mesin dan radiator melebihi
batas yang ditoleransi.
Sony Sugema College 39
25. Susunan kata-kata atau frase di bawah
ini yang dapat diterima sebagai kalimat
adalah
(A) Sebab belum apa-apa telah muncul
semacam ketidakkonsistenan sikap
atau pernyataan dari berbagai
lembaga itu sendiri.
(B) Kepemimpinan bangsa yang sangat
diharapkan oleh masyarakat untuk
berani bersikap lebih sederhana dan
jelas.
(C) Masyarakat yang sadar akan
keadaan lingkungan yang bersih dan
aman
(D) Demikian juga dalam perkara Bank
Mandiri yang disinyalir ada
ketidakberesan pengembalian kredit
macet sebesar Rp 1 trilyun.
(E) Semuanya ingin terpuaskan dengan
skala masing-masing.

40 Sony Sugema College


Sony Sugema College
7. Tugas Mandiri Bahasa Indonesia

Pilihlah satu jawaban yang tepat ! menambah pasukan dalam pengamanan


Pakar hukum laut internasional Prof. Dr. Ambalat pasca pembangunan Mercusuar
Hasjim Djalal menyatakan, dalam berunding Karang Unarang. “Kan sudah disepakati
dengan Malaysia soal Blok Ambalat, diselesaikan secara diplomasi. Sudah lama
Pemerintah Indonesia perlu memiliki kita sepakat agar kehadiran angkatan
kesabaran dan tidak memilih opsi perang perang masing-masing tidak untuk
sebab perang akan mengancam hilangnya memperuncing ketegangan,” ujarnya, Kamis
konsep Wawasan Nusantara. (21/4).
Dalam Seminar “Masalah Ambalat” di Sumber : Kompas, 23 April 2005.
Center for Strategic and International
Studies, Kamis (21/4), Hasjim Djalal 1. Blok Ambalat yang menjadi wilayah
mengingatkan bahwa Konvensi Hukum Laut sengketa RI dengan Malaysia terletak di
1982 Pasal 83 Ayat (1) menegaskan (A) laut antara Sumatera dan Malaysia
Penentuan batas-batas landas kontinen dua (B) laut antara Kalimantan Timur dan
negara yang pesisirnya berhadapan atau Sulawesi
berdekatan harus dijalankan dengan (C) laut antara Jawa dan Sumatera
permufakatan lewat hukum international. (D) Laut Jawa
“Kalau mau langsung perang, berarti (E) laut antara Kalimantan dan
melanggar Pasal 83. Artinya, ada alasan Sumatera
orang lain untuk mengatakan kita tidak
menghormati Wawasan Nusantara. Padahal, 2. Wawasan Nusantara dalam tulisan di
ada persetujuan Indonesia dan Malaysia atas merujuk pada ...
1982 yang mengakui Wawasan Nusantara,” (A) kedaulatan Indonesia atas wilayah
katanya. Karena itu, negosiasi harus darat, laut, dan udara
dilakukan walaupun tidak sederhana dan (B) cara bangsa Indonesia
butuh waktu. Negosiasi antara India dan mempertahankan Blok Ambalat
China atau China dan Vietnam, misalnya, (C) konflik antara Indonesia dan
sudah berlangsung 50 tahun. Selain Malaysia
berunding, ada jalan keluar lain, yaitu (D) perbatasan wilayah laut Indonesia
menggunakan Pasal 33 Piagam PBB yang dengan negara-negara tetangga
menyatakan ada tiga mekanisme (E) cara memandang hubungan antar
penyelesaian sengketa damai, yaitu lewat negara
pihak ketiga, mekanisme regional, dan
mekanisme hukum. “Jadi pengadilan adalah
opsi terakhir,” katanya.
Secara terpisah Paglima Tentara
Nasional Indonesia Jenderal Endriartono
Sutarto mengatakan, belum berencana
Sony Sugema College 41
3. Yang dinyatakan Hasjim Djalal dalam 5. Kapan Krakatau meletus sedahsyat
paragraf pertama adalah tahun 1883? Menurut paraali
(A) Dalam berunding dengan Malaysia vulkanologi untuk membuat letusan
soal Blok Ambalat Indonesia sedahsyat itu dibutuhkan energi yang
menyatakan perang. besar. Untuk mengumpulkan energi
(B) Pemerintah Indonesia perlu memiliki baru, Krakatau membutuhkan waktu
kesabaran dan tidak memilih opsi berabad-abad. Akhir Maret 1980,
perang; Gunung Krakatau menunjukkan
(C) Perang akan mengancam hilangnya kegiatannya lagi. Kawahnya
konsep Wawasan Nusantara. mengeluarkan gumpalan asap yang
(D) Pemerintah Malaysia dan Indonesia membumbung setinggi 800 meter, serta
berunding soal Blok Ambalat. semburan api yang mengangkasa
(E) Pemerintah Indonesia perlu memiliki sampai 70 meter. Banyak wisatawan
opsi perang yang akan mengancam yang sengaja datang ke pantai-pantai
hilangnya konsep Wawasan Banten untuk melihat kegiatan gunung
Nusantara. yang menciptakan pemandangan yang
amat indah.
4. Abad ke-21 merupakan abad kemajuan Berdasarkan letak kalimat utamanya,
Asia Pasifik. Ibarat Departemen paragraf di atas tergolong paragraf
Pekerjaan Umum, Jepang kini berusaha (A) induktif
memetakan dan membuat rute (B) deduktif
perjalanan menuju liberalisme (C) induktif – deduktif
perdagangan dan investasi bebas dan (D) deduktif – induktif
terbuka pada abad ke-21. (E) sebab – akibat
Kalimat yang tepat untuk
menyempurnakan paragraf di atas 6. Kalimat-kalimat berikut ini tidak baku,
adalah KECUALI
(A) Jepang menjadi motor penggerak (A) Di era globalisasi seperti sekarang
dinamisme ekonomi negara-negara ini, yakni jarak, ruang, dan waktu
Asia Pasifik. menjadi begitu relatif karena dalam
(B) Jepang sedang menuju kemajuan hitungan detik pelbagai arus
abad ke-21. informasi dapat menyebar ke
(C) Jepang menyadarkan bangsa-bangsa seluruh dunia.
di kawasan Asia Pasifik. (B) Bagi calon kepala desa yang berusia
(D) Jepang merupakan negara paling di atas 54 tahun sampai dengan 60
maju di kawasan Asia Pasifik. tahun diharuskan mendapatkan
(E) Jepang mengantisipasi rekomendasi dari camat dan
perkembangan ekonomi global. dinyatakan sehat oleh dokter rumah
sakit daerah setempat.
(C) Memperingati Sumpah Pemuda,
Komite Nasional Pemuda Indonesia
(KNPI) selain mengandalkan
upacara, juga mengadakan
kampanye antinarkoba.

42 Sony Sugema College


Sony Sugema College
(D) Untuk mengembangkan semua desa 8. Dalam sidang tersebut, setiap kelompok
pada sekitar 460 kabupaten di diminta menyampaikan konsiderasi dari
Indonesia, dibutuhkan perencanaan kelompoknya dengan maksud agar
dan waktu yang panjang. dalam pembuatan putusan dapat
(E) Membicarakan tentang Mafia dihasilkan putusan yang adil
Camorra tidak dapat dilepaskan dari Pilihan kata yang tepat untuk
peran serta Raffaele Cutolo (65 menggantikan kata konsiderasi dalam
tahun) yang saat ini mendekam di kalimat tersebut adalah
penjara maksimum Asinara, (A) wawasan
Sardinia. (B) usulan
(C) pertimbangan
7. Penutup surat lamaran pekerjaan yang (D) pemikiran
paling tepat adalah (E) tanggapan
(A) Mudah-mudahan Bapak dapat
menerima saya. Atas perhatiannya 9. Dari kalimat-kalimat berikut yang
saya ucapkan terima kasih. termasuk kalimat tidak baku adalah
(B) Demikian surat lamaran kerja ini (A) Kekurangan tenaga profesional
saya ajukan. Atas perhatian dan untuk tingkat manajer
kebijaksanaan Bapak, saya kepariwisataan mungkin sekali akan
mengucapkan terima kasih. terjadi.
(C) Besar harapan saya Bapak sudi (B) Maka dari itu, karyawan yang cukup
menerima saya untuk mengisi senior di suatu hotel berbintang
lowongan pada perusahaan Bapak. sering frustasi akibat tuntutan untuk
Atas perhatian Bapak, saya ucapkan memenuhi target yang tinggi.
terima kasih. (C) Beberapa motif protes dari
(D) Besar harapan saya Bapak sudi beberapa karyawan tersebut adalah
menerima saya mengisi lowongan kurangnya kesempatan bagi mereka
pada perusahaan Bapak. Atas untuk menjadi puncak pimpinan.
perhatian Bapak, saya ucapkan (D) Karena hotel, restoran, dan biro
terima kasih banyak. perjalanan belum banyak menarik
(E) Kiranya Bapak sudi menerima saya minat calon-calon manajer dari
sebagai pegawai Bapak. Atas lembaga pendidikan manajemen.
perhatiannya saya ucapkan terima (E) Selama bertahun-tahun bekerja di
kasih sebelumnya. perusahaan itu, mereka selalu
menjadi orang kedua, ketiga, dan
seterusnya.

Sony Sugema College 43


10. Semula kewajiban memakai helm hanya Kalimat tersebut dapat menjadi kalimat
dikenakan pada anggota Polri yang baku jika diperbaiki dengan cara berikut
bersepeda motor. Ketentuan ini KECUALI
kemudian diterapkan secara lebih luas. (A) menambahkan kata dengan
Dengan Surat Keputusan tanggal 6 sebelum kata melihat.
Desember 1984, Kapolri mewajibkan (B) mengganti kata di mana dengan
setiap pemohon Surat Izin Mengemudi ketika.
(SIM) C menggunakan helm. Bahkan (C) menghilangkan kata maka.
Gubernur KDH Tingkat I mengintruksikan (D) menambahkan tanda koma (,)
agar semua pegawai negeri di sebelum kata sehingga.
daerahnya menggunakan helm pada sat (E) mengubah perkeluarga menjadi per
mengendarai sepeda motor. Instruksi ini keluarga.
sangat tepat karena pegawai negeri
sebagai abdi negara dan abdi 12. Penggunaan bentuk pasif yang tidak
masyarakat dituntut untuk memberikan tepat terdapat dalam kalimat
teladan dalam mewujudkan ketertiban (A) Ia akan kerjakan tugas itu setelah
lalu lintas, khususnya melalui pemakaian makan.
helm. (B) Pelarian itu belum diketahui
Simpulan paragraf di atas adalah persembunyiannya.
(A) Pemohon SIM diwajibkan memakai (C) Semua PR bahasa Indonesia sudah
helm menurut instruksi Kapolri. kukerjakan.
(B) Menurut instruksi Gubernur KDH (D) Jangan kaubiarkan mereka
Tingkat I semua harus memakai mencemarkan namamu.
helm. (E) Siapa yang bersalah akan diberi
(C) Pegawai negeri diharuskan memakai sanksi yang berat.
helm bila berkendaraan.
13. DISEWAKAN RUANG USAHA
(D) Masyarakat diwajibkan memakai
Disewakan segera 2 buah ruko
helm bila mengendarai sepeda
(kopel)terletak di Jl. Ps. Minggu (sebelah
motor.
Bank Danamon), lokasi sangat strategis,
(E) Polri yang bersepeda motor wajib 2
luas bangunan 362 M , 3 lantai, luas
memakai helm. 2
tanah 352 M . Fasilitas : listrik 53.000
VA, telepon3lines, key telepon 8 lines,
11. Melihat kondisi yang kita miliki saat ini,
tempat parkir luas (depan dan samping),
dimana persediaan beras yang dimiliki
AC central & split, cocok untuk
Bulog terbatas jumlah dan jenis
perkantoran, restauran, bank, atau
berasnya dan kemampuan untuk
tempat kursus, hubungi : Bapak Jefry,
membayar subsidi yang belum jelas,
telepon: 8717006-10 (5 saluran) Fax :
maka mungkin libih tepat apabila
8717011
alokasi beras miskin saja yang
Menurut isinya contoh iklan di atas
diperbesar sehingga mereka yang
termasuk iklan
tergolong keluarga miskin dapat
(A) penawaran
terlayani semuanya dengan 20 kg
(B) permintaan
perkeluarga.
(C) pemberitahuan
(D) pengumuman
(E) penyewaan
44 Sony Sugema College
Sony Sugema College
16. Premis mayor : Semua pegawai 18. Kalimat-kalimat berikut ini tidak dapat
pemerintah bila hari Jumat diwajibkan digunakan dalam karya ilmiah karena
melaksanakan Senam Kesegaran Jasmani penulisan tanda bacanya tidak mengikuti
di kantor masing-masing EYD, KECUALI
Premis minor : …….. (A) Acara Puncak Penghijauan dan
Kesimpulan : Kustiyani melaksanakan Konservasi Alam ke-40 di Desa
Senam Kesegaran Jasmani Jabiran merupakan kegiatan yang
setiap hari Jumat. harus dilakukan.
Isi Premis minor di atas adalah (B) Kendati pemakaian jamu cukup
(A) Senam Kesegaran Jasmani harus marak dan industri jamu juga
dilaksanakan Kustiyani. berkembang sungguh
(B) Kustiyani salah seorang pegawai mengherankan, kalau ternyata
yang melaksanakan Senam produksi tanaman obat beberapa
Kesegaran Jasmani tahun belakang stagnan.
(C) Kustiyani seorang pegawai di sebuah (C) Dalam catatan Kepala Badan POM
kantor pemerintah. Indonesia memiliki 30.000 jenis
(D) Kantor mewajibakn senam kepada tumbuhan.
karyawannya. (D) Penurunan angka produksi menjadi
(E) Pelaksanaan Senam Kesegaran ironi kalau kita bandingkan dengan
Jasmani pada hari Jumat. pasar yang sesungguhnya
membentang luas.
17. Di antara kalimat-kalimat berikut ini, (E) Tanpa pelaksanaan PPKAN,
yang termasuk kalimat baku adalah masyarakat, tidak dapat menyadari
(A) Data bagi studi prasejarah tidak pentingnya pengelolaan hutan
berupa data tertulis, karena itu dengan baik.
diperlukan perhatian yang khusus
untuk merekonstruksi kebudayaan 19. Penulisan kata asing yang sesuai dengan
manusia prasejarah. ejaan resmi terdapat dalam kalimat
(B) Sehingga upaya pelestarian Gua (A) Jika kebocoran akibat virus
Pawon sebagai data arkeologi diabaikan, penderita akan banyak
penting bagi Jawa Barat, khususnya kehilangan cairan, akhirnya akan
dan Indonesia umumnya. mengalami s h o c .k
(C) Dengan perkembangan teknologi (B) Genangan air merupakan tempat
yang begitu pesat, maka dapat berkembang biaknya nyamuk
dipastikan arus informasi akan “Aedes Aegepty”.
sangat cepat. (C) Asuransi ini memiliki program
(D) Namun seiring dengan professional mem-back up dokter
perkembangan zaman dan telah yang mendapat gugatan malpraktek
terjadi banyak perubahan. dari pasien.
(E) Pertunjukan rakyat yang lahir di (D) Lahar dingin itu “nggojog” dari atas
Jompang dan berkembang pesat di seperti ombak besar.
Surabaya itu merupakan satu paket (E) Ronaldo mencetak hattrick pada
kesenian berisi sandiwara, tari, menit ke-89 sehinggapetandingan
nyayian, musik, dan lawak. berakhir “draw”.

Sony Sugema College 45


20. Bentuk kalimat yang baku adalah 22. Di antara kalimat-kalimat berikut yang
(A) Budaya syawalan harus kita benar ejaannya adalah
lestarikan dan kembangkan (A) Pelantikan para guru di
(B) Budaya syawalan kita harus selenggarakan di gedung serba
melestarikan dan mengembangkan guna.
(C) Budaya syawalan kita harus (B) Kemarin, Gubernur Yogyakarta
lestarikan dan kembangkan meresmikan taman wisata di
(D) Budaya syawalan kita harus Sleman.
melestarikan dan kembangkannya (C) Jarak museum itu dari pusat kota
(E) Budaya syawalan harus kita kurang lebih 1 km.
lestarikan dan dikembangkan (D) Menjelang operasi, pasien itu
berdoa kepada Tuhan Yang Maha
21. Kalimat berikut ini tidak baku, KECUALI Esa.
(A) Dengan kejadian ini, kepolisian (E) Secara kebetulan lokasi gedung itu
menghimbau kepada warga yang berada di antara perbatasan dua
mempunyai anggota keluarga yang wilayah.
mengalami gangguan jiwa
hendaknya segera dibawa berobat 23. Dari buku-buku dan banyak
atau ditempatkan di tempat yang tulisan/karangan di berbagai media
aman. massa yang beredar, dapat dilihat
(B) Mengenai ketidaksamaan habisnya betapa banyak kaum wanita yang
masa bakti para kepala daerah merasa dikecewakan oleh pelayanan
terjadi karena pemilihannya selama kesehatan modern, khususnya di bidang
ini tidak sama. penyakit wanita.
(C) Kudapan nusantara sebagai suatu Kalimat di atas merupakan perluasan
produk yang bersifat khas tentunya dari kalimat
memiliki daya tarik tersendiri, (A) Media massa beredar.
apalagi jika ditunjang dengan (B) Wanita dilihat.
inovasi, secara otomatis nilai jualnya (C) Wanita dikecewakan.
menjadi lebih tinggi. (D) Pelayanan mengecawakan.
(D) Berkaitan jadwal pelaksanaan ujian (E) Penyakit wanita.
nasional dimajukan dan
direncanakan akan diselenggarakan
bulan April mendatang, karenanya
para siswa diharapkan
mempersiapkan diri lebih awal agar
angka ketidaklulusan dapat
diminimalkan.
(E) Era globalisasi yang secara tidak
langsung menjadikan persaingan
dalam berbagai bidang kehidupan,
termasuk usaha jasa boga yang
semakin kompetitif.

46 Sony Sugema College


Sony Sugema College
24. Faktor deteritorialisasi yakni dalam
pengertian reduksi ruang dan waktu,
telah mengubah paradigma
komunikasiantar individu di kota-kota
besar untuk tidak lagi harus melalui
sebuah modus tatap muka, yang sedikit
banyak harus mempertimbangkan
tersedianya tenaga dan luangnya waktu
sehingga tatap mukanya dapat diatur di
suatu” tempat”
Kalimat inti dari kalimat luas tersebut
adalah
(A) Faktor deteritorialisasi mengubah
paradigma.
(B) Deteritorialisasi mereduksi ruang
dan waktu.
(C) Komunikasi di kota-kota besar tidak
melalui tatap muka.
(D) Komunikasi tatap muka dapat diatur
tempatnya.
(E) Tatap muka harus
mempertimbangkan tenaga dan
waktu.

Sony Sugema College 47


8. Tugas Mandiri Bahasa Indonesia

Kekhawatiran akan merosotnya “imagined” adalah entitas abstrak yang


nasionalisme dan terjadinya disintegrasi berisikan bayangan-bayangan, cita-cita, dan
nasional integrasi – merebak di mana-mana harapan-harapan bahwa nasion akan
akhir-akhir ini. tumbuh makin kuat dan mampu memberikan
Kebangsaan (nationality) dan rasa perlindungan, kenyamanan, dan
kebangsaan (nationalism) saling berkaitan kesejahteraan hidup. Selama 60 tahun
satu sama lain. Rasa kebangsaan, biasanya imajinasi itu hidup dan terepelihara, rakyat
juga disebut nasionalisme, adalah dimensi terus menggantungkan harapan bahwa
sensoris – meminjam istilah Benedict suatu waktu kemakmuran, keadilan, dan
Anderson (1991[1983]) Imagined kesejahteraan itu akan terwujud.
Communities – merupakan konsep Secara internal kita berhadapan
antropologi yang tidak semata-mata dengan fenomena meningkatnya kemiskinan,
memandang nasionalisme sebagai prinsip korupsi, konflik-konflik kepentingan partai
politik. dan golongan, kesenjangan sosial-ekonomi,
Rasa kebangsaan atau yang kerap ketidakpastian pelaksanaan hukum, jurang
kali juga disebut nasionalisme adalah topik generasi, dan banyak lagi; secara eksternal
baru dalam kajian antropologi. Nasionalisme kita menghadapi fenomena global, seperti
sebagai ideologi negara-negara modern liberalisasi ekonomi, memudarkan ideologi,
sejak lama adalah rubrik ilmu politik, dan meningkatnya komunikasi lintas batas
sosiologi makro, dan sejarah. negara dan kebudayaan.
Perhatikan antropologi terhadap Achmad Fedyani Saifuddin dalam Kompas
nasionalisme menempuh jalur yang berbeda Cyber Media, 12 Agustus 2006.
dari disiplin-disiplin tersebut yang
menempatkan negara sebagai titik awal 1. Kata integrasi dalam tulisan di atas
pembahasan. Sejalan dengan tradisinya, merujuk pada proses ...
antropologi menempatkan nasionalisme (A) perpecahan bangsa
bersamaan dengan negara karena kesetiaan, (B) persebaran bangsa
komitmen, dan rasa memiliki negara tidak (C) perluasan bangsa
hanya bersifat instrumental – yakni (D) persatuan bangsa
keterikatan oleh prinsip politik – melainkan (E) persamaan bangsa
juga bersifat sensorik yang berisikan
sentimen-sentimen, emosi-emosi, dan 2. Dalam tulisan di atas, kata imajinasi
perasaan-perasaan. merujuk pada hal berikut, KECUALI :
Sebagai suatu konstruksi posteriori, (A) cita-cita (D) nostalgia
maka nasionalisme harus dijaga, diperihara, (B) harapan (E) obsesi
dan dijamin mampu menghadapi perubahan (C) ekspektasi
zaman. Selain itu, nasion sebagai suatu yang
48 Sony Sugema College
Sony Sugema College
3. Dalam tulisan di atas, nasionalisme yang 6. Penggunaan tanda baca yang mengikuti
tidak hanya bersifat instrumental – yakni EYD terdapat dalam kalimat
keterikatan oleh prinsip politik – (A) Apa yang dimakan anak itu?
,melainkan juga bersifat sensorik. Jawabnya, “Kue”.
Kalimat di atas tidak baku agar menjadi (B) Saya hendak memaparkan kekuatan
baku harus diperbaiki dengan cara pengungkapan “Chairil Anwar”
(A) Mengganti kata melainkan dengan dalam sajak Aku.
tetapi (C) Oleh karena itu upaya rehabilitasi
(B) Membuang kata dalam dan konservasi alam harus dirancang
(C) Tidak perlu memakai tanda pisah (--) partisipatif sehingga dapat
(D) Sebelum kata yakni harus memakai dipraktekkan secara
tanda baca koma berkesinambungan.
(E) Sebelum kata melainkan tidak perlu (D) Puisi Chairil Anwar berjudul 1943
tanda baca koma mencerikatan suasana zaman
penjajahan Jepang.
4. Modal dasar bangsa Indonesia dalam (E) Kendati pemakaian jamu cukup
membangun nasionalisme adalah marak dan industri jamu juga
sebagai berikut, kecuali ... bekembang, sungguh
(A) bahasa persatuan Bahasa Indonesia mengherankan kalau ternyata
(B) sektarianisme yang dibangun atas produksi tanaman obat beberapa
dasar karakteristik kelompok etnis tahun belakangan stagnan.
(C) latarbelakang historis yang relatif
sama 7. Nasionality dan nasionalism saling
(D) simbol-simbol budaya bersama pada berkaitan satu sama lain. Rasa
semua kelompok etnis kebangsaan biasanya juga disebut
(E) persamaan nasib pada masa lalu nasionalisme.
sebagai bangsa terjajah Kata yang tepat untuk mengganti kata
yang bercetak tebal di atas adalah
5. Masyarakat terbiasa menyumbangkan (A) bangsa– bersifat kebangsaan
sebagian harta benda yang dimilikinya (B) kebangsaan dan rasa kebangsaan
untuk membantu meringankan beban (C) rasa kebangsaan - kebangsaan
kelompok masyarakat yang kurang (D) bersifat kebangsaan- rasa
beruntung. kebangsaan
Kalimat tersebut berasal dari kalimat inti (E) kebangsaan- jiwa kebangsaan
(A) Masyarakat menyumbangkan harta.
(B) Masyarakat terbiasa membantu.
(C) Harta benda meringankan.
(D) Masyarakat kurang beruntung.
(E) Masyarakat membantu
meringankan.

Sony Sugema College 49


8. Semburan lumpur,air, dan gas baru 10. Dimensi sensoris pengertian
keluar dari halaman belakang rumah nasionalisme dari Benedict Anderson
seorang penduduk, warga Desa Mindi, menempatkan nasionalisme ... sebagai
Kecamatan Porong, Kabupaten Sidoarjo. prinsip politik ... ensitas abstrak berupa
Semburan itu merupakan semburan ke- bayangan diri dan lingkungan
59 yang muncul di sekitar pusat bangsanya.
semburan utama. Konjungsi korelasi yang tepat untuk
Menurut seorang ahli dari Leader bagian yang rumpang kalimat di atas
Team Fergano, perusahaan yang adalah
mengawasi gas-gas berbahaya di sekitar (A) tidak ... tetapi ...
pusat semburan, semburan itu sama (B) baik... maupun ...
dengan 58 semburan liar sebelumnya. (C) sedemikian rupa ... sehingga ...
Semburan itu juga tidak berbahaya dan (D) bukan .... tetapi ...
tidak akan membesar. Kalau dibiarkan (E) bukan...melainkan...
semburan itu akan mengecil sendiri.
Untuk menutup semburan itu,hari ini 11. Potensi kekayaan alam yang terkandung
akan dimasukkan sekitar 100 kilogram di wilayah perairan Indonesia, seperti
semen ke dalam lubang asal semburan. perikanan, minyak dan gas bumi,
Ide pokok paragraf kedua teks di atas pariwisata, mineral (emas, mangan, dan
adalah budidaya laut) belum dimanfaatkan
(A) pengawasan gas oleh tim ahli secara optimal untuk memanfaatkan
(B) pendapat tentang semburan gas sumber daya tsb, dibutuhkan ilmu
(C) munculnya semburan liar pengetahuan dan teknologi yang maju.
(D) mengecilnya semburan liar Kalimat di atas merupakan perluasan
(E) penutupan lubang semburan dari kalimat inti …
(A) Potensi kekayaan alam belum
9. Fakta dalam teks tersebut yang tepat dimanfaatkan.
adalah (B) Pemanfaatan potensi alam di
(A) Semburan lumpur baru merupakan Indonesia.
semburan ke-59. (C) Potensi kekayaan alam
(B) Semburan liar itu tidak berbahaya membutuhkan IPTEK.
seperti semburan yang lain. (D) Potensi kekayaan alam meliputi
(C) Semburan liar itu akan mengecil minyak dan gas bumi.
dengan sendirinya. (E) IPTEK dibutuhkan untuk
(D) Semburan liar itu sema dengan memanfaatkan SDA.
semburan sebelumnya.
(E) Untuk menutup semua semburan
liar itu, diperlukan sekitar 100
kilogram semen.

50 Sony Sugema College


Sony Sugema College
12. Kalimat berikut ini merupakan kalimat (C) Mitigasi struktural biasanya terkait
baku, KECUALI dengan bangunan tahan gempa,
(A) Karena orientasi pasar berubah, namun yang nonstruktural
ratusan petambak di kawasan Benoa berhubungan dengan pendididkan
dan pesisir timur, Bandung dan Bali, dan tata guna lahan.
beramai-ramai mengubah jenis (D) Mitigasi struktural biasanya terkait
budidaya ikan bandeng ke udang dengan bangunan tahan gempa,
vaname sedangkan yang nonstruktural
(B) Gempa di Ternate ini diperkirakan berhubungan dengan pendididkan
sebagai akibat pertemuan antara dan tata guna lahan.
Lempeng Tektonik Indo Australia (E) Mitigasi struktural biasanya terkait
dan Pasifik. dengan bangunan tahan gempa,
(C) Setelah promosi wisata ke Australia melainkan yang nonstruktural
dan beberapa negara di Eropa, berhubungan dengan pendididkan
Pemerintah Provinsi Bali berpromosi dan tata guna lahan.
ke beberapa wilayah di Cina.
14. Objek wisata Pangandaran menyediakan
(D) Dari semakin meningkatnya jumlah
transfortasi rekreasi untuk memudahkan
pengangguran di Indonesia
wisatawan menikmati keindahan pantai.
berdampak kepada negara dan
Wisatawan domestik maupun
masyarakat.
mancanegara dapat menggunakan
(E) Kejahatan maya atau cyber crime
transformasi untuk menikmati
merupakan jenis kejahatan masa
keindahan alam. Para pedagang ikut
depan yang menunjukan gejala yang
meramaikan situasi Pantai
meningkat, termasuk di Indonesia.
Pangandaran.Di sepanjang tepi Pantai
Pangandaran terlihat berjejer perahu
13. (1) mitigasi struktural biasanya terkait
untuk disewakan. Dengan biaya Rp
dengan bangunan tahan gempa
1.000,00 saja per orang, para wisatawan
(2) yang nonstruktural berhubungan
dapat berputar di sekitar pantai dan
dengan pendidikan dan tata guna
menikmati keindahan taman laut.
lahan
Kalimat yang tidak koheren dalam teks di
Penggabungan kedua klausa tersebut
atas adalah kalimat
yang benar adalah … .
(A) pertama
(A) Mitigasi struktural biasanya terkait
(B) kedua
dengan bangunan tahan gempa,
(C) ketiga
tetapi yang nonstruktural
(D) keempat
berhubungan dengan pendididkan
(E) kelima
dan tata guna lahan.
(B) Mitigasi struktural biasanya terkait
dengan bangunan tahan gempa
akan tetapi, yang nonstruktural
berhubungan dengan pendididkan
dan tata guna lahan.

Sony Sugema College 51


15. Pendidikan merupakan proses (C) Biskuit sebagai satu-satunya camilan
pendewasaan bangsa yang akan mnjadi yang menyehatkan.
modal utama pembangunan. Karena itu, (D) Penyediaan camilan dalam keluarga
pemerintah Indonesia .… anggaran oleh seorang ibu.
belanja yang sangat besar di bidang (E) Penyediaan camilan yang menjadi
pendidikan. alternatif para ibu.
Kata yang tepat untuk mengisi bagian
yang kosong kalimat di atas adalah 18. Dua minggu lalu, sekitar 200-an
(A) memanipulasi perempuan berkumpul di sekitar hotel
(B) mengakomodasi terkenal. Mereka mengenakan “busana
(C) menginvestasikan nasional” yang umumnya berupa kain
(D) mengasuransikan sarung dari berbagai daerah dengan
(E) mengalkulasi kebaya beragam bentuk. Begitu banyak
perempuan berkumpul untuk
16. Teknologi pertelevisian dalam dua rezim, memeriahkan hari jadi sebuah
hampir tiga puluh tahun, didominasi oleh perkumpulan yang mendedikasikan
pemerintah sebelum akhirnya kegiatan mereka untuk mempromosikan
diliberalisasikan pada tahun 1987 - 1989 busana nusantara.
yang membebaskan masyarakat dari Inti paragraf di atas adalah
sentralisasi informasi. (A) Mereka berkumpul di hotel untuk
Pengertian sentralisasi dalam kalimat peragaan busana nasional.
tersebut adalah (B) Mereka berkumpul untuk
(A) pemusatan mengadakan promosi pakaian
(B) pengikatan daerah.
(C) penyeragaman (C) Mereka berkumpul dalam rangka
(D) penyatuan ulang tahun organisasi.
(E) pengekangan (D) Mereka berkumpul untuk
menghadiri promosi mode busana
17. Dengan mengudap makanan ringan, nasional.
perut akan merasa hangat. Apalagi (E) Mereka berkumpul untuk saling
kalau camilan itu dimakan pada waktu bersilaturahmi.
cuaca mendung. Ibu rumah tangga bisa
19. Kata serapan yang berasal dari bahasa
menyediakan camilan yang
asing digunakan secara tepat dalam
menyehatkan setiap hari. Untuk ibu yang
kalimat
hobinya masak, hal itu bukan masalah.
(A) Desainer mesin pertanian itu dibuat
Akan tetapi, apa jadinya kalau ibu itu
oleh mahasiswa fakultas teknik
seorang wanita karier yang sibuk dengan
(B) Pada bulan Agustus biasanya
urusan di luar rumah ? Kalau sudah
ditayangkan film dokumentasi di
begini, siapkn saja biscuit siap saji yang
televisi.
menyehatkan.
(C) Sebagian musafirin menempuh
Pokok yang dibicarakan dalam teks di
perjalanan tiga bulan melalui jalan
atas adalah
darat.
(A) Makanan camilan yang harus
tersedia di setiap rumah tangga.
(B) Fungsi camilan bagi setiap orang.
52 Sony Sugema College
Sony Sugema College
(D) Sumber pengetahuan ialah (C) Kesibukan itu membuat mereka
pengamatan dan pengalaman atau senang.
empiris. (D) Bagaimanakah mereka akan
(E) Orang itu dapat memanfaatkan kesepian dengan kesibukan itu ?
waktu secara efektif sehingga sukses (E) Kegiatan yang demikian itulah yang
dalam usahanya. menghibur mereka di lembaga itu.
20. Untuk mengumpulkan data, peneliti
22. Bedah jantung telah berkembang sangat
melakukan teknik wawancara mendalam
diakhir abad ke-20. banyak peralatan
terhadap tiga belas informan yang
operasi yang canggih dan juga berbagai
dianggap menguasai permasalahan
teknik operasi ditemukan. Pada mulanya
yang diteliti. Selain itu, peneliti juga
operasi yang dianggap sebagai suatu
mengamati objek-objek yang relevan
jenis operasi yang sangat besar dan
dengan masalah yang diteliti. Apabila
beresko tinggi. Tindakan operasi
dalam pengamatan tersebut terdapat
merupakan suatu bentuk pengobatan
hal yang perlu diperjelas, peneliti
penyakit jantung yang terakhir apabila
melakukan wawancara dengan para
tidak ada jalan lain lagi.
subjek amatan. Karena itu, dalam
Kalimat berikut ini yang tepat untuk
pelaksanaan penelitian ini peneliti
mengakhiri paragraf di atas adalah
memanfaatkan alat perekam elektronik,
(A) Oleh Karena itu, operasi tidak
selain itu digunakan juga catatan
dilakukan pada pasien dengan
lapangan.
keadaan yang kritis.
Bentukan kata yang tepat dalam
(B) Oleh Karena itu, operasi sering
paragraf tersebut adalah
dilakukan pada pasien dengan
(A) mengumpulkan
keadaan yang kritis.
(B) permasalahan
(C) Oleh Karena itu, operasi jarang
(C) diperjelas
dilakukan pada pasien dengan
(D) amatan
keadaan yang kritis.
(E) digunakan
(D) Oleh Karena itu, operasi menjadi
21. Pada lembaga itu para lansia
satu-satunya harapan bagi pasien
menyibukkan diri dengan berbagai
dengan keadaan kritis.
kegiatan. Mereka mengikuti kegiatan
(E) Oleh Karena itu, operasi bagi pasien
percakapan bahasa Ingris, bekerja
dengan keadaan yang kritis memang
dengan komputer, melukis aau bermain
dilakukan.
musik. Mereka juga dapat memilih
cabang olahraga yang mereka senangi
dan yang sesuai dengan usia mereka.
Bahkan mereka dapat membantu dokter
di pusat pelayanan kesehatan yang
tersedia di lembaga itu.
Kalimat yang paling tepat untuk
menutup paragraf di atas adalah
(A) Dengan demikian, mereka tidak bisa
menjadi pikun.
(B) Itulah kegiatan mereka sehari-hari.
Sony Sugema College 53
23. Penerapan EYD yang tepat terdapat 25. Tanggal 1 April 2006, di teras Hotel
dalam kalimat Afamia Curitiba Brazil, di mana delegasi
(A) Pahit-getirnya masa penjajahan Forum Masyrakat Adat Internasional
dialami pejuang yang bolak-balik mengenai Keanekaragaman Hayati
masuk hutan agar desanya tidak menginap seorang ketua suku India
dibumihanguskan. menangis sambil berkata, “ternyata
(B) Desentralisasi yang diberlakukan mereka akan tetap membunuh kami,
saat ini secara “de facto” sudah seperti yang sudah-sudah”.
menjangkau seluruh desa. Kalimat tersebut akan menjadi baku jika
(C) Sorak sorai membahana saat para ejaannya diperbaiki dengan cara berikut
atlet Indonesia mempersembahkan KECUALI
medali emas di evant internasional. (A) menuliskan ketua suku dengan huruf
(D) “Naik-naik ke puncak gunung, tinggi- awal kapital.
tinggi sekali, kiri kanan ...,” lantun (B) menambahkan tanda koma (,)
Ibu Kasur. setelah kata Curitiba.
(E) Pernikahan Siti Nurhaliza yang (C) mengganti kata di mana dengan
“hanya” menghabiskan dana tempat.
sepuluh milyar itu dihadiri artis (D) menambahkan tanda koma (,)
manca negara. setelah kata menginap.
(E) menuliskan kata ternyata menjadi
24. Momentum halal bi halal pada Hari Raya Ternyata.
Idul Fitri dapat dijadikan media untuk
mengevaluasi gerak langkah, sikap serta
perilaku kita bersama dalam
meningkatkan kinerja dan pengabdian
kepada masyarakat, bangsa dan negara.
Kalimat tersebut akan menjadi kalimat
baku apabila ejaannya diperbaiki dengan
cara berikut, KECUALI
(A) menggabungkan penulisan halal bi
halal menjadi halalbihalal.
(B) mengubah penulisan Hari Raya Idul
Fitri menjadi hari raya Idul Fitri.
(C) menambahkan koma (,) sebelum
kata serta.
(D) mengubah kinerja menjadi kerja.
(E) menambahkan koma (,) sebelum
kata dan.

54 Sony Sugema College


Sony Sugema College
9. Tugas Mandiri Bahasa Indonesia

Peradaban manusia umumnya lahir terserap ke tanah. Dalam kondisi normal,


dari sebuah permukiman di tepi atau seperti daerah hutan yang masih hijau,
sepanjang sungai. Lutetia cikal bakal Paris koefisien limpas paling besar adalah 15
dibelah Sungai Seine yang indah. Sebaliknya, persen dari total air hujan. Jumlah curah
Sungai Ciliwung yang menjadi urat nadi hujan selama puluhan tahun berada dalam
Buitenzorg-Batavia justru sumber bencana angka yang relatif sama. Banjir justru terjadi
banjir tahunan semasa pemerintahan anak akibat menyusutnya daerah resapan karena
negeri berkuasa di Jakarta. perubahan peruntukan lahan secara drastis
Keindahan tepian sungai kota-kota dan membabi-buta. Bahkan, di sejumlah area
di Eropa, seperti London dengan Sungai pada daerah aliran sungai Ciliwung, koefisien
Thames, Bupadest di Hongaria, atau St limpas sudah mencapai 95 persen di daerah
Petersburg di Rusia, terkenal di seantero industri, pertokoan, dan hunian. Sedangkan
dunia sebagai kombinasi hunian dan di daerah pertanian berkisar 21 sampai 65
keindahan alam. Demikian pula di Asia. persen.
Shanghai Bund – tepian indah Kota Shang Hai
– berada di tepi Sungai Huang Pu, Saigon di Sumber : kompas Cyber Media tanggal 29 Januari 2005.

tepi Sungai Mekong, dan Bangkok dibelah


Chao Praya sangatlah eksotis. Peradaban 1. Buitenzorg-Batavia dalam tulisan di atas
dan peningkatan taraf hidup berkembang di merupakan istilah Belanda yang
sepanjang tepi sungai kota-kota tersebut. menunjukkan hubungan antarkota ...
Dari kehidupan sebagai penghuni (A) Bandung – Jakarta
goa, manusia membangun peradaban di (B) Bogor – jakarta
tepian sungai yang subur hingga kini. (C) Bekasi – Tanggerang
Namun, ironi kehidupan menjadi takdir (D) Depok – Jakarta
mutlak bagi warga Jakarta dan masyarakat (E) Bogor – Bekasi
Indonesia penghuni kawasan hilir daerah
aliran sungai. Alih-alih memiliki kota yang 2. Istilah potret “Indonesia Way” dalam
indah, jamban terapung, lautan sampah, tulisan di atas adalah istilah penulis
limpah pabrik, dan permukiman kumuh untuk menggambarkan ...
menjadi citra “Ciliwung” sekaligus potret “ (A) jalur sungai yang ada di Indonesia
Indonesia Way”. Alhasil, bencana banjir dan (B) jalur lalu lintas di Sungai Ciliwung
kekeringan menjadi ritus tahunan di saat (C) budaya Indonesia yang
negeri lain mampu membangun peradaban berhubungan dengan sungai
ideal: keserasian manusia dan ekologi sungai. (D) bencana banjir di Sungai Ciliwung
Pakar konservasi air Naik Sinukaban (E) sungai di Indonesia sebagai
mengatakan, degradasi lingkungan itu penyebab bencana banjir
langsung berdampak pada kenaikan koefisien
limpas air, yaitu jumlah air hujan yang tidak
Sony Sugema College 55
3. Degradasi lingkungan dalam tulisan di 7. Ada beberapa penyebab kurang
atas merujuk pada hal-hal berikut, tertariknya investor asing untuk
KECUALI ... melakukan investasi di Indonesia.
(A) menurunnya kualitas air sungai Masalah tumpang tindihnya wilayah
(B) kerusakan lingkungan pertambangan dengan UU Kehutanan
(C) menurunnya daya dukung yang menyngkut hutan lindung di
lingkungan Indonesia cukup membuat para investor
(D) meningkatnya kerusakan daerah ketakutan. ….. tidak adanya kepastian
aliran sungai hukum, tabilitas politik, keamanan, dan
(E) kemampuan lingkungan menyerap masalah perpajakan juga menambah
air keraguan para investor ….. . Pemerintah
di negara lain malah mendukung
4. Koefisien limpas dari total air hujan di investasi pertambangan dengan
daerah aliran sungai yang tidak menerapkan peraturan pro-investor.
menyebabkan banjir adalah : Kata yang tepat untuk melengkapi
(A) kurang dari 15 persen paragraf di atas adalah
(B) kurang dari 30 persen (A) walaupun demikian, sepertinya
(C) lebih dari 30 persen (B) selain itu, padahal
(D) lebih dari 60 persen (C) selain itu, bahkan
(E) kurang dari 60 persen (D) walaupun, padahal
(E) walaupun, mestinya
5. Fenomena degradasi lingkungan
sepanjang sungai-sungai di Indonesia 8. Pada kalimat Sambutan hangat terhadap
menggambarkan hal-hal di bawah ini, deklarsi kemerdekaan Timor Leste dan
KECUALI ... pernyataan politik Indonesia beberapa
(A) peradaban rendah tahun silam harus dikaji secara serius.
(B) keterbelakangan Yang harus dikaji secara serius adalah
(C) peradaban tinggi (A) Sambutan hangat terhadap deklarsi
(D) pesatnya ekspansi manusia Timor Leste.
(E) kemiskinan yang terstruktur (B) Sambutan hangat terhadap deklarsi
kemerdekaan.
6. Pemakaian tanda baca yang benar (C) Sambutan hangat terhadap deklarsi
terdapat pada kalimat kemerdekaan Timor Leste dan
(A) Wacana lisan yang mementingkan pernyataan politik Indonesia.
“isi” dapat berupa pidato, ceramah, (D) Deklarsi kemerdekaan Timor Leste
dakwah, kuliah atau deklarasi. dan pernyataan politik Indonesia.
(B) Semua peserta TO SSC, yang tidak (E) Kemerdekaan Timor Leste dan
membawa kartu peserta, harus pernyataan politik Indonesia.
melaporkan diri ke panitia.
(C) Ir. Sahala S.P.S tinggal di Jalan
Pelesiran 83 , Bandung, Jawa Barat.
(D) Ia harus pulang sekarang karena
sakit.
(E) Sahabat saya Amin tinggal di
Manokwari.
56 Sony Sugema College
Sony Sugema College
9. Peternak itu sedang membersihkan 11. ... . Anda harus menjadi seorang
kandang. mengamat manusia. Bila Anda benar-
Kalimat manakah yang memuat kata benar mampu mengerti manusia atau
yang makna prefiks pe-nya sama dengan orang, tahu akan ketakutan, harapan,
makna prefiks pada peternak? dan impian mereka Anda akan memiliki
(A) Semua pelatih itu praktisi. kemampuan mengembangkan hubungan
(B) Para pelamar tampak memadati tersebut. Bicaralah dengan orang-orang.
aula sebuah gedung pemerintah. Dengarkanlah keinginan hati mereka.
(C) Kini beliau menjadi penerjemah Amatilah mereka dan pelajarilah cara
handal. mereka berpikir. Tentu saja Anda harus
(D) Ayahnya seorang pedagang yang membaca buku dan mendengarkan pita
sukses di kota kelahirannya. kaset (raihlah apa yang Anda peroleh
(E) Para petatar itu semuanya guru dari kebijakan orang lain) namun, jangan
madrasah swasta. abaikan bergaul dengan orang lain dan
pelajarilah tabiat mereka. Ini adalah satu
10. Sampai akhir tahun 2002, wisatawan gaya hidup yang harus dikembangkan
Jepang yang langsung ke Bali masih bukan satu studi ilmiah.
sekitar 300.000 orang meskipun secara Kalimat yang tepat untuk melengkapi
keseluruhan terjadi penurunan jumlah kalimat pertama paragraf di atas adalah
kunjungan wisatawan ke Bali akibat (A) Pergaulan sangat penting dalam
peledakan bom. masyarakat.
Inti kalimat di atas adalah (B) Mengembangkan hubungan positif
(A) Wisatawan Jepang langsung ke Bali dengan orang lain sebenarnya
(B) Jumlah wisatawan terjadi bertujuan pada satu hal.
penurunan (C) Kita harus dapat bergaul di tengah
(C) Wisatawan Jepang sekitar 300.000 masyarakat .
orang (D) Gaya hidup harus dapat
(D) Terjadinya penurunan jumlah dikembangankan di tengah
wisatawan akibat peledakan bom masyarakat.
(E) Wisatawan Jepang menurun (E) Masyarakat harus bertanggung
jawab.

12. Dalam kalimat berikut ini, terdapat


bentukan kata yang salah, KECUALI
(A) Dalam persaingan secara akademis,
perbuatan mendeskriditkan sesama
calon dipandang sebagai tindakan
yang bernilai rendah.
(B) Berbareng dengan kebakaran itu
turun hujan yang sangat lebat
sehingga jalan-jalan bergenang air.
(C) Setiap calon perwakilan mahasiswa
diberi kesempatan untuk
mengkampanyekan program-
programnya.
Sony Sugema College 57
(D) Sudah dua bulan ini pemrosesan 15. Pemakaian yang tepat konjungsi
perkara itu belum juga selesai antarkalimat yang menyatakan adanya
sehingga menimbulkan perkara hal, peristiwa, atau keadaan lain diluar
baru. dari yang telah dinyatakan
(E) Tujuan penyetandaran ejaan bahasa sebelumnya adalah
Indonesia merupakan salah satu (A) Pak Daniel terkena penyakit kencing
kegiatan dalam upaya pembakuan manis. Selain itu, dia juga mengidap
bahasa Indonesia. tekanan darah tinggi.
(B) Masalah yang dihadapinya memang
13. Pada saat akan dilaksanakannya gawat. Sesungguhnya, masalah itu
pemilihan kepala desa, banyak orang sudah diramalkan sebelumnya.
berkasak-kusuk untuk menjatuhkan dan (C) Simon sudah tahu soal itu. Bahkan,
melemahkan kewibawaan calon yang ia bisa menanganinya.
menjadi lawannya. (D) Keadaan sudah mulai aman. Akan
Perbuatan orang tersebut dikatakan tetapi, kita harus tetap waspada.
(A) mengeksploatasi. (E) Sudah tiga hari Yanti tidak sekolah.
(B) memfalsifikasi. Oleh karena itu, ia mendapat surat
(C) mendiskualifikasikan. peringatan dari sekolah
(D) mendiskriminasikan.
(E) mendeskreditkan. Usaha perlindungan telah menyebabkan
timbulnya tanah kritis. Tanah kritis ini
14. Dengan kompetisi yang sehat, bukan selanjutnya menimbulkan berbagai bencana
hanya penjual yang diuntungkan, besar yang mengancam ketentraman hidup
melainkan pembeli juga sangat manusia dan menghalangi tercapainya
diuntungkan karena mendapat harga tujuan pembangunan ekonomi. Seluruh curah
yang murah. uang pemerintah dengan satuan hitung
Pasangan kata yang memiliki hubungan miliar itu tidak ada gunanya dituangkan
makna yang sama terdapat pada kalimat. dalam bentuk waduk, dam, dan bendungan
(A) Seminar itu diikuti oleh guru SD dan di Pulau Jawa kalau hutan-hutan tidak ada.
guru SMP. Seluruh bangunan mahal itu hanyalah
(B) Mereka memesan pangsit kering merupakan mekanisme fisik pencegah banjir
dan basah masing-masing sebanyak dan jaminan buat hidup pertanian yang tidak
lima bungkus. akan mencapai sasaran. Dengan demikian,
(C) Dari kopral hingga jenderal bangunan yang mahal itu merupakan
diharuskan menaati peraturan pemborosan besar bila daerah-daerah aliran
sebagai prajurit negara. sungai di hulu-hulu dan di gunung-gunung
(D) Jalan-jalan di kota sangat padat tidak terlindung oleh selimut hutan. Di
sedangkan jalan di desa sangat samping itu tanah kritis menyebabkan erosi
lengang. dan banjir kronis di Pulau Jawa dan Madura
(E) Orang tua dan anaknya sama-sama sehingga setiap tahun menimbulkan kerugian
meninggal dalam musibah banjir. bernilai 1,3 juta ton beras.

58 Sony Sugema College


Sony Sugema College
16. Hubungan sebab – akibat yang dapat di 18. Bangsa Indonesia dan bangsa Jepang
ringkas dari seluruh isi teks di atas sama-sama memiliki sistem feodalisme
adalah yang berorientasi vertikal. Namun,
(A) Usaha perladangan menyebabkan bangsa Jepang memiliki budaya malu
timbulnya tanah kritis. yang tinggi. Mereka sangat malu jika
(B) Usaha perladangan menyebabkan melakukan kesalahan atau pelanggaran.
timbulnya tanah kritis dan bencana Di Indonesia, penerapan kedisiplinan kita
alam. saat ini masih terdorong oleh rasa takut
(C) Usaha perladangan menyebabkan terhadap sanksi atau kerugian yang
terancamnya pembangunan akan diterimanya apabila melanggar
ekonomi. peraturan. Rasa malu cenderung timbul
(D) Usaha perladangan menyebabkan apabila kesalahannya diketahui orang
timbulnya tanah kritis dan lain dan menyebabkan harga dirinya
pemborosan besar. berkurang. Orang Jepang dan orang
(E) Usaha perladangan menyebabkan Indonesia sama-sama memiliki daya
timbulnya tanah kritis sehingga tahan dalam menjalankan kehidupan
merugikan pembangunan ekonomi yang sulit dan penuh rintangan. Bedanya
dan menimbulkan berbagai bencana orang Indonesia menerjemahkan tahan
alam. menderita itu sebagai kepasrahan pada
nasib, hanya menjalani dan menerima
17. Proses pendaurulangan plastik menjadi hidup apa adanya sedangkan orang
bahan bakar minyak yang dilakukan Jepang melakukan usaha nyata untuk
mereka bertahap agar kandungan kimia mengubah kondisi yang kurang
pada plastik tersebut sesuai seperti yang menguntungkan itu nilai budaya yang
terdapat pada bensin dan solar. Proses mengutamakan kedamaian dan
ini dipisahkan antara damar eksposi dari keselarasan menyebabkan masayrakat
plastik. Kemudian, plastik dihancurkan Indonesia, seperti halnya masyarakat
agar dapat dicairkan dalam bak minyak Jepang, dikenal sebagai masyarakat
dengan suhu 400 derajad Celcius lebih. yang ramah tamah. Namun apabila
Masalah yang dibicarakan pada paragraf dikaitkan dengan segi ekonomis, orang
di atas adalah Jepang dikenal sangat hemat, cenderung
(A) Pencairan plastik dalam bak minyak sulit untuk mengeluarkan uangnya untuk
degan suhu 400 derajad Celcius hal-hal yang di luar rencana.
lebih. Ringkasan perbandingan yang sesuai
(B) Pemisahan damar eksposi dari dengan isi teks di atas adalah
plastik. (A) Bangsa Jepang dan bangsa Indonesia
(C) Kandungan kimia pada plastik memiliki persamaan dalam hal
disamakan seperti pada bensin dan sistem feodalisme, daya tahan tinggi
solar. terhadap penderitaan, dan sifat
(D) Sebuah proses pendaurulangan keramahannya. Bangsa Jepang dan
dalam lingkungan hidup. bangsa Indonesia berbeda dalam hal
(E) Proses pendaurulangan plastik budaya malu,cara menyikapi
menjadi BBM. penderitaan, dan segi ekonominya.

Sony Sugema College 59


(B) Bangsa Jepang dan bangsa Indonesia 20. Urutkan kalimat 1, 2, 3, 4, dan 5
memiliki persamaan dalam hal sehingga menjadi paragraf yang baik !
sistem feodalisme, sama-sama Asia 1. Ia menggunakan getah di perutnya
dan sifat keramahannya, bangsa untumembalut pasir yang
Jepang dan bangsa Indonesia melukainya.
berbeda dalam hal budaya malu, 2. proses itu bisa berlangsung
cara menyikapi penderitaan, dan bertahun-tahun sampai akhirnya
segi ekonominya. terbentuklah sebuah mutiara yang
(C) Bangsa Jepang dan bangsa Indonesia cantik dan berharga.
memiliki persamaan dalam hal 3. tiram berusaha mengatasi rasa
sistem feodalisme, daya tahan tinggi sakitnya.
terhadap penderitaan, dan sikap 4. mutiara sebenarnya terjadi dari
dalam menghadapi kesulitan. sebutir pasir yang memasuki tubuh
Bangsa Jepang dan bangsa Indonesia seekor tiram di dasar lautan.
berbeda dalam hal budaya 5. pasir ini menimbulkan rasa sangat
malu,cara menyikapi penderitaan, sakit di tubuhnya.
dan segi ekonominya. Urutan yang benar adalah
(D) Bangsa Jepang dan bangsa Indonesia (A) 1 – 2 – 3 – 4 – 5 (D) 1 – 4 – 2 – 3 – 5
memiliki persamaan dalam hal (B) 1 – 3 – 2 – 5 – 4 (E) 4 – 5 – 3 – 1 – 2
sistem feodalisme tetapi berbeda (C) 2 – 4 – 1 – 3 – 5
dari segi sistem ekonominya.
(E) Bangsa Jepang dan bangsa Indonesia 21. Salah satu di antaranya adalah acne
memiliki persamaan dalam hal vulgaris. Tidak ada seorang pun di dunia
sistem kebudayaan tetapi berbeda ini yang tidak pernah menderita penyakit
dari segi sistem ekonominya. itu. Menurut Syarif, Acne Vulgaris adalah
penyakit peradangan folikel sebasea
19. Kata serapan yang berasal dari bahasa
yang umumnya terjadi pada masa
asing digunakan secara tidak tepat
remaja dan dapat sembuh sendiri. Dalam
terdapat dalam kalimat
masyarakat umum, acne vulgaris biasa
(A) Erwin Gutawa terkenal karena
dikenal dengan sebutan jerawat.
kemampuannya
Kalimat topik yang tepat untuk
mengimprovisasikan sesuatu yang
melengkapi paragraf tersebut adalah
serba kurang menjadi hal-hal yang
(A) Acne vulgaris adalah penyakit kulit
istimewa.
biasa.
(B) Hubungan antara dua sifat
(B) Syarif mengemukakan jenis penyakit
kuantitatif yang disebabkan oleh
kulit.
lingkungan yang sama-sama
(C) Penyakit kulit banyak jenis dan
mempengaruhi kedua sifat disebut
macamnya.
korelasi lingkungan.
(D) Jerawat banyak jenis dan
(C) Kami tidak suka membeli barang
macamnya.
yang kuantitasnya kurang baik.
(E) Penyakit remaja bermacam-macam.
(D) Film Indonesia harus mengandung
nilai kultural yang tinggi.
(E) Masalah periodisasi sejarah
perjuangan bangsa Indonesia masih
terus mendapat perhatian para ahli.
60 Sony Sugema College
Sony Sugema College
22. Bentuklah paragraf yang belum lengkap 23. Kebebasan yang ditawarkan oleh
berikut ! internet telah terjadi karena untuk hadir
…. Ritual minum teh yang dijalankan di jaringan tersebut tidak dibutuhkan
penduduk Cina sejak ribuan tahun silam biaya yang besar. Di Indonesia jangan
menjadi bukti. Ritual serupa yang ada di harap Anda mendapatkan kebebasan
Jepang turut menjulangkan nama teh tersebut karena ada aturan-aturan yang
hijau. Tanaman yang memiliki nama telah menjadi kesepekatan berbagai ISP
ilmiah Camelia Sinensis ini diyakini (Internet Service Provider) yang
sanggup menangkal berbagai jenis tergabung dalam APJII (Asosiasi
penyakit. Mulai mencegah kanker dan Penyedia Jasa Internet Indonesia).
serangan jantung hingga memanjangkan Disamping itu, banyak ISP yang tidak
umur dan melangsingkan tubuh. mau mengambil risiko dengan
Keampuhan teh hijau jauh melebihi menghandirkan informasi-informasi
kendala teh hitam yang lebih dulu eksis. tertentu, terutama pornografi di server
Kalimat utama yang tepat untuk mereka. Padahal hingga saat ini, belum
memulai paragraf tersebut adalah ada undang-undang yang secara khusus
(A) Reputasi teh hijau sebagai minuman membahas permasalah seputar
yang diyakini berkhasiat tidak pornografi internet.
diragukan lagi. Simpulan paragraf di atas adalah
(B) Khasiat minun teh hijau Cina dapat (A) Kebebasan dan permasalahan yang
membangkitkan semangat. ditawarkan oleh internet di
(C) Ritual minum teh sangat Indonesia.
membangkitkan semangat. (B) Ketidakberanian Indonesia
(D) Teh mengandung antioksidan yang mengambil risiko dalam kebebasan
dapat mengendurkan pembuluh yang ditawarkan oleh internet .
darah. (C) Permasalah kebebasan internet di
(E) Apabila dikonsumsi secara teratur, Indonesia menimbulkan risiko yang
ekstrak teh hijau dapat mereduksi mengerikan.
risiko kanker. (D) Undang-undang yang membahas
permasalahan internet di Indonesia.
(E) Aturan yang telah disepakati oleh
Internet Service Provider dalam
kebebasan memakai internet di
Indonesia.

Sony Sugema College 61


24. (1) Sebelum perahu bertolak ke tengah 25. Presiden pernah berjanji dalam debat
laut, Suhardi disibukkan oleh tugas calon presiden bahwa pembangunan
membenahi semua perlengkapan. (2) kesehatan masyarakat mendatang
Kalau tempat yang dituju sudah dicapai bukan hanya sekedar program biaya
dan jaring telah ditebarkan, anak laki- pengobatan yang terjangkau oleh
laki sembilan tahun ini meloncat ke air rakyat. Visi ini masih terlalu dangkal jika
bersama sepotong bambu sepanjang dibandingkan dengan cita-cita reformasi
tiga meter sebagai pelampung. (3) Dia bidang kesehatan yang ingin mencapai
harus mencebur ke air waktu malam hari penduduk dengan perilaku hidup sehat,
sekali pun. (4) Tugasnya saat itu adalah mempunyai kemampuan untuk
membetulkan jaring atau menjaganya menjangkau pelayanan kesehatan yang
jangan sampai tersangkut di dalam air. bermutu secara adil dan merata, serta
(5) Untuk itu, ia mengapung di laut mempunyai derajat kesehatan setinggi-
selama satu setengah atau dua jam. (6) tingginya di seluruh Indonesia.
Dia akan kembali ke perahu berbarengan Pernyataan yang sesuai dengan kutipan
dengan naiknya jaring. di atas _____
Pernyataan yang sesuai dengan kutipan (A) Masyarakat mengharapkan
di atas adalah _____ pembangunan kesehatan.
(A) Tugas Suhardi hanya membetulkan (B) Presiden merancang sistem
jaring atau menjaganya jangan kesehatan masyarakat.
sampai tersangkut di dalam air. (C) Janji calon presiden tidak sebanding
(B) Suhardi hanya bekerja pada malam dengan visi reformasi.
hari. (D) Biaya pengobatan akan terjangkau
(C) Pekerjaan Suhardi mengapung di oleh rakyat.
laut selama satu setengah atau dua (E) Visi reformasi bidang kesehatan
jam. sesuai dengan janji calon presiden.
(D) Pekerjaan Suhardi
mengharuskannya mengapung di
laut sekitar satu setengah sampai
dua jam.
(E) Pekerjaan Suhardi baru akan dimulai
jika jaring sudah ditebar.

62 Sony Sugema College


Sony Sugema College
10. Tugas Mandiri Bahasa Indonesia

Bagi petani tambak penurunan 3. Kata keempat kalimat pertama bacaan di


kualitas air laut merupakan mimpi buruk atas mengandung makna ...
karena keadaan ini dapat menurunkan (A) sesuatu yang diturunkan
produksi hasil tambak. Untuk mengatasi (B) hal menurunkan
masalah ini, telah diperkenalkan data baru, (C) membuat turun
yaitu cara polikultur dengan menggunakan (D) cara menurunkan
rumput laut. (E) akibat diturunkan
Di daerah pertambakan Serang,
Banten, sejak beberapa tahun yang lalu 4. Pada kalimat keempat bacaan di atas,
rumput laut jenis Gracilaria sp mulai disebar bagian kalimat yang menunjukkan pokok
di areal pertambakan udang dan bandeng. pembicaraan adalah ...
Rumput laut mampu menyerap unsur-unsur (A) petani tambak bandeng
kimia dari air dan mengurai kotoran ikan (B) bandeng dan rumput laut
menjadi makanan ikan baru. Akibatnya, (C) biaya produksi bandeng per kg
bobot ikan naik. (D) produksi bandeng per kg
Bagi petani tambak bandeng, (E) biaya makanan ikan
dengan mencampur bandeng dan rumput
laut, biaya produksi bandeng per kg dapat 5. Penulisan kata yang salah terdapat
ditekan dari Rp 4.000,00 menjadi Rp dalam kalimat
1.500,00. Hasil produksi meningkat dari 150 (A) Kita tidak boleh terpancing oleh
kg per hektar menajdi 300 kg per hektar hasutan-hasutan yang dilakukan
setiap panenan. Ditambah lagi tiap dua pihak-pihak yang tidak bertanggung
bulan dari satu hektare tambak dapat jawab.
dihasilkan 1,5 ton rumput laut kering yang (B) Jabatan kepala kantor itu sudah
harganya Rp 3.000,00 per kg. diserahterimakan pada hari Sabtu
minggu yang lalu.
1. Usaha memperoleh hasil lebih seperti
(C) Pesta olahraga ittu akan
tercermin dari bacaan di atas pada alinea
mempererat persahabatan
dua dan tiga, lebih sesuai disebut ...
antarbangsa di Asia Tenggara.
(A) intensifikasi (D) rehabilitasi
(D) Pembedaan warga negara menjadi
(B) diversifikasi (E) spesifikasi
pribumidan nonpribumi sudah tidak
(C) ekstensifikasi
relevan dengan perkembangan
2. Jenis ikan yang terbaca di alinea 3 zaman.
bacaan di atas bertelur dan menetas di (E) Aktifitas para siswa akhir-akhir ini
... adalah mengadakan aksi turun ke
(A) danau (C) tampak (E) sungai jalan menentang tindakan
(B) laut (D) kolam kekerasan.

Sony Sugema College 63


6. Unsur serapan yang tertulis dalam (C) Saya bermaksud mengunjungi
kalimat berikut salah, KECUALI nenek.
(A) Siapakah nama gupernur yang baru? (D) Ibu berbelanja di pasar.
(B) Kataloog disediakan untuk mencari (E) Petinju itu berkeringat selama
buku di perpustakaan. bertanding.
(C) Kakakku diterima di fakultas technik
UI. 9. Bentuk perulangan yang menyatakan
(D) Anak-anak kecil menyukai film makna resiprokal terdapat dalam kalimat
kartun. (A) Kita perlu mewaspadai gerak-gerik
(E) Saya pernah melihat ikan arwana di orang asing yang baru datang ke
aquarium. desa ini.
(B) Sesama tetangga yang tinggal dalam
7. Penulisan yang benar menurut EYD
satu kawasan diharapkan dapat
adalah
saling membantu.
(A) Menurut Tanri Abeng, sinyalemen
(C) Polisi mematai-matai kegiatan
perkembangan dinasti-dinasti
kelompok itu selama satu tahun.
ekonomi yang muncul sekarang ini
(D) Hormat-menghormati sesama
adalah kekurang beranian
tetangga merupakan perilaku yang
mengambil resiko
baik untuk menciptakan kedamaian
(B) Faktor lain yang menghambat orang-
dalam bermasyarakat.
orang Indonesia bermental
(E) Ketika ditanya orang itu
wiraswasta ialah idealisme yang
memberikan jawaban yang berbelit-
mendambakan jabatan yang
belit sehingga penduduk di sana
mengandung predikat formal
menjadi marah.
seperti, menjadi pilot, dokter,
insinyur dan jaksa.
10. Pemakaian EYD yang tepat terdapat
(C) Pemikiran yang mengkhawatirkan
dalam kalimat
itu diungkapkan dalam ceramah Dr.
(A) Setiap karyawan tetap mendapat
N. Iskandar dari Lembaga Demografi
THR tiga ratus ribu rupiah sedangkan
Fakultas Ekonomi Universitas
karyawan harian mendapat dua
Indonesia dalam seminar
ratus ribu rupiah.
"Penyempurnaan Pola Dasar
(B) Perkara yang diajukan dalam sidang
Penerapan dan Motivasi Keluarga
justru bukan perkara penipuan
Berencana" di gedung Departemen
melainkan perkara penganiayaan.
Kesehatan.
(C) Prediksinya ternyata terbalik yakni
(D) Dalam pertemuan itu Dr. N. Iskandar
kelompok yang seharusnya menjadi
diwakili oleh Dr. Sutarsih.
juara malah tidak masuk dalam
(E) Apabila program KB gagal, maka
nominasi.
penduduk Indonesia akan berjumlah
(D) Semua yang hadir di ruang tersebut
tiga miliar.
diminta berdiri, ketika para menteri
8. Imbuhan ber yang mempunyai arti memasuki ruang pertemuan.
mempunyai terdapat pada kalimat (E) Minatnya untuk menjadi dokter
(A) Adik bersepeda ke sekolah. kandas karena hasil ujian tulis dan
(B) Murid berusaha menyelesaikan hasil wawancaranya tidak bagus.
tugasnya.
64 Sony Sugema College
Sony Sugema College
11. Pola gabungan kata sikat gigi sama 15. Ratusan warga Kelurahan Citeureup dan
dengan pola gabungan kata kelurahan Puspanegara Kecamatan
(A) sepasang sepatu Citeureup Kabupaten Bogor mengantri
(B) tenda biru minyak tanah, mengandung makna ...
(C) ikan asin (A) pembagian minyak tanah murah
(D) rumah sakit untuk warga Kecamatan Citeureup
(E) angkat kaki (B) pembagian minyak tanah gratis bagi
warga Kecamatan Citeureup
12. Kesinambungan pembangunan hanya (C) larangan penggunaan minyak tanah
mungkin dilaksanakan jika ketahanan bagi warga Kecamatan Citeureup
nasional dan kualitas trilogy (D) kelangkaan minyak tanah terjadi di
pembangunan meningkat. Kecamatan Citeureup
Kalimat di atas menggunakan frase (E) pembatasan jatah minyak tanah bagi
nomina berikut, KECUALI Kecamatan Citeureup
(A) kesinambungan pembangunan
(B) ketahanan nasional 16. Mereka berperangai ekstrem,
(C) kualitas trilogi pembangunan menjunjung tokoh yang disenangi secara
(D) trilogi pembangunan berlebihan dan sebaliknya
(E) mungkin dilaksanakan membelakukan secara buruk tokoh yang
dimusuhi.
13. “Kasus korupsi yang melibatkan mantan Kalimat tersebut menyatakan hubungan
pejabat itu dipetieskan oleh penyidik.” (A) penjumlahan dan kelanjutan
Istilah dipetieskan bermakna … (B) pertentangan dan akibat
(A) tidak dilanjutkan kasusnya (C) penjumlahan dan akibat
(B) tidak diumumkan (D) penjumlahan dan pertentangan
(C) tidak ditanggapi (E) pertentangan dan sebab
(D) diproses
(E) dimasukkan ke dalam peti 17. Yang merupakan kalimat majemuk
koordinatif adalah
14. Kota Bandung dikelilingi oleh (A) Ternyata pernyataan tersebut dapat
pegunungan. mengacaukan proses penelitian.
Kota Bandung berhawa dingin. (B) Lalu bagaimana perekonomian
Jika kedua kalimat di atas digabungkan rakyat dapat berkembang kalau
yang menimbulkan makna akibat, kata suasana politik tidak stabil.
penghubung yang tepat adalah, KECUALI (C) Mereka pasti akan menjadi peserta
…. terbaik dan akan menjaga nama baik
(A) oleh karena itu almamater.
(B) sehingga (D) Berbanggalah menjadi bangsa
(C) akibatnya Indonesia yang memiliki Pancasila
(D) karena sebagai dasar negara.
(E) memang (E) Bangsa Indonesia adalah bangsa
yang besar sekaligus ramah-tamah
murah senyum.

Sony Sugema College 65


18. Kami semua tengah menantikan angin 20. Cara pemerintah menggunakan tangan
baik untuk melancarkan aksi menentang besi dalam menjalankan roda
mereka. pemerintahannya tidak akan mendapat
Makna angin dalam kalimat di atas dukungan dari rakyat.
adalah Ungkapan tangan besi dalam kalimat di
(A) cuaca atas tidak bermakna
(B) keadilan (A) kekuasaan (D) persamaan
(C) suasana (B) kekuatan (E) pemaksaan
(D) kabar (C) penindasan
(E) kesempatan
21. Terdakwa menangis melihat
19. Pemakaian tanda koma (,) yang tidak pelaksanaan vonis eksekusi
tepat dijumpai dalam kalimat pembongkaran rumahnya.
(A) Ratusan warga Kelurahan Citeureup, Kata eksekusi bermakna
dan Kelurahan Puspanegara (A) hukuman seumur hidup
Kecamatan Citeureup Kabupaten (B) pelaksanaan hukuman
Bogor mengantri minyak tanah di (C) tahanan
pangkalan Jalan Mayor Oking (D) hukuman mati
Kabupaten Bogor. (E) hukuman penjara
(B) Migrasi penduduk miskin pedesaan
ke wilayah perkotaan telah 22. Wawasan Nusantara tidak hanya
melahirkan kemiskinan kota dengan bertujuan untuk mewujudkan
jumlah penduduk perkotaan yang kesejahteraan bagi bangsa Indonesia
terus meningkat ditopang oleh saja, tetapi juga ikut serta dalam
hadirnya penduduk miskin yang mewujudkan kebahagiaan bagi seluruh
mencapai 40% dari seluruh umat manusia.
penduduk kota. Penggunaan kata pada kalimat di atas
(C) Sementara itu, jika harga jual RSH dapat dihemat dengan menghilangkan
naik hingga Rp60,5 juta, penjualan (A) untuk dan saja
dipastikan akan anjlok. (B) untuk dan tetapi
(D) Di kota Oaxaca, sekitar dua ratus (C) untuk, saja, dan tetapi
polisi dengan pakaian pelindung dan (D) untuk, saja, dan bagi
membawa perisai maju ke pintu (E) untuk, tetapi, dan bagi
gerbang Universitas Oaxaca.
(E) Untuk memulihkan situasi politik di
Lebanon, diplomat asal Arab Saudi
diam-diam bernegosiasi untuk
membujuk Pemerintah Lebanon
agar bersedia membentuk
pemerintahan bersatu.

66 Sony Sugema College


Sony Sugema College
23. (1) perhatian pemerintah terhadap 25. PU : Segala jenis kekurangan di bidang
bidang industri, terutama industri kecil pendidikan itu berat dan
dan kerajinan semakin bertambah mendesak.
penting PK : Pendidikan di Indonesia menjelang
(2)pembangunan negara kita dewasa ini pelaksanaan Repelita I banyak
mulai memasuki tahap awal kekurangannya.
industrialisasi S :
Konjungsi yang tepat untuk
menggabungkan kedua klausa di atas Simpulan yang tepat untuk mengisi
adalah bagian yang rumpang adalah
(A) karena (A) Pendidikan di Indonesia menjelang
(B) sehingga pelaksanaan Repelita I menghadapi
(C) sedangkan masalah-masalah yang berat dan
(D) namun mendesak.
(E) karena itu (B) Di Indonesia menjelang pelaksanaan
Repelita I menghadapi masalah-
24. Permintaan sumbangan di jalan raya, masalah
ketidakdisiplinan pengemudi, (C) Pendidikan di Indonesia menghadapi
ketidaksiapan sarana dan prasarana masalah-masalah yang berat dan
jalan, juga kehadiran pedagang asongan mendesak.
merupakan beberapa faktor yang (D) Pendidikan menghadapi masalah-
menimbulkan kemacetan. masalah yang berat dan mendesak.
Berdasarkan hal-hal tersebut dapat (E) menjelang pelaksanaan Repelita I
ditarik satu simpulan berikut menghadapi masalah-masalah yang
(A) Kemacetan lalu lintas disebabkan berat dan mendesak.
oleh berbagai faktor.
(B) Kemacetan lalu lintas dapat diatasi
bila da kepedulian masyarakat dan
instansi terkait.
(C) Kemacetan dapat diatasi bila
masyarakat berperan aktif.
(D) Kemacetan lalu lintas belum dapat
diatasi secara maksimal.
(E) Kemacetan lalu lintas selalu
menimbulkan ketegangan fisik dan
mental.

Sony Sugema College 67


11. Tugas Mandiri Bahasa Indonesia

Dampak kenaikan harga minyak mengusulkan kenaikan harga RSH, hal itu
dunia mulai dirasakan pengembangan yang seharusnya sudah harus diantisipasi sejak
biasa membangun rumah sederhana sehat lama oleh para pengembang.
atau RSH untuk masyarakat berpenghasilan (Sumber : Harian Kompas, 17/3/08)
rendah. Para pengembang yang tergabung
dalam Dewan Pimpinan Daerah Real Estat 1. Judul yang tepat untuk bacaan di atas
Indonesia Jawa Timur mengusulkan kepada ialah
pemerintah melalui DPP REI untuk menaikan (A) Dampak Kenaikan Harga Minyak
harga jual maksimal RSH dari Rp 49 juta Dunia.
menjadi 60,5 juta per unit. (B) Dampak Pendapatan Per Kapita
Harga jual RSH sudah dua tahun Masyarakat Tidak Naik.
tidak naik, sedangkan harga-harga bahan (C) Hambatan-hambatan Pembangunan
bangunan saat ini sudah naik terutama besi. RSH Untuk Masyarakat.
Dampak kenaikan harga minyak dunia yang (D) Dampak Krisis Ekonomi Pada
saat ini sudah mencapai 110 dolar AS per Pembangunan Perumahan.
barel secara langsung maupun tidak (E) Harga RSH Diusulkan Naik Jadi
langsung menyebabkan harga material Rp60,5 juta.
bangunan naik 20-30 persen.
Penetapan plafon harga baru RSH, 2. Berdasarkan isi bacaan di atas,
diharapkan sudah dapat diberlakukan pernyataan berikut ini yang salah
pemerintah mulai April 2008. adapun adalah
kemampuan pengembang untuk (A) Para pengembang yang tergabung
merealisasikan penjualan RSH memang dalam Dewan Pimpinan Pusat Real
dilematis. Di satu sisi daya beli masyarakat Estat Indonesia mengusulkan
berpenghasilan rendah tidak mengalami kepada pemerintah melalui DPP REI
kenaikan secara signifikan. Di sisi lain, untuk menaikan harga jual maksimal
berdasarkan hitungan biaya produksi, marjin RSH dari Rp49 juta menjadi 60,5 juta
keuntungan pengembangan makin per unit.
berkurang jika menjual RSH dengan harga (B) Dampak kenaikan harga minyak
yang berlaku saat ini. dunia yang saat ini sudah mencapai
Sementara itu, jika harga jual RSH 110 dolar AS per barel secara
naik hingga Rp 60,5 juta penjualan dipastikan langsung maupun tidak langsung
akan anjlok. Menurut para pengamat bahwa menyebabkan harga material
kalaupun harga jual RSH akan dinaikan, tidak bangunan naik 20-30 persen.
sampai sebesar itu, karena harga jual RSH (C) Sudah dua tahun harga jual RSH
yang realistis saat ini sekitar Rp 53 juta per tidak naik, sedangkan harga-harga
unit. Kenaikan harga minyak dunia, tidak bisa bahan bangunan saat ini sudah naik
dijadikan sebagai alas utama untuk terutama besi.
68 Sony Sugema College
Sony Sugema College
(D) Berdasarkan hitungan biaya 4. Penulisan kata yang dicetak miring pada
produksi, marjin keuntungan kalimat di bawah ini benar, KECUALI
pengembangan makin berkurang (A) Program prauniversitas itu
jika menjual RSH dengan harga yang diselenggarakan dalam rangka
berlaku saat ini. mempersiapkan lulusan SMA
(E) Jika harga jual RSH naik hingga memasuki universitas.
Rp60,5 juta, penjualan dipastikan (B) Semua agama yang ada di Indonesia
akan anjlok. mendasarkan diri pada Ketuhanan
Yang Maha Esa
3. Penulisan yang benar menurut EYD (C) Kegiatan nonformal sangat
adalah membantu dalam meningkatkan
(A) Pegawai Subbagian Kerja Sama Luar ketrampilan anak.
Negeri yang bernama Sutarjo itu (D) Penyusunan kekuatan negara
pernah bertugas di Kedutaan besar adikuasa di wilayah itu berlangsung
Inggris ketika masih berpangkat diam-diam.
letnan dua sehingga kadang-kadang (E) Selama berada di luar negeri, ia di
gayanya keinggris-inggrisan. tunjuk sebagai menteri ad interim.
(B) Pegawai Subbagian Kerjasama Luar
Negeri yang bernama Sutarjo itu 5. Untuk memberantas hama dan jamur
pernah bertugas di Kedutaan besar pada tanaman kakao, para petani
Inggris ketika masih berpangkat biasanya menggunakan bahan kimia.
letnan dua sehingga kadang-kadang Insektisida yang biasa digunakan adalah
gayanya ke-Inggris-Inggrisan. Baytroid 50 EC, Lannate 25 WP,
(C) Pegawai Subbagian Kerja Sama Luar Sumithion 50 EC, Leboycid 50 EC dan
Negeri yang bernama Sutarjo itu Orthene 75 SP. ... .fungisidanya, biasa
pernah bertugas di Kedutaan besar digunakan Kocide 77 WP. ... zat-zat itu
Inggris ketika masih berpangkat terutama menimbulkan persoalan . ...
Letnan Dua sehingga kadang-kadang dapat merusak unsur hara tanah, pohon
gayanya keinggris-inggrisan. kakao menjadi kerdil dan tidak sehat.
(D) Pegawai Subbagian Kerja sama luar Kata yang tepat untuk mengisi ke tiga
negeri yang bernama Sutarjo itu bagian yang kosong dalam teks di atas
pernah bertugas di Kedutaan besar berturut-turut adalah
Inggris ketika masih berpangkat (A) dan, tetapi, bahkan
Letnan Dua sehingga kadang-kadang (B) dan, akan tetapi, bahkan
gayanya keinggris-inggrisan. (C) dan, tetepi, selain
(E) Pegawai Sub-Bagian Kerja Sama Luar (D) untuk, akan tetapi, selain.
Negeri yang bernama Sutarjo itu (E) untuk, akan tetapi, selain itu.
pernah bertugas di Kedutaan besar
Inggris ketika masih berpangkat
Letnan Dua sehingga kadang-kadang
gayanya keinggris-inggrisan.

Sony Sugema College 69


6. Tidak ada orang yang suka dipaksa (D) Menurut polisi mengatakan bahwa
kawin…di India anak balita juga dipaksa angka kejahatan menunjukkan
kawin…sudah dilarang sejak tahun 1930, peningkatan drastis.
... masih ada saja pihak-pihak yang (E) Penanganan kasus di pengadilan
menghidupkan tradisi kawin paksa. sebaiknya dilakukan secara
Kata yang tepat untuk mengisi bagian transparan.
kosong dalam bacaan di atas berturut-
turut adalah 9. Kalimat yang menggunakan ragam baku
(A) walaupun, bahkan, namun adalah
(B) sekalipun, namun, meskipun (A) Memberikan informasi secara benar
(C) bahkan, tetapi,namun merupakan kewajiban setiap
(D) biarpun, namun, bahkan. pejabat.
(E) Sungguhpun, bahkan, tetapi (B) Pemerintah telah mengumumkan
kenaikan harga BBM, TDL, dan
7. Makna pasif yang ditimbulkan dari telepon tetapi akhirnya diubah
pemakaian awalan ter-dapat ditemukan karena diprotes dan kemudian
dalam kalimat berikut diturunkan kembali.
(A) Kesempatan menjadi pegawai negeri (C) Mendidik dan mengarahkan anak
baru sekarang telah tertutup adalah merupakan tugas dan
baginya. kewajiban orang tua.
(B) Jika terus naik, ongkos perjalanan (D) Menurut para wisatawan asing
rumah-tempat kerja bapak tidak mengatakan bahwa Indonesia
akan terbayar lagi. merupakan salah satu tujuan liburan
(C) Karena lapar, malam itu ia tertidur favorit mereka.
di kursi tunggu rumah sakit. (E) Pemain sinetron kadang-kadang
(D) Dengan terbatuk-batuk, mereka diidentikkan dengan tokoh yang
mengemasi dagangannya di depan diperankannya sehingga ada
pasar . sutradara yang dilempari batu di
(E) Bencana tsunami awal tahun ini jalan.
adalah yang terberat menimpa
Indonesia. 10. Pergeseran lempeng tektoneik
menyebabkan daratan terpencar hingga
8. Kalimat yang tidak menggunakan ragam posisi benua seperti saat ini.
baku adalah Pola kalimat tersebut tidak sama dengan
(A) Pemilu 2004 ternyata berjalan pola kalimat
demokratis, aman, dan lancar. (A) Merokok dapat mengakibatkan
(B) Pemilu multipartai telah dua kali di berbagai penyakit dari sesak napas
gelar pemerintah, tahun 1955 dan sampai kanker.
2004. (B) Peristiwa itu membuat mereka
(C) Pengurus partai politik tidak boleh terpisah untuk selama-lamanya
berstatus pegawai negeri. (C) Proses pembakaran itu
menghasilkan sejumlah gas yang
berbahaya bagi kesehatan.

70 Sony Sugema College


Sony Sugema College
(D) Kasus itu memaksa pemerintah 12. Deretan kata yang berbentuk kalimat
pusat untuk turun tangan dan terdapat pada
mengambil alih penanganannya. (A) Pengaruh tayangan kekerasan
(E) Ilmu ekonomi berupaya untuk terhadap perilaku manusia,
merumuskan dan memilih asumsi- khususnya remaja
asumsi yang memungkinkan mereka (B) Perjuangan melawan penindasan
memberikan jawaban-jawaban yang dan ketidakadilan yang harus
tepat sesuai dengan kebutuhan. dilakukan oleh semua orang.
(C) Menghilangnya BBM menjelang
11. Ayam yang dibalut dengan roti dan diberlakukannya tarif baru oleh
kemudian digoreng ini menjadi pemerintah
kegemaran anak-anak dijadikan lauk (D) Tingginya angka kecelakaan di jalan
pendamping nasi. raya merupakan salah satu indikator
Diantara kalimat-kalimat di bawah ini kurangnya tingkat kesadaran
yang sepola dengan kalimat di atas berkendaraan kita.
adalah … (E) Bupati yang akan melakukan
(A) Waktu yang terasa semakin pendek, peninjauan ke berbagai wilayah
ketergesa-gesaan membuat orang kecamatan di wilayahnya itu.
banyak orang cenderung semakin
memilih hal yang serba praktis. 13. Daratan di muka bumi ini yang terdiri
(B) Meski jenis makanan kaleng dari atas benua hingga pulau-pulau kecil,
waktu ke waktu mendorong sejak jutaan tahun lalu terus bergerak
bertambah variasinya, penjualan dinamis.
makanan dalam kaleng itu tidak Informasi utama kalimat tersebut adalah
segencar makanan seperti nugget (A) daratan ini terdiri atas benua dan
misalnya. pulau-pulau kecil.
(C) Bumbu-bumbu instan yang (B) daratan ini yang terdiri sejak jutaan
ditawarkan dan variasinya kian hari tahun lalu, terus bergerak dinamis.
kian bertambah memberikan (C) bumi ini terdiri atas benua hingga
alternatif untuk penyajian yang pulau-pulau kecil.
serba cepat. (D) bumi ini sejak jutaan tahun lalu,
(D) Untuk mengawetkan supaya terus bergerak dinamis.
makanan tahan lama, ada beberapa (E) daratan yang terdiri atas benua
cara yang biasa digunakan, yakni hingga pulau-pulau kecil berada di
sterilisasi, memanggang, atau muka bumi.
mengasapkan.
(E) Para ilmuwan Inggris yang telah 14. Makna asosiasi dapat ditemukan dalam
menemukan senyawa kimia dari kalimat-kalimat berikut, KECUALI
tubuh manusia yang bisa mengusir (A) Mereka sering mengolok-oloknya
nyamuk merencanakan akan karena berasal dari Ragunan.
menjadikannya obat anti hama (B) Mulut gang itu telah di tutup pagar
alami yang tidak berbau. kawat berduri agar para perusuh
tidak dapat masuk ke perkampunan.

Sony Sugema College 71


(C) Beberapa saksi mata mengatakan 17. Kalimat yang berisi fakta adalah
bahwa pada hari-hari tertentu di (A) Harga tiket kereta api selalu
sekitar pohon itu sering terlihat mengalami kenaikan pada saat
putih-putih melintas dengan cepat. liburan
(D) Sejak tahun 1976 warung Pak Kumis (B) Usia peserta seleksi menjadi PNS
memang telah digandrungi orang. tidak dibatasi
(E) Dengan khidmad semua perserta (C) Tuntutan untuk bekerja secara
upacara memberi hormat kepada profesional harus diikuti dengan
sang Merah Putih pemberian penghargaan yang
sepadan.
15. Kalimat-kalimat di bawah ini (D) Areal parkir untuk kendaraan roda
mengandung idiom, KECUALI empat selalu diperluas
(A) Setelah semua kelengkapan (E) Masa jabatan presiden adalah lima
berkasnya terpenuhi, lebih dari 50 tahun.
kasus korupsi siap diajukan ke meja
hijau. 18. Diduga hampir 95 persen residu pestisida
(B) Tiba-tiba juru bicara kepresidenan dari makanan kita datang dari daging,
mengadakan jumpa pers mendadak produk hewani lainnya, dan terutama
dalam upaya membela polisi sambil ikan.Ikan mengandung zat pemicu
mengambinghitamkan para sopir kanker yang berasal dari semprotan
dan para korban. pertanian yang larut dalam air seperti
(C) Meskipun telah membanting tulang DDT dan PCB. Selain itu, ikan juga
bertahun-tahun, tetap saja biaya mengandung logam berat seperti
sekolah anak-anaknya tidak terbayar merkuri, arsen, timbal, dan
karena tingginya harga buku wajib kadmium.jaringan lemak dalam ikan
yang tiap tahun senantiasa berganti. ,juga bahan pangan hewani lainnya,
(D) Sekali lagi si kaca mata membuat mengikat racun-racun tersebut sehingga
kejutan dengan manuver-manuver sulit terkikis.Meskipun ikan sudah dicuci,
politiknya yang tidak terduga. dibekukan, bahkan dimasak. Membatasi
(E) Walaupun menjadi tangan kanan mengonsumsi bahan makanan
beliau, ia tidak banyak tahu soal hewani,karenanya, berarti meminimkan
kekayaan yang dihibahkan ke masuknya zat pencemar ke dalam tubuh.
beberapa yayasan itu. Bacaan di atas akan menjadi beragam
baku apabila
16. Untuk mencegah terjadinya perpecahan, (A) Kata diduga dalam kalimat pertama
semua pihak yang bertikai dalam tubuh diletakkan diantara kata kita dan
partai itu akan melakukan rekonsiliasi. datang.
Kata rekonsiliasi dalam kalimat tersebut (B) Sesudah kata air dalam kalimat
berarti kedua ditambahkan tanda koma.
(A) Perundingan mencapai kesamaan (C) Kata ikan dalam kalimat ketiga
pandangan dihilangkan
(B) Perbuatan memulihkan (D) Kalimat keempat dan kelima
persahabatan pada keadaan semula digabungkan.
(C) Tindakan memperbaiki keadaan. (E) Kata karenanya dalam kalimat
(D) Kesepakatan perdamaian keenam diletakkan pada awal
(E) Pengembalian seperti semula kalimat.
72 Sony Sugema College
Sony Sugema College
19. Vitamin D dikenal sebagai salah satu (B) Pada umumnya kegiatan berbicara
unsur penting bagi kesehatan. Jika lebih banyak dilakukan daripada
kekurangan vitamin ini, Anda berisiko mendengar.
menderita beragam penyakit mulai dari (C) Kepandaian mendengar sangat
keropos tulang, kanker usus, dan penting peranannya dalam
problem kekebalan tubuh. Selain itu, kehidupan manusia.
para ahli juga mengindikasikan bahwa (D) Setiap orang yang ingin sukses
vitamin D berkaitan dengan penyakit hidupnya haruslah bekerja keras.
rematik. Indikasi tersebut terlihat (E) Kegiatan mendengar haruslah
dengan tingginya kasus defisiensi dilakukan sebelum melakukan
vitamin D pada pasien yang dirawat di pekerjaan lainnya.
klinik rematologi.
Inti kutipan di atas adalah .... 21. Waktu kapal sudah di tengah teluk saya
(A) vitamin D sangat penting bagi memandang balik ke arah lembah yang
kesehatan tulang tertidur dalam samar pagi buta. Di
(B) mengkonsumsi vitamin D membuat sebelah kiri jelas menghujam pucuk bukit
tulang menjadi sehat ke langit kelabu biru. Sambil menyalakan
(C) kekurangan vitamin D menimbulkan rokok buat menghalau angin dingin saya
berbagai problem kekebalan tubuh teringat bahwa selama hidup dewasa
(D) kekurangan vitamin D menimbulkan saya, sebenarnya ayah belum pernah
berbagai penyakit bercakap-cakap dengan saya, kecuali
(E) vitamin D berhubungan erat dengan semalamnya ketika ia melepaskan saya
penyakit tulang dengan kata-kata. Lusa kamu akan
berangkat pulang. Pergilah. Aku tahu
20. Dalam lapangan apa pun kita bekerja, kamu banyak kerja.
perbuatan sehari-hari akan lebih banyak Isi atau ide pokok karangan di atas
ditentukan oleh apa yang kita dengar adalah
daripada apa yang kita lihat, kita (A) pemandangan ke arah lembah yang
rasakan, dan sebagainya. Apakah kita tidur
pekerja kasar atau pekerja halus, (B) hujaman pucuk bukit ke langit
masalahnya akan sama saja. Oleh sebab kelabu
itu, sudah pada tempatnyalah “pandai (C) menyalakan rokok untuk mengusir
mendengar” ini mendapat perhatian dingin
yang sepatutnya di sekolah-sekolah. (D) kenangan yang timbul dalam kapal
Kemampuan mendengar yang sudah (E) gambaran perilaku ayah
dibawa anak sejak lahir itu harus dibina
dan dikembangkan dengan sebaik-
baiknya.
Kalimat topik yang tepat untuk
melengkapi paragraf di atas adalah …
(A) Tidak semua orang suka mendengar
dan menghargai pendapat orang
lain.

Sony Sugema College 73


22. DICARI 23. Setelah pemancar televisi swasta
Seorang tenaga kerja pemasaran yang menayangkan sinetron berseri, ibu saya
selalu ingin menambah pengalaman selalu mengikuti tayangan tersebut,
serta penghasilan. demikian pula dengan ibu Nina,
Kirimkan lamaran Anda ke PO BOX 6020 tetangga saya. Di rumah sakit, baik
Jakarta Kompas, 23 September 2008 karyawan maupun pasien tampak sering
Alinea pembuka surat lamaran pekerjaan menyaksikan acara tersebut. Para
yang sesuai dengan isi iklan di atas pedagang pun menyempatkan diri untuk
adalah menontonnya. Dengan demikian, acara
(A) Menurut informasi yang saya sinetron boleh dikatakan digemari oleh
peroleh dari iklan, tentang banyak orang.
penerimaan tenaga pemasaran, Wacana di atas menarik simpulan
maka dengan ini saya … dengan cara
(B) Berdasarkan iklan yang dimuat (A) deduksi
dalam harian Kompas tentang (B) induksi
tenaga pemasaran, maka saya (C) analogi
bermaksud … (D) sebab – akibat
(C) Sesuai dengan isi iklan yang dimuat (E) akibat – sebab
dalam harian Kompas pada tanggal
23 September 2007, tentang tenaga
pemasaran maka dengan ini saya …
(D) Saya memperoleh informasi dari
Koran Kompas bahwa perusahaan
yang Bapak pimpin memerlukan
tenaga pemasaran. Maka dengan ini
saya mengajukan lamaran kepada
Bapak ...
(E) Berkenaan iklan yang dimuat dalam
harian Kompas dengan ini …

74 Sony Sugema College


Sony Sugema College
12. Tugas Mandiri Bahasa Indonesia

Bangsa Indonesia cenderung tidak 2. Makna kalimat pertama pada alinea


memedulikan fakta historis yang berkaitan pertama bacaan di atas menunjukkan
dengan pelaksanaan konstitusi atau suatu fakta ...
penyelenggaraan negara. Padahal Indonesia (A) pengabaian
punya aset pembelajaran penyelenggaraan (B) perumpamaan
pemerintahan sejak dulu. (C) pengingkaran
“Dua kesultanan besar, yakni (D) perlawanan
Kesultanan Cirebon dan Kesultanan Banten, (E) penyerangan
telah memberikan pelajaran berharga
tentang praktik penyenggaraan pemerintah,” 3. Pola kalimat pertama paragraf pertama
kata Ketua Mahkamah Konstitusi, Prof. Dr. teks bacaan di atas sama dengan
Jimly Asshiddiqie pada seminar nasional (A) Sebelum perahu bertolak ke tengah
“Melacak Konstitusi Kesultanan Banten dan laut, Suhardi disibukkan oleh tugas
Jabar” di Hotel Puri Khatulistiwa Jatinangor membenahi semua perlengkapan
Sumedang, Sabtu (5/4/2008). (B) Tugas Suhardi hanya membetulkan
Menurut Jimly, studi hukum tata jaring atau menjaganya jangan
negara adat yang tidak hanya diperlukan sampai tersangkut di dalam air.
dalam kaitan dengan penerapan norma (C) Pekerjaan Suhardi baru akan dimulai
hukum tata negara adat itu sendiri. Namun, jika jaring sudah ditebar.
mempelajari hukum tata negara adat tetap (D) Emisi karbon dioksida dari Amerika
diperlukan untuk mendekatkan dan Serikat naik dua persen dari tahun
menjadikan UUD 1945 sebagai bagian dari sebelumnya.
sejarah perkembangan masyarakat. Hal itu (E) Dia menjadi hilang semangat setelah
berarti konsep dalam UUD 1945 memiliki mengetahui pembalap unggulannya
akar sejarah. “Kita perlu mempelajari kalah.
konstitusi kerajaan atau kesultanan seperti
Kesultanan Cirebon dan Banten. Kesultanan 4. Penulisan kata bercetak miring yang
Cirebon itu mampu berdiri selama hampir benar terdapat pada kalimat
dua abad,” kata Jimly. (A) Dalam proses
(Sumber : Harian Pikiran Rakyat, 7 April 2008) mengindustrialisasikan, diperlukan
tampilnya sikap-sikap inovatif dan
1. yang bukan kalimat pada paragraf sikap wiraswasta.
terakhir teks di atas adalah kalimat (B) Dengan mengawin silangkan
(A) pertama berbagai padi jenis unggul itu di
(B) kedua peroleh jenis padi unggul baru.
(C) ketiga (C) Kultur jaringan sebesar satu
(D) keempat sentimeter kubik berisi kurang lebih
(E) kelima sejuta sel.
Sony Sugema College 75
(D) Siapakah diantara Anda yang 7. Analisisnya tentang sesuatu sangat
bersedia membantu kami ? tajam sehingga ia selalu berhasil
(E) Sekalipun kami belum pernah ke melakukan persiapan penanganan apa
sana. yang akan terjadi.
Istilah yang tepat untuk menamai
5. Deretan kata yang berbentuk kalimat tindakan yang terkandung dalam kalimat
terdapat pada di atas adalah
(A) Banjir, longsor, dan kekeringan yang (A) antisipasi (D) asumsi
digolongkan sebagai kejadian alam (B) apresiasi (E) partisipasi
yang dipicu dan diakibatkan oleh (C) akselerasi
aktivitas manusia
(B) Teknologi yang paling murah dan 8. hasil penelitian yang diadakan oleh
mudah dengan revegetasi, reboisasi, Departemen Tenaga Kerja bersama
penghijauan atau sejenisnya. Departemen Pendidikan dan
(C) Gerakan nasional rehabilitasi lahan kebudayaan, diperoleh data bahwa
kritis dan penghijauan hutan yang tamatan SMA yang berminat untuk
telah lama dilakukan, namun sering melanjutkan pendidikan ke perguruan
gagal. tinggi berjumlah 90 persen, 10 persen
(D) Kesalahan dalam implementasi bermaksud melanjutkan pendidikan ke
kebijakan tata ruang, lemahnya tingkat akademi
pengawasan dan penegakan hukum Teks di atas akan menjadi baku apabila
menjadi awal sembrawutnya (A) kata bersama diganti dengan kata
penanganan masalah hutan dan atau
lingkungan. (B) menambah kata berdasarkan diawal
(E) Para pejabat yang sangat terampil kalimat
dalam membuat aturan dan tidak (C) kata bahwa diganti dengan tanda
mahir dalam melaksanakan aturan baca koma (,)
tersebut secara konsisten. (D) kata perguruan tinggi ditulis dengan
huruf kapital
6. Kata berimbuhan yang bercetak miring (E) kata persen diganti dengan
berikut digunakan secara tepat dalam prosentase
konteks kalimat
(A) Setiap mandor di perusahaan ini 9. Sikap negara-negara Barat yang
membawahkan sepuluh orang kuli. melarang warganya bepergian ke
(B) Pemimpin itu menugaskan stafnya ASEAN, terutama Indonesia, dikritik oleh
untuk mengikuti rapat. para pemimpin ASEAN dengan
(C) Semoga Anda mendapatkan rahmat menyebutkan itu sebagai hal yang tidak
dari Tuhan Yang Maha Esa. perlu.
(D) Dia berusaha keras mencarikan Inti kalimat di atas adalah
pekerjaan untuk temannya. (A) Negara Barat melarang warganya
(E) Kepala sekolah menanyakan guru BP pergi ke ASEAN.
tentang anak-anak yang berkasus. (B) Negara Barat tidak perlu melarang
warganya pergi ke ASEAN.
(C) Sikap negara Barat dikritik pemimpin
ASEAN.
76 Sony Sugema College
Sony Sugema College
(D) Sikap negara Barat merupakan hal 12. Meledaknya bom di Bali berimbas pada
yang tidak perlu. lesunya aktivitas ekonomi. Karena itu,
(E) Negara Barat melarang warganya. perlu dilakukan revisi target pada
anggaran 2003 mendatang. Namun,
10. 1) Sebagian nasabah masih saja resah. putusan tentang adjusment tersebut
2) Pemerintah memberikan jaminan bergantung pada kesempatan
bahwa uang nasabah tetap aman. pemerintahan.
Penghubung yang paling tepat untuk Kata-kata yang tepat untuk
menggabungkan kedua kalimat di atas menggantikan kata-kata yang bercetak
adalah tebal dalam bacaan di atas adalah
(A) jika (D) sementara itu (A) berdampak, perbaikan, penyesuaian
(B) agar (E) sebaliknya (B) berakibat, pembenahan,
(C) walaupun penyesuaian
(C) berpengaruh, perbaikan, penetapan
11. Penerapan EYD yang salah terdapat (D) berdampak, pembenahan,
dalam kalimat penetapan
(A) Banyaknya perguruan tinggi, baik (E) berakibat, peninjauan, penyesuaian
negeri maupun swasta, yang
13. Dengan perubahan zaman, telah
membuka program pendidikan
menuntut para pendidik untuk mencari
diploma, mulai D1 hingga D3, sangat
metode-metode mengajar yang baru.
mempengaruhi keberlangsungan
Kalimat di atas tidak baku dan dapat
Lembaga Pendidikan Kejuruan.
dijadikan kalimat baku dengan
(B) Kedatangan Penjajah Belanda yang
(A) mengubah menuntut menjadi
ditandai dengan mendaratnya
dituntut
Cornelis de Houtman pada tahun
(B) meletakkan para pendidik pada awal
1596, membuat kehidupan bangsa
kalimat
ini senantiasa dalam belenggu
(C) menghilangkan kata dengan
kemiskinan, kebodohan dan
(D) menghilangkan kata telah
keterbelakangan.
(E) meletakkan dengan perubahan
(C) Secara akademis, perguruan tinggi
zaman pada akhir kalimat
akan mencetak para sarjana,
intelektual, dan calon-calon 14. Kalimat yang mengandung kata
pemimpin bangsa dan tokoh berhipernim adalah
nasional lainnya. (A) Para penyair, novelis, dan
(D) Padahal, idealnya, seorang dramawan telah hadir di ruangan
akademisi senantiasa itu.
menyampaikan argumen dengan (B) Adik tersedu-sedu ketika
objektif dan jujur. mengetahui bonekanya rusak.
(E) Kondisi sosial, politik, ekonomi, dan (C) Dengan berjingkat-jingkat dia masuk
budaya yang tidak membebaskan ke halaman rumah besar itu.
ketidakadilan dan peminggiran hak- (D) Ketika ia melirik pekerjaan
hak orang miskin adalah beberapa temannya, ibu guru menegur
agenda penting yang harus segera dengan keras.
diselesaikan oleh para mahasiswa (E) Toko kami menjual berbagai alat
tulis keperluan kantor dan sekolah
Sony Sugema College 77
15. 1) Manusia selalu ingin berkomunikasi 17. Kusta dikenal sebagai penipu ulung.
dengan sesamanya. Gejalanya kerap meniru-niru penyakit
2) Untuk maksud tertentu hanya kulit lain sehingga agak sulit
bahasalah yang mampu menjadi membedakannya. Bedanya adalah
perantaranya. bahwa kulit yang mengidap kusta akan
3) Di pihak lain manusia mati rasa. Penyakit ini sering diderita
berkecenderungan suka bercerita oleh masyarakat yang ekonominya
tentang sesuatu. rendah.
4) Kemampuan berbahasa dengan Pernyataan di atas mengungkapkan
demikian merupakan kebutuhan (A) kusta sama dengan penyakit lain.
mutlak bagi kehidupan manusia (B) Antara kusta dan penyakit kulit tidak
dalam masyarakat. dapat dibedakan.
5) Kodrat manusia selalu ingin (C) Penyakit kusta dapat juga diderita
mengetahui sesuatu yang belum orang kaya.
diketahuinya. (D) Penderita kusta rentan pada para
Pernyataan-pernyataan tersebut dapat orang miskin.
disusun menjadi sebuah alinea yang (E) Orang yang tingkat ekonominya
padu dengan susunan tinggi kebal terhadap kusta.
(A) 5 – 1 – 2 – 4 – 3
(B) 5 – 4 – 2 – 1 – 3 18. Pernyataan berikut ini yang termasuk
(C) 5 – 3 – 1 – 2 – 4 ragam bahasa dalam surat keputusan
(D) 5 – 2 – 1 – 4 – 3 resmi adalah
(E) 5 – 3 – 2 – 4 - 1 (A) Direktur Bank Buana Cipta
menetapkan nama-nama berikut ini
16. Setelah pelaku pasar menghitung sebagai panitia lelang untuk tahun
dampak tragedi Bali dan pengaruhnya 2007.
terhadap pasar, sejumlah analis (B) Berkenaan dengan hasil rapat ketua
menyatakan kondisi pasar modal saat ini organisasi mahaiswa, kami
sudah mulai normal dan pelaku pasar sampaikan putusan hasil rapat
tampak rasional. sebagai berikut.
Pelaku pasar tampak rasional dalam (C) Yang bertanda tangan di bawah ini
kutipan di atas berarti bahwa pelaku kedua belah pihak dan sepakat
pasar menetapkan hal-hal sebagai berikut.
(A) berani menghadapi resiko (D) Berdasarkan Surat Keputusan
berdagang Dewan nomor 15/UP/J36.1.1/2007,
(B) adalah orang yang cerdas dalam setiap kegiatan mahasiswa harus
berdagang diketahui oleh dosen kokurikuler.
(C) suka melakukan spekulasi dalam (E) Ketua panitia lomba mnerangkan
usaha bahwa nama-nama mahasiswa
(D) telah menghitung keuntungan yang berikut ini telah melaksanakan tugas
diperoleh sebagai juri.
(E) telah mempertimbangkan secara
matang

78 Sony Sugema College


Sony Sugema College
19. Semua masalah penting telah dibahas 21. ... hasil berbagai studi tentang
dalam majelis itu. Beberapa masalah transmigrasi dapat diketahui ...
yang dikemukakan Pak Zakaria tidak transmigrasi swakarsa murni lebih
dibahas dalam majelis itu. berhasil. ... terjadi karena para
Simpulan dari kedua pernyataan itu transmigran swakarsa murni mempunyai
adalah tekad dan kemauan yang besar ...
(A) Semua masalah yang dikemukakan memperbaiki hidupnya.
Pak Zakaria tidak penting. Kata yang tepat untuk mengisi bagian
(B) Beberapa masalah yang penting yang rumpang dalam kalimat di atas
dikemukakan oleh Pak Zakaria, adalah
tetapi tidak dibahas. (A) setelah, maka, ini, agar
(C) Semua masalah yang dikemukakan (B) dari, bahwa, hal ini, untuk
dalam majelis tidak dikemukakan (C) agar, jadi, peristiwa ini, untuk
oleh Pak Zakaria. (D) dari, maka, jadi, agar
(D) Beberapa masalah yang (E) ketika, bahwa, hal ini, karena
dikemukakan Pak Zakaria tidak
penting. 22. Kalimat yang menggunakan kata
(E) Majelis itu tidak membahas semua berimbuhan peN-an yang mengandung
masalah yang dikemukakan oleh Pak makna proses adalah
Zakaria. (A) Peradaban manusia umumnya lahir
dari sebuah permukiman di tepi atau
20. Musibah Gunung Merapi merupakan sepanjang sungai.
contoh konkret betapa masih banyaknya (B) Pembukuan perusahaan itu
kekurangan manusia. Dalam peristiwa diperiksa oleh akuntan swasta.
itu Direktorat Vulkanologi secara jelas (C) Ucapannya tidak dapat dijadikan
bisa memantau peningkatan kegiatan pedoman karena pendiriannya
seismiknya sejak awal November selalu berubah.
sehingga jauh-jauh hari lembaga itu (D) Penempatan guru di daerah
memberitahu aparat terkait. Namun, terpencil harus diimbangi dengan
mereka tidak bisa menentukan kapan fasilitas yang cukup.
Merapi meletus. (E) Ia mendapat penghargaan karena
Pikiran utama alinea di atas adalah prestasinya yang gemilang.
(A) manusia mempunyai banyak
kekurangan
(B) musibah Merapi memberi contoh
(C) Direktorat Vulkanologi memantau
kegiatan seismik
(D) Direktorat Vulkanologi memberitahu
aparat terkait
(E) mereka tidak tahu kapan Merapi
meletus

Sony Sugema College 79


23. Mengemukakan pendapat atau 25. Cina tercatat sebagai negara yang
keterangan dalam panel tidak sama menyumbang polusi terbesar di dunia
dengan berpidato. Pembicara tidak menggantikan posisi AS sejak tahun
berdiri di tempat tertentu, tetapi tetap di 2006. Berdasarkan estimasi BP PLC,
tempat duduknya. Kadang-kadang si salah satu perusahaan minyak raksasa
pembicra merujukkan pembicaraannya dunia, emisi karbon dioksida yang
kepada peserta lain, adakalanya dihasilkan aktivitas industri di Cina
pemimpin diskusi dan kadang-kadang mencapai 2 miliar ton selama tahun
langsung kepada pendengar. 2007 atau naik 7,5 persen dari tahun
Paragraf di atas merupakan paragraf sebelumnya. Sementara, emisi karbon
(A) campuran dioksida dari AS adalah 1,75 miliar ton
(B) deduktif atau naik 2 persen dari tahun
(C) naratif sebelumnya. Laju pencemaran
(D) induktif diperkirakan akan semakin tinggi
(E) tanpa kalimat pokok terutama dari negara-negara
berkembang yang tengah mengalami
24. Keterampilan membaca kamus pertumbuhan industri sangat tinggi,
merupakan keterampilan yang penting seperti Cina. India kini juga menggeser
dimiliki. Keterampilan itu akan sangat Rusia dari posisi ketiga.
membantu Anda dalam mencari dan
menemukan arti kata secara cepat. Jika Jika didasarkan pada kutipan di atas,
hal itu tidak dimiliki, bisa jadi Anda pernyataan berikut benar, KECUALI
menghabiskan waktu setengah jam atau (A) Sejak tahun 2006, emisi karbon
lebih hanya untuk mencari arti sebuah dioksida dari Amerika Serikat lebih
kata. rendah daripada Cina.
Kalimat berikut yang tepat mengakhiri (B) Negara-negara berkembang
paragraf tersebut adalah … merupakan penumbang polusi
(A) Oleh karena itu, beli dan milikilah karbon dioksida tertinggi di dunia.
kamus agar mudah menemukan arti (C) Laju pencemaran karbon dioksida di
kata. negara-negara berkembang
(B) Dengan demikian, keberadaan mengalami kenaikan.
kamus di perpustakaan sekolah (D) India menduduki posisi ketiga
sangat penting. sebagai negara yang menyumbang
(C) Jadi, Anda perlu mengoleksi polusi terbesar di dunia.
berbagai kamus agar mudah (E) Emisi karbon dioksida dari Amerika
mencari arti kata. Serikat naik dua persen dari tahun
(D) Dengan demikian, milikilah kamus sebelumnya.
dan biasakan membaca kamus
dengan cermat.
(E) Oleh sebab itu, biasakanlah
membaca kamus secara tepat dan
cepat.

80 Sony Sugema College


Sony Sugema College
13. Tugas Mandiri Bahasa Indonesia

Selama ini, kasus-kasus pelanggaran mutu guru atau pengadaan sarana


hak asasi manusia selalu dikaitkan dengan laboratorium dan kegiatan penelitian.
hukum. Tidak dipersoalkan bahwa tidak
memberikan pendidikan dan kesehatan yang 1. Judul yang tepat untuk bacaan di atas
baik pun adalah pelanggaran hak asasi adalah
manusia. Padahal, dalam Deklarasi Hak Asasi (A) Sektor Pendidikan dan Kesehatan
Manusia PBB, tercantum jelas hak warga sebagai Hak Asasi Manusia.
negara untuk memperoleh pendidikan dan (B) Ketidakpedulian Pemerintah dalam
pelayanan kesehatan. Dalam UUD 1945 Bidang Pendidikan dan Kesehatan.
tercantum pasal tentang hak memperoleh (C) Komitmen Pemerintah tentang
pendidikan. Sektor Pendidikan dan Kesehatan.
Kurangnya menempatkan sektor (D) Perlunya meningkatkan Anggaran
pendidikan dan kesehatan sebagai hak asasi untuk Pendidikan dan Kesehatan.
manusia, ditambah lagi dengan sikap (E) Sektor Pendidikan dan Kesehatan
menempatkan kedua sektor itu sebagai sebagai Suatu Investasi.
sektor konsumtif sungguh amat disayangkan.
Dengan pemikiran ini, sampai kapan pun 2. Pernyataan yang tersurat di dalam
keduanya tidak pernah dilihat sebagai bacaan di atas adalah sebagai berikut
investasi. Keduanya hanya menjadi (A) Alokasi anggaran untuk sektor
komitmen politis, tetapi belum sempat pendidikan dan kesehatan sudah
tecermin dalam alokasi anggaran. Padahal, memada.i
belajar dari negara-negara lain yang (B) Sektor pendidikan lebih penting
sekarang maju, sejak mereka belum menjadi daripada sektor kesehatan.
negara kaya, mereka sudah menempatkan (C) Deklarasi Hak Asasi Manusia PBB
alokasi anggaran cukup untuk pendidikan tidak mencantumkan hak untuk
dan kesehatan. Bahkan, untuk sejumlah memperoleh pendidikan.
negara di ASEAN pun, kedua sektor ini cukup (D) Selama ini, pembangunan fisik
memperoleh perhatian. sektor pendidikan lebih diutamakan
Sejalan dengan otonomi daerah daripada peningkatan mutu guru.
yang akan dimulai pertengahan tahun 2001, (E) Sejumlah negara ASEAN merupakan
perubahan pemikiran ini merupakan negara-negara maj.u
keharusan. Alokasi anggaran yang lebih
besar harus menjadi kenyataan. Kedua sektor
tersebut adalah hak asasi manusia sehingga
sampai ditempatkan sebagai sektor
konsumtif. Sementara itu, alokasi dananya
pun jangan dipakai untuk pembangunan fisik,
yang lebih mendesak adalah peningkatan
Sony Sugema College 81
3. Semua pernyataan mengenai anggaran 5. Pemakaian konfiks yang benar pada
sektor pendidikan dan kesehatan berikut gabungan kata terdapat pada
benar, KECUALI (A) pertanggungan jawab
(A) Pendidikan dan Kesehatan masih (B) keikutsertaan
dianggap sebagai sektor konsumtif (C) pengalihan bahasa
(B) Pendidikan dan Kesehatan masih (D) pendayaan guna
dianggap sebagai investasi masa (E) dibebaskan tugas
depan.
(C) Di negara-negara maju pendidikan 6. Pemakaian tanda baca yang betul
dan kesehatan sangat diperhatikan. terdapat pada kalimat
(D) Alokasi pemanfaatannya harus (A) Celana “jengki” sudah tidak populer
sesuai dengan kebutuhan lagi di Indonesia.
masyarakat. (B) Rate of Inflation, ‘laju inflasi’, di
(E) Peningkatan jumlahnya masih Indonesia memang agak tinggi pada
menjadi komitmen politis. bulan Oktober 2005.
(C) Surat yang dikirimkan itu No:
4. Dalam mencapai tujuan terbentuknya 124/PP/Pes/I/2007.
karakter kota, desain pencahayaan (D) Ali bertanya: “Di mana kaubeli buku
harus dipertimbangkan secara ini?”
komprehensif dan mengacu sebuah (E) Ia bertempat tinggal di Jalan
konsep makro yang jelas. Desain Diponegoro IV-5.
pencahayaan kota harus dapat terbaca 7. Pada waktu otonomi daerah sudah
sebagai pola besar yang jelas dalam berjalan, perubahan pemikiran yang
lingkungan perkotaan, yaitu elemen mengalokasikan anggaran yang cukup
mana yang merupakan titik krimaksnya. untuk pendidikan dan kesehatan harus
Tentu saja, setiap elemen perancangan sudah dilaksanakan.
harus berangkat dari kearifan budaya Pernyataan ini terdapat pada paragraf
lokal setiap kota. Tanpa itu semua, (A) pertama
hanya ada satu identitas kota seluruh (B) kedua
dunia yang mengacu satu kultur. (C) ketiga
Pernyataan berikut ini berhubungan (D) pertama dan kedua
dengan paragraf tersebut, KECUALI (E) kedua dan ketiga
(A) Konsep makro perlu dijadikan acuan
desain pencahayaan kota. 8. Manajer perusahaan multiproduk itu
(B) Sifat komperhensif perlu berasal dari kalangan elit. Ia dalam
diperhatikan dalam desain manajemennya selalu menggunakan
pencahayaan kota. sistem kontrak.
(C) Kearifan budaya lokal dijadikan titik Dalam pernyataan di atas, terdapat
tolak desain pencahayaan kota. kesalahan penulisan kata serapan, yakni
(D) Ciri khas suatu kota dapat (A) manajer seharusnya manager
diperhatikan dari desain (B) multiproduk seharusnya
pencahayaannya. multiproduck
(E) Desain pencahayaan kota yang satu (C) elit seharusnya elite
dengan yang lain mempunyai pola (D) manajemen seharusnya managemen
yang sama. (E) sistem seharusnya sistim
82 Sony Sugema College
Sony Sugema College
9. Misi damai di Aceh kembali memasuki (C) Pendekatan metakognisi dipilih
babak baru setelah sidang Paripurna sebagai alternatif pemecahan
DPR menyetujui Rancangan Undang- dampak krisis.
Undang Pemerintah Aceh. Namun, (D) Pendekatan metakognisi melibatkan
persetujuan tersebut masih harus aspek kognitif dan aspek afektif
menghadapi banyak tantangan sebelum (E) Pendekatan metakognisi
damai betul-betul kukuh hadir di Aceh. mengintegrasikan aspek afektif dan
Sebuah langkah revolusioner pun telah aspek kognitif
diambil pemerintah dan DPR.
Ide pokok paragraf tersebut adalah 11. Kata ulang yang mengandung makna
(A) DPR menyetujui Rancangan UU PA. menyangatkan terdapat pada kalimat
(B) Misi damai Aceh memasuki era baru (A) Ayah bekerja segiat-giatnya untuk
kembali. mencukupi kebutuhan keluarga.
(C) Persetujuan DPR Pemerintah Aceh (B) Ibu Hasan mondar-mandir mencari
menghadapi tantangan. anaknya yang terlambat pulang dari
(D) Rancangan Undang-Undang sekolah.
sebelum damai betul-betul kukuh di (C) Mereka menggeleng-gelengkan
Aceh. kepadanya karena jengkel sekali.
(E) RUUPA disetujui. (D) Ia berpukul-pukulan dengan si Dul.
(E) Para undangan bersalam-salaman,
10. Pendekatan yang digunakan dalam kemudian pulang.
pelatihan kewirausahaan ini adalah
pendekatan metakognisi. Pendekatan ini 12. Kata kerja berkonfiks ke-an yang
dipilih karena pelatihan kewirausahaan berkaitan dengan kata kerja pasif
di negara kita saat ini merupakan suatu berafiks di-i, terlihat dalam kalimat
alternatif pemecahan dari berbagai berikut, KECUALI pada.
dampak krisis multidimensional yang (A) Kasihan sekali anak itu, dia gegar
melanda bangsa kita. Dengan demikian, otak karena kejatuhan kelapa.
pelatihan kewirausahaan berkaitan (B) Rumah pak Sinaga semalam
dengan orang-orang kurang beruntung kedatangan tamu yang tidak
dan menjadi korban krisis. Karena itu, diundang.
pelatihan kewirausahaan melibatkan (C) Pintu rumahnya kedapatan sudah
aspek afektif (sikap) dan aspek kognitif terbuka
(pengetahuan dan keterampilan) secara (D) Puncak merapi sudah tidak kelihatan
integral. Bahkan, dari beberapa hasil lagi
penelitian, tampak bahwa keberhasilan (E) Mobilnya kehabisan bensin dalam
pelatihan ditentukan oleh keberhasilan perjalanan pulang.
mengubah sikap peserta pelatihan
dengan berbagai motivasi 13. Kata yang menjadi superordinat dari
Simpulan berikut sesuai dengan paragraf kredit, deposito, tabanas adalah
tersebut, KECUALI (A) perekonomian
(A) Pendekatan metakognisi bertujuan (B) perdagangan
mengubah sikap peserta pelatihan. (C) perbankan
(B) Pendekatan metakognisi digunakan (D) pembukuan
dalam pelatihan kewirausahaan. (E) peminjaman
Sony Sugema College 83
14. Pola pembentukan kata jaksa agung (C) untuk memberikan pelayanan bagi
sama dengan pola pembentukan kata di para wisatawan yang umumnya
bawah ini, KECUALI keluarga, penduduk setempat
(A) kursi presiden membangun dan menyewakan
(B) rumah mewah puluhan penginapan sederhana
(C) ekonomi lemah (D) penduduk setempat membangun
(D) politik bebas dan menyewakan puluhan
(E) kekuasaan terbatas penginapan sederhana
(E) para wisatawan menyewa
15. Meraih gelar dunia di bidang ilmu penginapan sederhana
pengetahuan merupakan sebuah
prestasi yang luar biasa. 17. Prestasi yang diraih putra-putri terbaik
Pola kalimat di atas sama dengan pola bangsa ini tidak datang begitu saja.
kalimat berikut. Frase yang menyatakan keterangan
(A) Mempertahankan prestasi di bidang subjek dalam kalimat di atas adalah
fisika tingkat dunia lebih sulit (A) prestasi yang diraih
daripada meraih prestasi itu sendiri. (B) putra-putri yang terbaik bangsa ini
(B) Keyakinan TOFI untuk mengalahkan (C) prestasi
Cina di Olimpiade Fisika tahun (D) yang diraih
depan akan terlaksana. (E) yang diraih putra-putri terbaik
(C) Asia yang selama ini merajai bangsa ini
Olimpiade Fisika Internasional,
seperti Taiwan dan Korea harus 18. Dalam kunjungannya ke panti jompo,
angkat topi. diperoleh informasi yang sangat
(D) Mimpi TOFI akan menjadi berharga.
kenyataan. Pola urutan unsur fungsi dalam kalimat
(E) Target Yohannes Surya beberapa di atas terdapat juga dalam kalimat
tahun ini mencurahkan seluruh (A) Ketika mengerjakan tugas-tugas,
perhatiannya pada TOFI. buku-buku paket sangat diperlukan.
(B) Sambil mendengarkan lagu-lagu,
16. Untuk memberikan pelayanan bagi para Sari menyelesaikan pekerjaan
wisatawan yang umumnya adalah rumahnya.
keluarga, penduduk setempat (C) Meskipun sudah ditegur oleh
membangun dan menyewakan puluhan gurunya, tetap saja dia mengganggu
penginapan sederhana yang terbuat dari temannya.
kayu beratap rumbia. (D) Bagi yang suka sastra, tersedia buku-
Inti kalimat di atas adalah _____ buku yang bermutu.
(A) untuk memberikan pelayanan bagi (E) Dalam menyampaikan laporannya,
para wisatawan yang umumnya Murni menggunakan bahasa Inggris.
adalah keluarga
(B) penduduk setempat membangun
dan menyewakan puluhan
penginapan sederhana yang terbuat
dari kayu beratap rumbia

84 Sony Sugema College


Sony Sugema College
19. Pola kalimat Masyarakat tidak perlu (D) orang membeli karcis
khawatir akan persediaan sembilan (E) pertunjukan drama “Surapati”
bahan pokok, sama dengan pola kalimat
(A) Pemerintah tidak akan mengadakan 22. Jika ada korban gempa yang rumahnya
penjadwalan kembali utang luar rusak berat atau roboh, tetapi tidak
negeri. terdata, yang bersangkutan berhak
(B) Kondisi itu tidak akan menambah mengajukan klaim atau pengaduan ke
perkembangan industri ternak. pemerintah kabupaten (pemkab) atau
(C) Hampir semua negara tidak peduli pengaduan ke pemerintah kota
akan pajak hasil ternak. (pemkot). Sekarang, di setiap pemkab
(D) Usaha ternak akan terhambat kalau atau pemkot telah dibuka layanan dan
dikenakan PPN 10%. posko pengaduan selama 24 jam.
(E) Peranan industri makanan ternak Gubernur DIY meminta agar pemkab dan
cukup strategis. pemkot menindaklanjuti setiap aduan
masyarakat yang rumahnya rusak berat
20. Kalimat berikut yang merupakan atau roboh, tetapi belum terdaftar
perluasan kalimat Setiap bangsa sebagai calon penerima bantuan. Hal ini
memiliki bahasa adalah dilakukan sebagai anggapan atas suara
(A) Setiap bangsa yang ada di dunia ini masyarakat yang mengeluh terhadap
memiliki dan menggunakan bahasa kebijakan pemerintah yang dinilai tidak
yang terus berubah sejalan dengan adil dalam membei bantuan.
perubahan zaman. Pesan yang terkandung dalam paragraf
(B) Setiap bangsa yang memiliki bahasa tersebut adalah
akan selalu menggunakan (A) Korban gempa berhak mengajukan
bahasanya itu untuk berkomunikasi pengaduan ke pemkab atau pemkot.
dalam kehidupan bermasyarakat. (B) Telah dibuka layanan pengaduan
(C) Setiap bangsa memiliki bahasa dan korban gempa di setiap
setiap bahasa selalu dimiliki dan pemkab/pemkot selama 24 jam.
dijunjung tinggi oleh masyarakatnya. (C) Setiap pemkab.pemkot perlu
(D) Dalam kehidupan berbangsa dan menindaklanjuti setiap pengaduan
bernegara, setiap bangsa harus masyarakat yang rumahnya rusak
memiliki bahasa sebagai sarana berat, tetapi belum terdaftar
pengembangan kebudayaan. sebagai calon penerima bantuan.
(E) Hampir dapat dipastikan bahwa (D) Gubernur DIY sangat merespon
tidak ada bangsa di dunia ini yang suara masyarakat yang
tidak memiliki bahasa. mengeluhkan kebijakan
pemkab/pemkot yang dinilai tidak
21. Kalimat inti dari kalimat Sekalipun udara adil dalam memberi bantuan.
dingin berhembus, orang tetap (E) Korban gempa ternyata belum
berduyun-duyun membeli karcis banyak yang menerima bantuan
pertunjukan drama “Surapati” adalah sehingga banyak mengeluh dan
(A) udara dingin berujar bahwa pemerintah tidak
(B) udara dingin berhembus adil.
(C) orang berduyun-duyun

Sony Sugema College 85


23. Pada hakikatnya, setiap masyarakat 24. Bintang sekelas Tom Hanks pasti akan
pasti mengalami perubahan. Perubahan berusaha semaksimal mungkin ….
sosial terjadi karena adanya perubahan perannya dalam film yang dibintanginya.
sosial di antara masyarakat itu sendiri, Kata yang tepat untuk mengisi
baik hubungan antara individu maupun kekosongan tersebut adalah
kelompok. Begitu banyaknya perubahan (A) memilih
yang terjadi dalam masyarakat sehingga (B) memerankan
untuk menelaah perubahannya harus (C) menghayati
ditentukan terlebih dahulu perubahan (D) menyelami
yang menjadi landasan untuk (E) menyampaikan
mengetahui perubahan lainnya. Sumber
perubahan itu ada yang datang dari luar 25. Kata bercetak miring yang bermakna
dan ada yang datang dari dalam konotasi terdapat dalam kalimat
masyarakat itu sendiri. (A) Setelah diperiksa secara menyeluruh
Pernyataan berikut yang merupakan diputuskan bahwa kaki tangan
ringkasan dari paragraf tersebut adalah pasien itu harus diamputasi.
(A) Setiap masyarakat mengalami (B) Barang-barang yang akan diekspor
perubahan yang terjadi karena melalui pelabuhan itu habis dimakan
hubungan antar individu dan tikus pelabuhan.
kelompok yang datang dari luar dan (C) Kutu buku itu berbahaya bagi
dalam. kesehatan manusia.
(B) Hakikatnya, masyarakat berubah (D) Kambing hitam yang sudah cukup
karena banyaknya perubahan dalam umur itu dijual dengan harga
masyarakat yang datang dari luar Rp500.000,00
dan dalam masyarakat. (E) Orang di desa itu mencari anjing gila
(C) Banyaknya perubahan yang terjadi yang sering makan ayam
dari luar dan dalam masyarakat peliharaannya.
menyebabkan yang datang dari luar
dan dalam masyarakat.
(D) Penelaahan perubahan sosial hanya
dapat dilakukan karena perubahan
yang datang dari luar dan dalam
masyarakat.
(E) Setiap masyarakat mengalami
perubahan yang penelaahannya
harus ditentukan terlebih dahulu
perubahan yang terjadi.

86 Sony Sugema College


Sony Sugema College
14. Tugas Mandiri Bahasa Indonesia

Orang Indonesia yang gemar di puncak produktivitas. Mereka pada


melancong ke luar negeri mungkin lebih umumnya berupa pasangan muda,
kenal Singapura atau Ertopa daripada daerah pengantin baru, keluarga dengan anak usia
dan budaya negeri sendiri. Penelitian di bawah lima tahun, dan dari kelas ekonomi
Departemen Kebudayaan dan Pariwisata menengah ke atas. Beban finalsial yang
tahun 2004 mengungkapkan bahwa mereka belum begitu berat memungkinkan mereka
yang bepergian ke mancanegara rata-rata dapat menyisihkan biaya berlibur.
baru pernah mengunjungi tiga dari 33
provinsi di Tanah Air. 1. Topik yang paling tepat untuk bacaan di
Selain wisata asing, turis local atas adalah …
sebetulnya dapat menjadi pasar pariwisata. (A) kecenderungan orang berlibur ke
Persoalannya adalah bahwa factor yang luar negeri
menarik bagi mereka untuk berwisata di (B) tujuan orang berlibur ke luar negeri
negeri sendiri sangat lemah. Objek wisata di (C) daya tarik wisata ke mancanegara
luar Bali, misalnya rata-rata kurang terawatt (D) berkunjung ke Negara tetangga
karena keterbatasan dana. Menjangkau (E) pelancong ke mancanegara
lokasinya pun tidak mudah karena sarana
transfortasi terbatas. Berbagai retribusi 2. Berikut ini adalah pernyataan yang tidak
memberatkan pengusaha wisata untuk sesuai dengan isi teks di atas
meningkatkan mutu pelayanan. Dukungan (A) Tempat wisata di Bali lebih terawat
pemerintah daerah pada usaha daripada tempat lainnya.
kepariwisataan pun masih kurang. (B) Bagian terbesar orang Indonesia
Pada tahun 2000, melalui 13 pintu yang berlibur ke luar negeri adalah
keberangkatan, tercacat 2,2 juta orang kelompok usia produktif.
Indonesia yang berkunjung ke negeri asing. (C) Jumlah orang Indonesia yang
Empat tahun kemudian melonjak menjadi 3,9 berlibur ke luar negeri meningkat
juta. Hingga November 2005, sudah 3,7 juta dari tahun ke tahun.
orang melancong ke negara lain. (D) Umumnua orang lebih suka
Menurut Sekjen Asosiasi Biro mengunjungi tempat wisata di
Perjalanan dan Wisata (ASITA), objek wisata dalam negeri sebelum menginjungi
di negeri orang menjanjikan atraksi menarik tempat wisata di luar negeri.
pada saat pergantian tahun. Bulan Juni dan (E) Pada tahun 2004 jumlah orang
Juli, masa libur sekolah, juga waktu favorit Indonesia yang berkunjung ke luar
warga Indonesia untuk berlibur ke luar negeri mengalami kenaikan sebesar
negeri. Kelompok pelancong ke luar negeri 1,7 juta dibanding dengan tahun
terbesar ini adalah penduduk berumur 25 2000.
hingga34 tahun, kelompok yang berada

Sony Sugema College 87


3. Karangan di atas bersifat (C) Hari ini Indonesia akan
(A) naratif memberangkatkan tiga belas
(B) argumentatif mahasiswa untuk mengikuti
(C) deskriptif olimpiade matematika di Perancis.
(D) eskpositoris (D) Menurut penelitian Depbudpar 2005
(E) kritik mengungkapkan orang Indonesia
yang berlibur ke luar negeri rata-rata
4. Pada paragraph kedua terdapat kalimat mengeluarga dana 860 dollar AS
tidak baku, yaitu kalimat dalam setiap kunjungan.
(A) kesatu (D) keempat (E) Karya sastra melukikan hakikat yang
(B) kedua (E) kelima nyata dari objek sebagai titik tolak,
(C) ketiga kemudian dengan perlahan bergerak
kearah imajinasi dan perasaan.
5. Pelayanan kesehatan di berbagai tempat
di tanah air belum berorientasi kepada 7. Setahun berikutnya, dengan merangkul
pasien. Hal ini ditandai oleh maraknya Jaeckel, Lea Rosh menerbitkan sebuah
pemberian obat antibiotic yang tidak inisiatif Berlian dalam Perspektif untuk
sesuai dengan kondisi klinis. Pemakaian menggolkan ide pembuatan memorial
antibiotik yang melebihi kebutuhan bisa untuk para keturunan Yahudi yang
mengakibatkan resistansi dalam tubuh dibunuh secara missal di Eropa.
pasien. Kumanb penyakit dalam tubuh Inti kalimat tersebut meliputi unsure-
menjadi kebal. Oleh karena itu, … unsur
Kalimat tang tepat untuk melengkapi (A) Lea Rosh, meranngkul, Jackel
paragraf di atas adalah (B) Lea Rosh, menggolkan, ide
(A) Perlu ditumbuhkan sikap kritis (C) Lea Rosh, menerbitkan, inisiatif
masyarakat terhadap layanan (D) Setahuan, Lea Rosh, menggolkan,
kesehatan. ide
(B) Pasien harus selalu mematuhi (E) keturunan Yahudi, dibunuh, di Eropa
semua instruksi dari dokter.
(C) Pasien harus pasrah pada layanan
kesehatan yang diterima.
(D) Pasien harus diberi antibiotic di atas
kebutuhan supaya daya tahan tubuh
lebih tinggi.
(E) Setiap penyakit harus diobati
dengan antibiotik.

6. Kalimat yang menggunakan ragam baku


adalah
(A) Dari hasil penelitian membuktikan
bahwa epilepsi bukan penyakit
menular.
(B) Mengenai rancangan undang-
undang itu belum disetujui anggota
DPR.
88 Sony Sugema College
Sony Sugema College
8. Setidaknya ada tiga hal yang dapat 9. Mahasiswa yang idealis yang memiliki
dilakukan oleh sebuah perusahaan untuk visi dan misi serta imajinasi bagus itu
meningkatkan kualitas dan disiplin tahun depan akan mengikuti program
pegawainya. Pertama, perusahaan pertukaran mahasiswa Indonesia –
menyediakan sarana kerja yang Jepang yang merupakan realisasi kerja
memadai. Kedua, dilakukan pemberian sama bidang pendidikan dan pengajaran
contoh oleh atasan. Ketiga, perusahaan Pernyataan yang benar berkaitan dengan
menciptakan suasana kerja yang kalimat di atas adalah … .
menyenangkan. (A) Menduduki fungsi subjek dalam
Empat kalimat di atas dapat diringkas kalimat di atas adalah mahasiswa.
menjadi satu kalimat tanpa mengubah (B) Kalimat di atas dapat diubah
ide dasarnya. Kalimat yang tepat menjadi Program mahasiswa
mencerminkan ide yang dikandung Indonesia- Jepang yang merupakan
empat kalimat di atas adalah … realisasi kerja sama bidang
(A) Kualitas dan disiplin pegawai pendidikan dan pengajaran tahun
perusahaan dapat ditingkatkan depan akan diikuti oleh mahasiswa
dengan menyediakan sarana kerja idialis yang emmiliki visi dsan misi
yang memadai, pemberian contoh serta imajinasi bagus itu.
oleh atasan, dan penciptaan suasana (C) Kalimat di atas sama idenya dengan
kerja yang menyenangkan. kalimat tahun depan program
(B) Kualitas dan disiplin pegawai pertukaran mahasiswa Indonesia –
perusahaan dapat ditingkatkan Jepang akan diikutinya.
dengan tiga hal, yaini disediakan (D) Kalimat di atas tidak dapat diubah
sarana kerja, diberikan contoh yang menjadi kalimat pasif
baik, dan perusahaan menciptakan (E) Dengan arti atau maksud yang sama,
suasana kerja yang menyenangkan. kalimat di atas dapat diubah
(C) Peningkatan kualitas dan disiplin menjadi Mahasiswa itu tahun depan
pegawai perusahaan dapat akan mengikuti program pertukaran
dilakukan dengan menyediakan Mahasiswa Indonesia – Jepang
sarana kerja yang memadai, karma memiliki visi dan misi serta
pemberian contoh dari atasan, dan imajinasi bagus.
diciptakannya suasana kerja yang
10. Penulisan kata serapan yang betul
menyenangkan.
terdapat dalam kalimat
(D) Untuk meningkatkan kualitas dan
(A) Di dunia olahraga yang dipentingkan
disiplin pegawai, perusahaan
adalah sportivitas.
sebaiknya menyediakan sarana,
(B) Sikap yang kaku dalam seorang
memberikan contoh, dan
pemimpin menimbulkan watak
menciptakan kesenangan.
arogan dan otoritair.
(E) Agar kualitas dan disiplin pegawai
(C) Majelis juga mengusulkan agar
perusahaan meningkat, kita harus
kalangan eksekutip dan legislatip
menyediakan sarana, memberikan
peka terhadap aspirasi rakyat.
contoh, dan suasana kerja pun
(D) Mereka juga memperotes kwalitas
tercipta.
beras yang mereka terima.
(E) Rapat itu gagal karena yang hadir
tidak memenuhi korum.
Sony Sugema College 89
11. Kalimat berikut tidak menggunakan 13. sifat Mariamin yang paling berharga
tanda baca yang tepat, KECUALI adalah pengabdian dirinya yang
(A) Telah dikemukakan bahwa, bahan menyeluruh. Salama dua tahun bekerja,
baker merupakan energi yang ia tidak pernah membiarkan pekerjaan
sangat penting dalam kehidupan menumpuk. Justru, ia lebih suka
masyarakat. menambah waktu kerja pada akhir
(B) Meskipun belum sempurna, uraian minggu daripada pekerjaan itu tertunda.
yang disampaikannya sangat Sebagian pekerjaannya mengetik.
menarik. Pekerjaan itu diselesaikan dengan cepat
(C) Investasi yang kita berikan ini, tidak dan efisien. Namun, jika ia harus
seluruhnya berupa uang. menyusun memo atau lainnya, ia kurang
(D) Mutu pelayanan harus selalu mahir. Dalam hal ini hasil pekerjaan dan
ditingkatkan, supaya tidak isinya perlu diperiksa.
mengecewakan pelanggan. Laporan di atas menginformasikan
(E) Kerusakan lingkungan yang parah di tentang
Daratan Tinggi Dieng, boleh jadi (A) keadaan Mariamin di tempat kerja
akan menimpa kawasan lereng (B) di samping kekurangan yang ada,
Gunung Ungaran. Mariamin adalah karyawan yang
baik
12. Penulisan kata yang bercetak miring (C) Mariamin tidak bisa membuat
dalam kalimat berikut yang sesuai memo
dengan EYD adalah (D) Mariamin sebagai karyawan yang
(A) Hasil analisa peneliti itu sangat baik
bermanfaat bagi masyarakat dan (E) kekurangan Mariamin
pengembangan ilmu pengetahuan.
(B) Dalam sistim pendidikan yang baik, 14. Penulisan kata serapan yang benar
proses pendidikan dapat terdapat dalam kalimat
diselenggarakan di dalam maupun di (A) Setiap wanita WNI yang menikah
luar kelas. dengan pria WNA harus siap
(C) Konduite merupakan salah satu menanggung konsekwensi bahwa
syarat pegawai untuk naik pangkat. status kewarganegaraan anaknya
(D) Pendapatnya sangat ekstrim akan mengikuti suaminya.
sehingga para peserta rapat tidak (B) Kita perlu segera mengkonkritkan
menyetujuinya. gagasannya itu.
(E) Tehnik penulisan ilmiah harus dapat (C) Penduduk Pasifik yang kebanyakan
dipahami oleh mahasiswa di tinggal di pulau Karang adalah
perguruan tinggi. kelompok yang paling beresiko.
(D) Kami mengusulkan agar salah satu di
antara kita bersedia mengkoordinir
pelaksanaan kegiatan ini.
(E) Dia berhasil memopulerkan jenis
olahraga itu di negaranya.

90 Sony Sugema College


Sony Sugema College
15. Deretan kata yang mengemukakan ide 17. Kalimat Koran Masuk Desa merupakan
yang utuh adalah langkah yang stertegis guna
(A) Pengaruh perubahan sitem seleksi meningkatkan kecerdasan masyarakat.
mahasiswa baru, khususnya di UGM. Memiliki pola yang sama dengan
(B) Penegakan keadilan yang harus (A) Pemerintah mengajukan beberapa
didukung oleh segenap elemen usul guna membangun desa – desa
bangsa. tertinggal.
(C) Memudanya semangat gotong (B) Pemerintah meningkatkan
royong yang perlu di perhatikan. kesejahteraan masyarakat dengan
(D) Tingginya usia harapan hidup proyek – proyek unggulan
merupakan sesuatu indikator makin (C) Usaha pemerintah berupa
baiknya tingkat kesehatan pemberian subsidi kepada rakyat
masyarakat. kecil harus didukung.
(E) Kerjasama Indonesia Timur Tengah (D) Usaha itu adalah stertegi yang tepat
yang tengah di galakan oleh kedua untuk membangun kesadaran
belah pihak. rakyat.
(E) Penerintah berusaha meningkatkan
16. Kalimat yang mencerminkan penalaran kesejahteraan rakyat dengan
yang baik adalah berbagai upaya.
(A) Karena kampanye yang melewati
ruas jalan itu terjebak macet 18. Pembangunan perkebunan belum
sehingga pertikaian antar simpatisan berhasil meningkatkan peningkatan
tidak dapat dihindarkan. petani kebun pada tingkat yang optimal.
(B) Dia yang tertua dari ketiga Proses pembentukan kata dengan me-
saudaranya yang kini sukses menjadi kan pada kata meningkatkan dalam
pengusaha. kalimat tersebut mempunyai makna
(C) Harga fotokopi dengan mesin Xerox yang sama dengan proses pembentukan
yang mengalami kenaikan drastis kata dengan kalimat
akhir – akhir ini (A) Pembangunan itu dinilai belum
(D) Kontrak jasa bertujuan menguntungkan.
meningkatkan produksi ladang – (B) Metode itu akan dicobaterapkan
ladang minyak di wilayah Nirm- untuk mengembangkan usaha
Karim dengan biaya yang efektif. perkebunan itu.
(E) Mahasiswa yang sedang berunjuk (C) Rancang bangun itu diarahkan
rasa tersebut yang terpaksa kepada sentra produksi dengan
menjadi Pressure force yang harus di menggunakan salah satu instrumen
perhitungkan yang ada.
(D) Untuk menjalankan misinya, peran
kantor cabang merupakan andalan
yang penting.
(E) Sejumlah nasabah telah
memanfaatka dana kredit program
ini.

Sony Sugema College 91


19. Gaya untuk menghaluskan makna (B) Kemajuan iptek juga menyebabkan
terdapat dalam kalimat hidup manusia berpola konsumtif.
(A) Laut biru membentang seluas mata (C) Sampah dari berbagai tempat terus
memandang. bertambah setia hari.
(B) Tubuhnya kurus kering tinggal kulit (D) TPA tidak mampu menampung
membalut tulang saja. sampah.
(C) Semenjak anaknya meninggal, (E) Sampah dapat menggangu
ingatannya agak terganggu. pemandangan dan menimbulkan
(D) Selama berminggu-minggu ia tidak bau
pernah menampakkan bentang
hidungnya. 22. Tanaman hias ini bisa membuat rumah
(E) Kesedihannya ditumpahkan pada menjadi sejuk. Baunya pun harum, tapi
rumput yang bergoyang. tidak menusuk. nyamuk-nyamuk yang
mengisap aromanya menjadi mabuk.
20. Perayaan Cap Go Meh ,misalnya diakui Serangga dan kutu busuk pun sungkan
sebagai hari libur fakultatif. untuk masuk. Yang paling penting,
Hari libur fakultatif dalam kalimat tetanaman ini lebih bersahabat daripada
tersebut maksudnya adalah semua jenis obat antinyamuk.
(A) hari libur yang berlaku untuk Penggunaan kata yang tidak baku dalam
kalangan terbatas teks tersebut adalah
(B) hari libur yang dianjurkan (A) mengisap
(C) hari libur yang bekum diakui (B) antinyamuk
(D) hari libur yang belum resmi (C) tapi
(E) hari libur yang tidak di wajibkan (D) daripada
(E) mabuk
21. Kemajuan iptek tersebut juga
menyebabkan hidup manusia berpola 23. Menjelang bulan Agustus Pembina OSIS
konsumtif. Pola hidup yang demikian SMA telah menerima surat dari kepala
menghasilkan sampah yang sangat sekolah untuk mempersiapkan tiga puluh
banyak. Sampah dari pasar, pabrik, siswa dalam rangka menyambut HUT RI.
industri, rumah tangga, dan sampah dari Lalu Pembina OSIS meminta kepada
tempat – tempat umum lainnya setiap ketua OSIS untuk mengikuti paduan
hari terus bertambah.TPA tidak mampu suara tersebut.
menampung sampah yang sangat Kalimat memo yang sesuai denga
banyak dan menggunung tersebut. Di sisi ilusterasi tersebut adalah
lain sampah yang menggunung tersebut (A) Diharapkan Saudara kumpulkan
jika tidak segera ditangani juga, dapat anggota OSIS sebanyak tiga puluh
menggangu pemandangan, orang untuk paduan suara tanggal17
menimbulkan bau yang tidak sedap, dan Agustus.
menjadi sumber penyakit. (B) Siapkan tiga puluh siswa untuk
Pikiran utama pernyataan di atas adalah mengikuti paduan suara dalam
…. rangka menyambut HUT RI, tanggal
(A) Pola hidup konsumtif menghasilkan 17 Agustus .
sampah yang banyak.

92 Sony Sugema College


Sony Sugema College
(C) Jangan lupa, tanggal 17 Agustus 25. Topik karangan : Pemandangan
siapkan tiga puluh orang untuk Kintamani, Bali
mengikuti paduan suara dalam Kalimat penjelas :
rangka menyambut HUT. (1) Pemandangan bukit dengan pohon
(D) OSIS, harap segera kumpulkan tiga pinus, persawahan, dan sayuran
puluh siswa untuk mengikuti paduan hijau yang menyejukkan dan
suara hari ulang tahun RI tanggal 17 menyegarkan mata.
Agustus . (2) Terlihat perkebunan kopi arabika
(E) Tolong ketua OSIS kumpulkan yang buahnya mulai kemerahan.
segera tiga puluh siswa untuk (3) Akan tetapi, harga kopi murah di
mengikuti paduan suara HUT RI Kintamani, Bali.
tanggal 17 Agustus. (4) Di antar tanaman kopi juga terlihat
pohon jeruk kintamani yang sedang
24. Pada tanggal 10 November 1945 telah berbuah.
dikeluarkan amanat perang pada (5) Rendahnya harga kopi disebabkan
sekuruh rakyat. Dengan demikian, secara kurang baiknya bahan baku,
Republik yang masih muda ini terlibat penjemuran, dan kadar air yang
dalam sebuah perang terbuka dengan tinggi.
negara lain. Seluruh rakyat diminta Kalimat penjelas yang relevan dengan
untuk menyumbangkan tenaga dan topik tersaji adalah….
hartanya untuk mempertahankan (A) (1), (2), dan (3)
Rebublik in (B) (1), (2), dan (4)
Kalimat yang tepat untuk melengkapi (C) (1), (2), dan (5)
paragraf tersebut adalah (D) (2), (3), dan (4)
(A) Berarti, masyarakat menyambut (E) (2), (3), dan (5)
amanat perang tersebut dengan
baik.
(B) Masyarakat secara bersama – sama
ikut perang terbuka.
(C) Dengan sukarela masyarakat
menyumbangkan darahnya.
(D) Seruan itu disambut hangat dengan
persiapan pengungsian.
(E) Walaupun demikian hanya lapisan
masyarakat mampu saja yang
terlibat.

Sony Sugema College 93


15. Tugas Mandiri Bahasa Indonesia

Karena terseret Perang Dunia ke-2 2. Salah satu tokoh dari dua proklamator
dan Perang Pasifik, Indonesia diduduki Bala dalam wacana di atas dengan tokoh
Tentara Jepang dari tahun 1942 sampai reformasi yang akhirnya menjadi
1945. Pada tanggal 9 Maret 1942, Presiden ke-5 Republik Indonesia
Pemerintah Hindia Belanda menyerah tanpa memiliki hubungan ...
syarat kepada Jepang. Seluruh bekas daerah (A) demografis
Hindia Belanda berada di bawah kekuasaan (B) antropologis
Pemerintah Bala Tentara Dai Nipon. (C) genealogis
Gubernur Hindia Belanda terakhir, Tjarda van (D) psikologis
Starkenborgh, ditawan Jepang dan diangkut (E) sosiologis
ke Formusa.
Dengan didahului oleh ledakan bom 3. Feri berpenumpang 1.318 orang milik
atom di Hiroshima dan Nagasaki, Jepang pengusaha Mesir tenggelam saat
menyerah kalah kepada Sekutu pada tanggal berlayar dari Dubah menuju Safaga yang
14 Agustus 1945. Pada tanggal 17 Agustus berjarak 193 km.
1945, atas nama Bangsa Indonesia, Soekarno Ide terpenting yang perlu diperhatikan
dan Hatta memproklamirkan kemerdekaan saat membaca kalimat tersebut adalah
Indonesia. Undang-Undang Dasar Negara (A) Feri berlayar dari Dubah menuju
Republik Indonesia ditetapkan pada tanggal Safaga
18 Agustus 1945. Sementara itu, tanggal 29 (B) Feri berpenumpang 1.318 orang dan
September 1945 tentara Sekutu yang milik pengusaha Mesir.
pertama mendarat di Pulau Jawa dipimpin (C) Dubah berjarak 193 km dari Safaga.
oleh Letnan Jenderal Philip Christison. (D) Feri tenggelam
(E) Feri tenggelam saat berlayar
1. Apabila kalimat ke-2 dan ke-3 alinea ke-1
bacaan di atas dijadikan kalimat 4. Kalimat berikut yang memenuhi syarat
majemuk, kata penghubung yang kepaduan adalah
diperlukan adalah ... (A) Apabila Jaksa Agung ingin
(A) dan (D) maka menggunakan hak hukum yang
(B) kemudian (E) sehingga tersedia, tetap bisa dilakukan.
(C) bahkan (B) Kalau kita tidak memperhatikan
secara cermat, akan sulit
menunjukan salahnya.
(C) Dalam penyusunan skripsi,
mengumpulkan data merukpakan
aktivitas yang harus dilakukan.

94 Sony Sugema College


Sony Sugema College
(D) Untuk mengermbangkan naskah (C) Proses pembentukan kata
pidato yang baik, mempersyaratkan pemberdayaan dalam kalimat
seseorang memiliki wawasan yang perusahaan mengadakan program
luas. pemberdayaan karyawan berbeda
(E) Karena alasannya tidak masuk akal, dengan pola tersebut.
pihak jaksa sebagai penuntut umum (D) Proses pembentukan kata
tidak menerima. pemberantasan dalam kalimat
pemberantasan penyakit menular
5. Kalimat berikut ini termasuk kalimat memerlukan partisipasi aktif warga
baku adalah berbeda dengan pola tersebut.
(A) Pada setiap pernyataan yang (E) Proses pembentukan kata
diucapkannya selalu mengandung pemberhentian dalam kalimat kita
kritikan yang tajam pada turun di pemberhentian berikutnya
pemerintah. sama dengan pola tersebut
(B) Dalam menghadapi konflik sosial
dan krisis ekonomi yang 7. Kalimat berikut ini yang seluruhnya
berkepanjangan ini tidak ada cara ditulis sesuai dengan aturan ejaan yang
lain kecuali berdoa dan berusaha. baku adalah
(C) Jika persoalan ini dirundingkan dan (A) Hampir setiap buku Bahasa
ditanggung bersama, maka akan Indonesia membahas masalah
dapat segera diselesaikan dan terasa ejaan.
lebih ringan. (B) Dalam tulisannya masih banyak
(D) Ketika sedang berada pompa bensin terdapat kesalahan ejaan, walaupun
untuk mengisi BBM, untuk tidak ia sudah tahu aturan ejaan yang
menghidupkan HP. baku.
(E) Setiap orang memiliki kekuasaan (C) Kata – kata asing dalam teks ilmiah,
bawaan entah berkepribadian penulisannya perlu digaris - bawahi
menarik entah berkarisma tinggi, atau dicetak miring
untuk memberikan pengaruh. (D) Kata universitas sebagai nama diri
ditulis dengan huruf awal kapital.
6. Proses pembentukan kata pemberlakuan (E) Setiap siswa perlu memiliki buku
dalam kalimat pemberlakuan undang – Pedoman Ejaan Yang
undang itu di tunda berpola laku> Disempurnakan.
berlaku> memberlakukan>
memberlakuan. Pernyataan benar, 8. Kalimat berikut ini yang seluruhnya
KECUALI ditulis sesuai dengan aturan EYD adalah
(A) Proses pembentukan kata (A) Wawancara dilakukan untuk
pemberontakan dalam kalimat memperoleh data yang berkaitan
pemberontakan terhadap dengan struktur sosial di desa, yang
pemerintah digagalkan berbeda meliputi : Kelas atas, kelas
dengan pola tersebut. menengah, kelas bawah.
(B) Proses pembentukan kata (B) Pengamatan secara mendalam
pemberangkatan dalam kalimat dilakukan untuk memperoleh data
pemberangkataan jemaah haji tentang prilaku kelompok elit
tertunda sama dengan pola tersebut maupun massa.
Sony Sugema College 95
(C) Untuk memperoleh data tentang 11. Kalimat berikut yang seluruh ejaannya
lokasi desa dan kependudukan sesuai dengan EYD adalah
dilakukan pengumpulan data (A) Setiap benda yang dimasukan
melalui dokumen – dokumen yang kedalam zat cair akan mendapatkan
ada di kelurahaan. tekanan keatas.
(D) Keabsahan data dilakukan dengan (B) Wayang bali berukuran lebih
memeriksa kredibilitas, Keteralihan, pendek, dengan tatahan tidak
Kebergantungan dan Kepastian dari terlalu banyak seperti yang biasa
sumber data. dijumpai pada wayang jawa.
(E) Dari kajian peralihan status sosial (C) Rencana PLN untuk menaikan tarif
ekonomi ke kelompok pamong desa, dasar listrik banyak ditentang
dapat dikemukakan intisarinya masyarakat.
sebagai berikut. (D) Sebagai catatan warga di kawasan
pegunungan Yahukimo memang
9. Kalimat berikut ini ditulis sesuai dengan telah terbiasa memasak beras.
aturan EYD, KECUALI (E) Tragisnya, osteoporosis yang sering
(A) Agar airnya tidak meluap disaat disebut silent desease ini muncul
musim hujan, semua sungai yang tanpa adanya keluhan klinis pasien.
dangkal harus segera dikeruk.
(B) Meluapnya Sungai Cibarani 12. Para anggota tidak ... dengan janji –
mengakibatkan jebolnya tembok di janjinya yang disampaikan pada saat
permukiman padat Kelurahan pertemuan teknis. Dalam mengambil
Cipaganti. langkah pun, mereka juga tidak …
(C) Pengerukan sungai tersebut .Keputusan yang diambil bersifat
dilakukan oleh para Camat, Lurah sporadis. Karena itu, banyak peserta
dan masyarakat dengan cara kerja lomba yang meminta … terhadap
bakti. putusan yang diberlakukannya.
(D) Biaya pengobatan dan perbaikan Pilihlah kata yang tepat untuk mengisi
rumah para korban ditanggung oleh kalimat rumpang tersebut adalah.
Bapa Wali Kota Bandung. (A) konsisten – konsekuen – konfirmasi
(E) Beliau berkata, “Mudah – mudahan (B) konsekuen – kompromi – konfirmasi
musibah tersebut tidak terjadi lagi.” (C) konsekuen – konsisten – konfirmasi
(D) konsisten – kompromi – klarifikasi
10. 1. Printer di kantor digunakan oleh (E) konsekuen – konsisten – klarifikasi
banyak orang.
2. Printer itu sekarang sering rusak. 13. Terkait dengan komposisi menteri, jika
Kata sambung yang sesuai untuk dilakukan reshuffle kabinet, faktor
menggabungkan kedua kalimat itu kapabilitas dan integritas menjadi
adalah kepentingan utama.
(A) dan Kata – kata yang tepat untuk
(B) maka menggantikan kata yang bercetak tebal
(C) sehingga dalam kalimat tersebut adalah
(D) supaya (A) pergeseran, kemampuan,
(E) agar kecakapan.
(B) bergantian, kepandaian, diakui.
96 Sony Sugema College
Sony Sugema College
(C) pengubahan, kecakapan, (A) Lagu – lagu Daerah sebagai
kemampuan. Cerminan Budaya Komunitas Sosial
(D) perubahan, kemampuan, kejujuran. Masyarakat Tutur
(E) pergeseran, kepandauan, kejujuran (B) Komunitas Sosial Masyarakat Tutur
dan Budayanya dalam Lagu – lagu
14. Perdebatan kedua belah pihak yang Daerah
memiliki paradigma yang berbeda tidak (C) Ekspresi Budaya Komunitas Sosial
akan dapat mencapai solusi yang saling Masyarakat Tutur dalam Lagu – lagu
memuaskan Daerah
Kata paradigma dalam kalimat tersebut (D) Ekspresi Budaya dalam lagu – lagu
bermakna Daerah dari Komunitas Sosial
(A) kepentingan kelompok Masyarakat Tutur
(B) kerangka pikir (E) Lagu- lagu Daerah sebagai Produk
(C) cara kerja Budaya Komunitas Sosial
(D) sudut pandang Masyarakat Tutur
(E) landasan teori
17. Jika sedang berada di bandara, sesekali
15. Bank besar yang terpengaruh adalah perhatikanlah pengatur lalu – lintas
portofolionya di sektor korporat dengan udara (air traffic controller, ATC) di
basis bisnisnya banyak berhubungan bandara. Tanpa menara pemantau ini,
dengan migas urusan penerbangan bisa kacau karena
Kata yang tepat untuk menggantikan pesawat aka saling senggol, saling
kata yang bercetak tebal dalam kalimat serempet, sehingga saling tabrak di
ter sebut adalah udara. Saat menanjak di area sekitar
(A) badan usaha. bandara, jarak horizontal antarpesawat
(B) perbankan tidak boleh kurang dari tiga mil (sekitar 5
(C) keuangan km), sedangkan jarak vertikal tidak boleh
(D) retail kuranng dari 1.000 kaki (sekitar 300
(E) niaga meter), saat melayang diangkasa bebas,
jarak horizontal minimal 8 km,
16. Setiap komunitas sosial masyarakat sedangkan jarak vertikal minimal 600
tutur etnik memiliki bahasa dan budaya. meter.
Mereka melakukan praktik berbahasa Generalisasi yang tepat untuk
yang sekaligus juga menjalankan praktik melengkapi paragraf tersebut ialah
sosial budaya. Dalam menjalankan (A) Menara inilah yang mengatur jarak
praktik sosial budaya itu, komunitas minimal antarpesawat.
sosial itu menciptakan produk bahasa (B) ATC sangat penting di badara.
dan produk budaya. Salah satu bentuk (C) Jarak horizontal dan vertical
produk tersebut adalah lagu – lagu antarpesawat harus diatur.
daerah. (D) Antarpesawat harus diatur supaya
Untuk meneliti masalah apakah lagu tidak saling senggol dan tidak saling
daerah menggambarkan budaya tabrak.
komunitas sosial, judul penelitian yang (E) Karenanya, di bendara selalu ada
paling tepat adalah ATC.

Sony Sugema College 97


18. Setelah dibersihkan oleh seorang terapis, disebut program pembudayaan khusus
wajah pasien mulai diurut perlahan bagi masyarakat Daya. Rumah - rumah
dengan memakai minyak herbal, lalu di panjang harus dilenyapkan, baik dengan
bilas. Selanjutnya, wajah pasien diolesi cara halus maupun secara kasar
adonan masker yang terbuat dari (dibakar) kemudian rakyat diperintahkan
campuran bubuk sandal wood. Merah membangun rumah baru yang terpisah –
yang berhasiat melembutkan dan pisah. Sistim pertanian tradisional
meremajakan kulit. … Untuk membuka (berladang) harus diganti cara baru
pori – pori kulit wajah pasien diuapi (bersawah).Pemerintah juaga
dengan uap rempah – rempah selama 5 mendatngkan ribuan transmigrasi dari
menit, dan terakhir diolesi minyak lagi Pulau Jawa ke daerah tersebut, dengan
untuk menjaga kelembapan kulit. maksud untuk mendukung rencana itu.
Kalimat yang paling tepat untuk Adat harus diseleksi selaras dengan
melengkapi kalimat kosong pada teks selera pemerintah yang berkiprah pada
tersebut ialah masyarakat modern.
(A) Setelah pengeringan, bilas wajah Pertanyaan berikut ini yang jawabannya
pasien, kemudian kembali diolesi dapat ditemukan dalam teks tersebut
krim sambil memijat adalah.
(B) Setelah kering, wajah pasien dibilas, (A) Mengapa gaya hidup masyarakat
lalu kembali mengolesi krim sambil Daya yang sederhana itu sering
memijat. membawa suatu konotasi yang
(C) Setelah kering, wajah dibilas, lalu tragis masyarakat tersebut
kembali diolesi krim sambil di pijat. (B) Mengapa oknum pemerintah RI
(D) Setelah mengeringkan, membilas memandang masyarakat Daya
wajah pasien, kemudian mengolesi sebagai masyarakat primitif dan
krim sambil dipijat. tidak berbudaya?
(E) Setelah dikeringkan, wajah pasien (C) Bagaimanakah proses pelaksanaan
dibilas, lalu kembali mengolesi krim program pembudayaan yang
sambil memijat. dilakukan oleh pemerintah terhadap
masyarakat Daya?
19. Gaya, cara, dan pandangan hidup (D) Bagaimanakah kondisi dan reaksi
masyarakat Daya Tradisional yang serba masyarakat Daya dalam
sederhana, bermukin di rumah panjang menghadapi program pembudayaan
(lamin), berbau adat, dan magis sering yang dilakukan oleh pemerintah?
membawa suatu konotasi yang tregis (E) Benarkah Program pembudayaan
bagi masyarakat tersebut. Dahulu, oleh yang dilakukan oleh pemerintah
oknum pemerintah RI, mereka tersebut mampu menjadi
dipandang sebagai masyarakat primitif masyarakat Daya berbudaya
dan tidak berbudaya. Mereka dinilai modern?
sebagai penghambat lajunya roda
pembangunan Negara RI yang memiliki
citra yang moderen dan progresif.
Karena itu, sejak awal tahun – tahun
kemerdekaannya, pemerintah dengan
sangat giat melancarkan aksi yang
98 Sony Sugema College
Sony Sugema College
20. Kelompok kata berikut menunjukan (E) Sebelum kata penghubung dan
benda atau hal sejenis, KECUALI (kalimat pertama) harus diberi tanda
(A) panci, wajan, periuk koma(,)
(B) gurame, kakap, mujaer
(C) gunting, jarum, benang 23. Apehensi komunikasi merupakan
(D) sepatu, sandal, selop penyakit mental yang berupa takutnya
(E) guru, buku, disket seseorang untuk berkomunikasi.
Penyakit ini akan berakibat patal
21. Pada empat tahun terakhit ini, makin terhadap karier seseorang. Oleh karenaa
banyak pemerintah yang negaranya itu, penyakit tersebut harus diupayakan
pernah dikuasai rezim yang zalim penyembuhannya, antara lain dengan
membuka kembali sebagian sejarah berlatih dan banyak tampil di depan
hitamnya yang diwarnai tindak umum.
kekejaman aparat bersenjatanya sendiri Dalam kutipan di atas terdapat kata yang
pada masa lampau. tidak baku. Kata tersebut terdapat dalam
Berikut ini bukan unsur kalimat inti (A) kalimat 1
dalam kalimat tersebut, KECUALI (B) kalimat 2
(A) membuka kembali (C) kalimat 3
(B) dikuasai (D) kalimat 4
(C) diwarnai (E) kalimat 5
(D) rezim yang zalim
(E) pada empat tahun terakhir ini 24. Bagi sebagian orang di muka bumi ini,
tidur merupakan salah satu kebutuhan
22. Merumuskan judul karangan erat dasar manusia yang harus dipenuhi
kaitannya dengan topik dan tema serta untuk membantu menjaga metabolisme
tujuan karangan.Apabila topik tubuh. Orang dewasa dianjurkan untuk
merupakan gagasan pokok yang akan tidur minimal delapan jam sehari.
dibahas, judul merupakan nama yang di Kedua kalimat tersebut dapat
berikan untuk karangan itu. Judul digabungkan menjadi satu kalimat.
berfungsi sebagai slogan promosi untuk Gabungan yang tepat adalah
menarik sifat minat pembaca dan (A) Bagi sebagian orang di muka bumi
sebagai gambaran isi karangan. Judul ini, tidur merupakan salah satu
dapat di rumuskan setelah atau sebelum kebutuhan dasar manusia yang
karangan dibuat. harus dipenuhi untuk membantu
Dalam kutipan di atas, terdapat menjaga metabolisme tubuh karena
kesalahan tata tulis. Kesalahan yang orang dewasa dianjurkan untuk
dimaksud adalah tidur minimal delapan jam sehari.
(A) Penulisan kata apabila seharusnya (B) Bagi sebagian orang di muka bumi
apa bila. ini, tidur merupakan salah satu
(B) Penggunaan tanda koma (,) pada kebutuhan dasar manusia yang
kalimat kedua. harus dipenuhi untuk membantu
(C) Penulisan kata slogan seharusnya menjaga metabolisme tubuh sebab
selogan. orang dewasa dianjurkan untuk
(D) Penulisan kata dan pada kalimat tidur minimal delapan jam sehari
pertama seharusnya atau.
Sony Sugema College 99
(C) Bagi sebagian orang di muka bumi
ini, tidur merupakan salah satu
kebutuhan dasar manusia yang
harus dipenuhi untuk membantu
menjaga metabolisme tubuh,
sedangkan orang dewasa dianjurkan
untuk tidur minimal delapan jam
sehari
(D) Bagi sebagian orang di muka bumi
ini, tidur merupakan salah satu
kebutuhan dasar manusia yang
harus dipenuhi untuk membantu
menjaga metabolisme tubuh
sehingga orang dewasa dianjurkan
untuk tidur minimal delapan jam
sehari
(E) Bagi sebagian orang di muka bumi
ini, tidur merupakan salah satu
kebutuhan dasar manusia yang
harus dipenuhi untuk membantu
menjaga metabolisme tubuh apalagi
orang dewasa dianjurkan untuk
tidur minimal delapan jam sehari

25. Tindakan anarkis dan aroga itu sebagai


menifestasi kemarahan dan
kekecewaannya yang mendalam.
Padanan kata manifestasi yang tidak
tepat adalah
(A) perlakuan
(B) perwujudan
(C) pengejawantahan
(D) pengungkapan
(E) pernyataan

100 Sony Sugema College


Sony Sugema College
16. Tugas Mandiri Bahasa Indonesia

Sekitar 1.000 orang pengecer, Kepala Dinas Peternakan Jabar


pemotong, dan penyuplai daging sapi di Rachmat Setiadi menyayangkan rencana
sejumlah pasar tradisional Kota Bandung pemogokan para pedagang daging sapi. Ia
akan melakukan aksi mogok jual daging sapi juga memperkirakan, langkah mereka akan
selama dua hari, mulai Kamis (13/3) hingga berjalan kurang efektif. “Kami juga pernah
Jumat (14/3). Aksi tersebut merupakan upaya melakukan pertemuan, namun tak ada titik
mereka untuk memperoleh kenaikan harga temu karena para pedagang daging sapi
jual eceran daging sapi di pasar tradisional cenderung mengarah pada perbaikan situasi
yang sudah setahun ini terus merugi. internal mereka.
Ketua Asosiasi Pengusaha Daging (Sumber : Harian Pikiran Rakyat, 12 Maret 2008).

Sapi (Apdasi) Jabar yang juga penasihat


Apdasi Kota Bandung Dadang Iskandar di 1. Gagasan dalam bacaan di atas
Bandung, Selasa (11/3) mengatakan, rencana dikembangkan dengan pola ...
pemogokan penjual daging sapi itu dilakukan (A) sudut pandang
setelah disepakati sejumlah lapisan bisnis (B) deskriptif
daging sapi. Alasan utama, mereka sudah tak (C) deduktif
tahan karena usaha bisnis daging sapi terus (D) induktif
menerus “babak belur” karena harga dari (E) persepsi
penggemukan dan distributor sapi terus naik,
namun harga jual di eceran sulit disesuaikan. 2. Alasan utama, mereka sudah tak tahan
“Ini sebagai langkah terakhir setelah karena usaha bisnis daging sapi terus
upaya kami menemui sejumlah pihak terkait, menerus “babak belur” Istilah ‘babak
mulai distributor dan perusahaan belur’ memiliki pengertian ...
penggemukan sapi, sampai instansi terakit, (A) keadaan membahayakan
tak memperoleh hasil sesuai harapan. Aksi ini (B) kondisi mematikan
dilakukan dengan damai, hanya agar (C) berada pada puncak kritis
masyarakat mengetahui kesulitan kami”, (D) mengalami kerugian
katanya. (E) mengalami kemunduran
Seorang pegusaha penggemukan
dan distributor sapi hidup di Bandung yang 3. Dalam wacana di atas, rencana mogok
juga pengurus Asosiasi Pengusaha Feedlotter Asosiasi Pengusaha Daging Sapi (Apdasi)
Sapi Indonesia (Apfindo) Jabar Yudi Guntara Jabar merupakan kondisi paradoks
mengatakan, jika dihitung sejak awal tahun, konsumen daging
harga sapi hidup sebenarnya tidak naik. (A) pemerintah pusat
Namun jika dikaitkan dengan harga akhir (B) peternak sapi
2007 memang sudah naik di mana harga sapi (C) pemerintah daerah
hidup kini Rp19.000,00/kg dan karkas (D) bagianggota masyarakat
Rp39.000,00/kg. (E) konsumen daging
Sony Sugema College 101
4. Kepala Dinas Peternakan Jabar, Rachmat (B) menghilangkan kata tetapi pada
Setiadi, memperkirakan langkah mereka kalimat kedua.
akan berjalan kurang efektif. (C) menghilangkan tanda koma (,) pada
Ini mengandung makna ... kalimat ketiga.
(A) aksi mogok para pedagang akan (D) mengubah kata karena pada kalimat
dilarang ketiga dengan kata agar.
(B) aksi mogok para pedagang akan (E) menghilangkan kata dengan pada
berakhir rebut kalimat keempat.
(C) aksi mogok akan gagal
(D) aksi mogok tidak akan mencapai hasil 7. Kacang kedelai dikenal sebagai makanan
(E) aksi mogok akan berakhir dengan terbaik kadar proteinnya, dapat
tangan hampa mencapai 35% dari beratnya. Dikatakan
bahwa kacang kedelai dibandingkan
5. Kerugian yang diderita oleh para dengan beratnya dapat menghasilkan
pedagang daging sapi di Jabar akibat dua kali protein daging, empat kali telur,
dari ... empat kali gandum, lima atau enam kali
(A) keterlambatan pemerintah dalam roti, dan dua belas kali susu. Ternyata
bertindak protein kacang kedelai bukan saja
(B) harga jual beli daging sapi yang tidak jumlahnya yang banyak,
stabil tetapi juga mempunyai kualitas yang
(C) harga beli daging sapi yang terus baik. Umumnya, cukup dikenal bahwa
menaik protein hewani seperti daging, susu, dan
(D) ketidaktegasan pemerintah dalam telur adalah protein yang lengkap.
mengambil keputusan Namun, ternyata protein kacang kedelai,
(E) harga jual daging sapi yang terus walaupun termasuk protein nabati, lebih
menurun mirip menggambarkan protein hewani
daripada protein nabati.
6. Mengenai produk rokok memang harus Pokok pikiran dalam paragraf di atas
diakui terdapat ambiguitas. Meskipun adalah
kebiasaan merokok menyebabkan (A) Kacang kedelai dapat menghasilkan
kesehatan terganggu, tetapi pemerintah dua kali protein daging
selalu berharap dari produk rokok. Hal (B) Protein kedelai mempunyai kualitas
ini tidak dimungkiri, karena penerimaan baik.
negara dari cukai rokok dapat mencapai (C) Kacang kedelai adalah makanan
lebih dari Rp. 25 triliun. Kenyataan ini terbaik kadar dan kualitas
dapat diungkapkan bahwa kebiasaan proteinnya.
merokok memang merugikan kesehatan, (D) Kacang kedelai adalah protein
tetapi dengan menghentikan kebiasaan nabati.
merokok dapat mengurangi pendapatan (E) Kacang kedelai lebih lengkap
negara. daripada potein hewani.
Kalimat-kalimat dalam paragraf tersebut
akan menjadi kalimat baku apabila
diperbaiki dengan cara berikut, KECUALI
(A) menghilangkan kata mengenai pada
kalimat pertama.
102 Sony Sugema College
Sony Sugema College
8. Penggunaan kata berimbuhan me-i yang 11. Setelah merenung selama beberapa hari,
tepat terdapat dalam kalimat ia dapat mengungkapkan ide-idenya
(A) Dia meminjami sejumlah uang yang cemerlang sehingga dapat
kepada sahabat dekatnya. menyusun karya yang orisinal.
(B) Para petani menanami ketela pohon Lawan kata orisinal dalam kalimat di atas
di ladangnya. adalah kata-kata berikut, KECUALI
(C) Pak guru mengajari cara belajar (A) saduran (D) plagiat
yang efektif kepada murid- (B) duplikat (E) tiruan
muridnya. (C) turunan
(D) Kepala sekolah menugasi gurunya
untuk mengawasi jalannya pekan 12. Ia hendak ... para pemimpin partai
orientasi siswa. dengan gerakan nasional untuk ...
(E) Presiden menganugrahi tanda jasa program yang telah ... bersama. Kata
kepada tokoh masyarakat yang yang tepat untuk mengisi bagian
berprestasi. rumpang kalimat tersebut adalah
9. Pendidikan budi perkeri lebih difokuskan (A) mempermalukan, menyamakan,
pada pembentukan pribadi luhur, …. menyusun
pendidikan kewarganegaraan (B) mempertahankan, berdiskusi,
difokuskan pada pembentukan watak dibicarakan
kebangsaan. (C) mempermasalahkan, meluruskan,
Kata penghubung yang cocok untuk didiskusikan
melengkapi kalimat tersebut adalah (D) mempertemukan, membicarakan,
(A) sedangkan (D) akan tetapi disusun
(B) padahal (E) sebaliknya (E) mempertaruhkan, memangkas,
(C) namun disepakati

10. Pada waktu itu, nilai-nilai pluralitas 13. Pertumbuhan otak berkaitan erat
masyarakat dapat dipahami oleh semua dengan kecerdasan. Karena itu, untuk
tokoh dan elite tertentu, yang pada saat memperoleh Sumber Daya Manusia yang
yang sama, pemahaman itu disamarkan bermutu perlu dialokasikan angggaran
hanya pada tataran sloganistis dengan yang besar pula. Padahal, anggaran
menggunakan ungkapan yang negara untuk sektor pendidikan dan
membingungkan. kesehatan sangatlah kecil, kurang dari
Kalimat inti dari kalimat luas tersebut enam persen total APBN 2001. ketika
adalah alokasi anggaran itu kecil, yang
(A) Tokoh dan elite memahami nilai diperoleh pun adalah SDM dengan mutu
pluralitas. yang kurang memadai. Hal ini akan
(B) Nilai-nilai pluralitas dipahami. berpengaruh pada proses pendidikan
(C) Nilai-nilai pluralitas disamarkan. SDM berikutnya. Oleh karena itu, sudah
(D) Nilai-nilai pluralitas berbentuk menjadi keharusan bahwa anggaran
slogan. untuk sektor kesehatan harus
(E) Tokoh dan elite sering ditingkatkan.
membingungkan.

Sony Sugema College 103


Pikiran utama dalam paragraf di atas 17. Penulisan kalimat yang baku terdapat
terdapat pada kalimat berikut pada kalimat
(A) Pertumbuhan otak berkaitan erat (A) Presiden Susilo Bambang
dengan kecerdasan. Yudhoyono dapat menerima dasar
(B) Sumber Daya Manusia bermutu alasan dipilihnya 65 tahun sebagai
memerlukan anggaran besar. batas usia pensiun hakim agung.
(C) Anggaran negara sangat kecil. (B) Pemerintah menurunkan harga
(D) Hal itu berpengaruh para proses premium bersubsidi sebesar Rp. 500
pendidikan SDM. per liter dari harga saat ini Rp. 6.000
(E) Peningkatan anggaran menjadi per liter.
keharusan. (C) Penurunan harga premium
bersubsidi ini kata Sri Mulyani,
14. Kenaikan harga BBM memicu masalah menyikapi terus melemahnya harga
ekonomi dan sosial. Masalah itu ditandai minyak dunia.
dengan naiknya harga bahan baku (D) Dia mengklaim, harga premium di
industri nasional. Dunia usaha akan Indonesia masih yang terendah di
mengalami penurunan produktivitas. Asia. Bahkan dengan Malaysia yang
Pemutusan hubungan kerja dan telah menurunkan empat kali, harga
pengangguran pun akan terus premium Rp 5.500 masih lebih
bertambah. Ayah budi pun terkena PHK rendah.
setelah 15 tahun bekerja di PT (E) “Gejolak ekonomi global sudah pasti
Pringgayuda. Selain itu, kenaikan harga melemahkan ekonomi kita. Karena
BBM juga berpotensi menuai perotes itu, perlu antisipasi guna
publik dan menyulut keresahan sosial. menetralisasi beban masyarakat,”
Hal ini menjadi ujian berat bagi katanya.
pemerintahaan saat ini.
18. Pulau Kela dalam cerita “1001 Malam”
itu diasumsikan hovig sebagai Pulau
15. Kalimat sumbang pada paragraf di atas
bangka sekarang.
terdapat pada
Ejaan yang perlu diperbaiki terdapat
(A) kalimat 1 (D) kalimat 4
dalam kalimat
(B) kalimat 2 (E) kalimat 5
(A) memberikan tanda koma (,) setelah
(C) kalimat 3
kata itu
(B) tanda petik (‘’) di depan dan di
16. Pemakaian huruf kapital yang tidak
belakang 1001 malam dihilangkan
benar berikut ini adalah
(C) mengubah huruf K pada kata Kela
(A) bahasa Inggris
menjadi k
(B) bangsa Inggris
(D) mengubah kata hovig menjadi
(C) kunci Inggris
bercetak tegak
(D) Perdana Menteri Inggris
(E) mengubah huruf h pada kata hovig
(E) rakyat Inggris
dan b pada kata bangka menjadi H
dan B

104 Sony Sugema College


Sony Sugema College
19. Penulisan gabungan kata di bawah ini 22. Saat berlaga di Yuan Shen Gymnasium of
benar semuanya, KECUALI Pudong, Shanghai, Taufik menyerah dua
(A) tanggung jawab, kerjasama, set langsung dari pemain tuan rumah.
tandatangan, terima kasih Ini kalimat tersebut adalah
(B) semipermanen, mancanegara, (A) Taufik berlaga di Yuan Shen
pascasarjana, saptakrida Gymnasium of Pudong, Shanghai
(C) acapkali, adakalanya, barangkali, (B) kekalahan Taufik dari pemain tuan
bagaimana rumah
(D) mata ajar, meja tulis, orang tua, (C) pemain tuan rumah menang atas
simpang empat Taufik
(E) kacamata, dukacita, sapu tangan, (D) pertandingan antara Taufik dan
halalbihalal pemain tuan rumah
(E) Taufik menyerah dua set langsung
20. Negara Indonesia mempunyai bahasa ...,
yakni bahasa Indonesia. Bahasa
23. (a) Teater tradisional berlatar belakang
Indonesia merupakan ... bangsa yang
kebudayaan Timu.
sangat penting kedudukannya.
(b) Teater modern berlatar belakang
Kata bentukan baku yang tepat untuk
kebudayaan Barat.
mengisi bagian yang rumpang dalam
Kedua kalimat di atas dapat digabungkan
kalimat di atas adalah
menjadi
(A) pemersatu, persatuan
(A) Teater tradisional berlatar belakang
(B) persatuan, pemersatu
kebudayaan Timur sehingga teater
(C) persatuan, persatuan
modern berlatar belakang
(D) persatuan, mempersatukan
kebudayaan Barat.
(E) pemersatu, mempersatukan
(B) Teater tradisional berlatar belakang
kebudayaan Timur, sedangkan
21. Kalimat di bawah ini tidak efektif
teater modern berlatar belakang
KECUALI
kebudayaan Barat.
(A) Dalam bab ini, akan menelusuri
(C) Teater tradisional berlatar belakang
kasus malpraktik.
kebudayaan Timur jadi teater
(B) Berita musibah gempa itu saya
modern berlatar belakang
sudah sampaikan pada Pak Lurah.
kebudayaan Barat.
(C) Bagi segenap pelajar yang akan
(D) Teater tradisional berlatar belakang
mengajukan proposal penelitian
kebudayaan Timur bahkan teater
harap mengirimkan datanya segera.
modern berlatar belakang
(D) Beberapa artikel-artikel ilmiah itu
kebudayaan Barat.
dimuat dalam jurnal ilmiah.
(E) Teater tradisional berlatar belakang
(E) Makalah ini membahas masalah
kebudayaan Timur apalagi teater
kesehatan masyarakat di daerah
modern berlatar belakang
rawan gempa.
kebudayaan Barat.

Sony Sugema College 105


24. Meskipun Indonesia merupakan negara
pengekspor minyak bumi, tetapi
pemerintah merasa berkewajiban untuk
melaksanakan program hemat energi
sejak tahun 2000.
Kalimat di atas tidak baku. Yang perlu
diperbaiki dalam kalimat di atas adalah
(A) menghilangkan kata tetapi
(B) memberikan tanda koma (,) sesudah
meskipun
(C) menghilangkan tanda koma (,)
sesudah bumi
(D) menghilangkan kata untuk
(E) mengganti kata sejak dengan dari

25. Makna penggabungan kata merdeka


berdaulat dapat juga dilihat pada
gabungan kata dalam kalimat
(A) Mereka mengobrol sambil sesekali
bersenda gurau.
(B) Kami merasa senasib
sepenanggungan.
(C) Orang itu malu-malu kucing ketika
dipersilakan makan oleh tuan
rumah.
(D) Dia menjadi hilang semangat
setelah mengetahui pembalap
unggulannya kalah.
(E) Pelari itu bersiap-siap sambil pasang
kuda-kuda.

106 Sony Sugema College


Sony Sugema College
SNMPTN Tahun 2008 Kode Soal 301

1. Asumsi nilai budaya bangsa kita 2. Pandai membaca peluang pasar dan
menjulang amat tinggi, melangit, dan tekun merupakan kunci untuk meraih
amat luhur, seperti tercermin dalam sukses bagi mereka yang menjalani
nilai-nilai Pancasila. Meskipun demikian, profesi sebagai wiraswasta. Prinsip
dalam kenyataannya, asumsi moral yang utama dalam dunia bisnis harus
amat hebat tersebut tidak berpijak di dipegang teguh, antara lain jangan takut
bumi kenyataan karena hanya gagal, tetapi mempunyai keberanian
mengambang di awang-awang dan tidak untuk mengambil langkah dan bebaskan
ada kaitannya dengan kehidupan nyata pikiran akan bertemu dengan kesalahan.
di tengah kehidupan masyarakat. Di Dengan bekal tekat itulah, Giyono
antara Pancasila dan kehidupan nyata pemuda di lereng Gunung Merapi
sehari-hari bangsa terdapat jurang yang berhasil merintis bisnis bibit sayuran.
luas. Orang tidak merasakan bahwa Saat ini pun telah memiliki lebih dari
nilai-nilai Pancasila menjiwai seluruh sepuluh anak buah yang membantu
tingkah laku dan sikap hidup setiap usaha pembibitannya.
anggota masyarakat kita, baik yang Simpulan yang dapat diambil dari
berada dalam dunia birokrasi dan di luar paragraf di atas adalah
birokrasi. (A) Giyono pemuda yang sukses dalam
Istilah yang paling tepat untuk berwirausaha pembibitan sayuran.
menggambarkan adanya kesenjangan (B) Kunci kesuksesan terletak pada
antara asumsi moral dan kenyataan pandai membaca peluang pasar.
sehari-hari sebagaimana diilustrasikan (C) Prinsip-prinsip dasar dalam dunia
dalam paragraf di atas adalah bisnis.
(A) kontradiksi (D) Usaha pembibitan Giyono telah
(B) ironi berhasil membuka lahan kerja di
(C) distorsi wilayahnya.
(D) persepsi (E) Ketekunan Giyono dapat menjadi
(E) kontraindikasi teladan bagi pemuda lainnya.
3. Suhu dan derajat keasaman menjadi
indikator konsentrasi bahan-bahan kimia
berbahaya yang terlarut dalam air
sungai.
Kata indikator dalam kalimat tersebut
sama maknanya dengan kata
(A) penentu (D) pengukur
(B) petunjuk (E) penyebab
(C) pembentuk
Sony Sugema College 107
4. (1) Burung termasuk dalam jenis unggas Hal yang dianalogikan dalam paragraf di
yang bisa terbang. (2) Namun, ada atas adalah
burung yang tidak bisa terbang. (3) (A) prestasi dan pisau
Contohnya burun unta (struthio (B) otak manusia dengan pisau
camelus). (4) Burung unta adalah burung (C) berlatih dengan diasah
terbesar yang masih hidup dan tidak bisa (D) manusia dengan pisau
terbang. (5) Dahulu, burung ini terdapat (E) kepandaian dengan ketajaman
dalam jumlah besar di Afrika dan Asia
Barat Daya. (6) Sekarang, burung ini 6. Seluruh rakyat mempunyai visi yang
hanya ditemukan di beberapa daerah di sama mengenai perjuangan bangsa.
Asia Timur. (7) Ada juga burung unta Arti kata visi dalam kalimat tersebut
yang hidup di peternakan di Afrika adalah
Selatan. (8) Sementara itu, di Australia (A) pandangan ke depan
Selatan, ada burung unta yang sudah (B) kemampuan untuk melihat pada inti
dijadikan hewan piaraan. (9) Seperti persoalan
halnya burung lain yang tidak bisa (C) ketajaman penglihatan melalui
terbang, burung unta sudah terbiasa kehalusan jiwa
pada kehidupan di tanah dan dapat (D) apa yang tampak dalam khayalan
berlari dengan baik. (10) Dalam hal ini (E) pengamatan
ayam tidak bisa disamakan dengan
burung unta. 7. Di Yayasan Galuh, jumlah pasien usia
Paragraf di atas akan merupakan remaja justru mendominasi, mulai dari
paragraf yang memiliki kepaduan jika ... remaja putus sekolah, remaja yang
(A) kalimat (2) dan (3) digabungkan. masih sekolah, remaja yang tidak diurusi
(B) kalimat (8) dan (9) dihilangkan. keluarga, hingga remaja yang memiliki
(C) kalimat (10) dihilangkan. obsesi tinggi, tetapi tidak kesampaian.
(D) kalimat (7) dihilangkan. Kalimat inti dari kalimat luas tersebut
(E) kalimat (7), (8), dan (9) adalah
digabungkan. (A) Jumlah pasien uisa remaja
mendominasi.
5. Otak manusia ibarat sebuah pisau. Otak (B) Pasien usia remaja memiliki obsesi.
manusia yang cerdas tidak akan (C) Obsesi remaja tidak kesampaian.
berprestasi tinggi bila tidak belajar dan (D) Yayasan Galuh didominasi oleh
berlatih. Otak manusia yang IQ-nya berbagai masalah remaja.
sedang-sedang saja akan mendapat (E) Mulai remaja putus sekolah sampai
prestasi gemilang bila belajar terus- remaja obsesi tinggi ada di Yayasan
menerus. Demikian pula dengan pisau. Galuh.
Sebilah pisau tajam akan menjadi
tumpul bila tidak diasah. Sebaliknya,
sebilah pisau yang tumpul akan menjadi
tajam bila diasah terus-menerus. Dengan
demikian, bila kita ingin menjadi
manusia yang berprestasi hendaknya
seperti pisau yang diasah terus.

108 Sony Sugema College


Sony Sugema College
8. Adanya disekuilibrium sosial budaya 11. Di antara kalimat-kalimat berikut, yang
dapat mengganggu jalannya bukan kalimat baku adalah
pembangunan. (A) Dengan pendidikan diharapkan
Padan kata yang tepat untuk kata dapat membentuk manusia
disekuilibrium dalam kalimat di atas Indonesia yang demokratis,
adalah berkeadilan, dan tidak diskriminatif.
(A) ketidaksinambungan (B) Bahan ajar yang berperspektif
(B) ketidakseimbangan gender sebaiknya dimulai sejak dini,
(C) ketidaksamaan mulai tingkat pendidikan dasar.
(D) keberlainan (C) Ketika diwawancarai media, Sarkozy
(E) keberagaman mengatakan bahwa kehadiran
Prancis di Afganistan bukan
9. Rancangan Undang-Undang Badan kuantitatif, melainkan kualitatif.
Hukum Pendidikan yang tampaknya (D) Beberapa jam sesudah menerima
akan disahkan dalam waktu dekat ini Sarkozy, Karzai menerima kunjungan
benar-benar menjadi instrumen hukum Rudd.
yang akan melegalisasi segala sesuatu (E) Berdasarkan laporan Dinas
yang bersifat internasional. Pertanian Jawa Tengah, setiap dua
bulan sekali dibuka produk
Kata instrumen dalam kalimat tersebut pertanian Jawa Tengah.
digunakan dengan makna
(A) pedoman 12. Setelah gas Pertamina di Porong
(B) dokumen resmi meledak dan menewaskan serta melukai
(C) sarana penelitian beberapa orang, pemerintah menyebut
(D) patokan luapan lumpur sebagai disaster.
(E) alat
Makna kata disaster dalam kalimat
10. Kematian tokoh oposisi Benazie Bhuto tersebut adalah
ketika sedang berkampanye itu (A) kecelakaan
menyebabkan kerusuhan hebat yang (B) musibah
memakan banyak korban. (C) bencana
(D) kelalaian
Kalimat di atas berasal dari kalimat inti (E) kecerobohan
(A) kerusuhan menyebakan
(B) Benazir Bhuto berkampanye
(C) kerusuhan memakan korban
(D) kematian Benazir Bhuto
(E) Benazir Bhuto menyebabkan
kerusuhan

Sony Sugema College 109


13. Dengan adanya bank syariah diharapkan 15. Untuk meningkatkan daya tahan tubuh,
dapat mendukung pengembangan umumnya suplemen vitamin dikonsumsi,
ekonomi nasional, memfasilitasi segmen tetapi ternyata banyak pula vitamin
pasar yang belum terjangkau atau tidak yang justru sangat berbahaya.
berminat dengan bank konvensional, Kalimat di atas dikembangkan dari
dan dapat memfasilitasi distribusi utilitas kalimat inti
barang modal untuk kegiatan produksi (A) Daya tahan tubuh meningkat.
melalui skema sewa-menyewa. (B) Meningkatkan daya tahan.
Ketidakbakuan kalimat tersebut terletak (C) Suplemen vitamin sangat
pada berbahaya.
(A) kurangnya tanda koma (,) sebelum (D) Suplemen vitamin meningkatkan.
kata diharapkan. (E) Suplemen vitamin dikonsumsi.
(B) pemakaian kata dengan.
(C) gabungan kata bank syariah yang 16. Otonomi sekolah berepran dalam
ditulis dengan huruf kecil. menampung konsensus umum tentang
(D) pemakaian pengembangan yang pemberdayaan sekolah yang meyakini
seharusnya perkembangan. bahwa untuk meningkatkan kualitas
(E) pemakaian tanda koma (,) sebelum pendidikan keputusan sedapat mungkin
kata dan. dibuat oleh mereka yang ada di garis
depan yang bertanggung jawab secara
14. Silang pendapat yang terjadi antara langsung terhadap pelaksanaan
Departemen Kesehatan dan Badan kebijakan serta guru dan kepala sekolah
Pengawasan Obat dan Makanan (POM) yang terkena akibat kebijakan tersebut.
dalam penanganan kasus formalin Kalimat tersebut dikembangkan dari
secara tidak langsung menggeser isu kalimat inti
pokok yang berpihak pada kepentingan (A) Otonomi sekolah berperan.
konsumen, yaitu mendapatkan produk (B) Otonomi sekolah menampung
makanan yang aman bagi kesehatan. konsensus.
Kalimat inti dari kalimat luas di atas (C) Otonomi sekolah meningkatkan
adalah kualitas.
(A) Silang pendapat terjadi. (D) Otonomi sekolah dibuat.
(B) Silang pendapat menggeser isu (E) Otonomi sekolah bertanggung
pokok. jawab.
(C) Kasus formalin berpihak pada
kepentingan konsumen. 17. Semua pihak wajib menjunjung tinggi
(D) Silang pendapat berpihak pada hasil kesepakatan bersama yang telah
kepentingan konsumen. disusunnya.
(E) Kepentingan konsumen, yaitu Istilah yang tepat untuk menggantikan
mendapatkan produk makanan. kata kesepakatan bersama adalah
(A) konsensus
(B) konvensi
(C) konsepsi
(D) konklusi
(E) konsesi

110 Sony Sugema College


Sony Sugema College
18. Penggunaan tanda baca dalam kalimat 20. Penerapan EYD yang benar terdapat
berikut ini sesuai dengan aturan EYD, dalam kalimat
KECUALI (A) Kegiatan perkuliahan dimulai,
(A) Di jalur Losari – Cirebon belum semua kewajibanpun administrasi
terlihat adanya perbaikan, meskipun harus diselesaikan.
kerusakannya tergolong parah. (B) H. Krupper, seorang ahli pra sejarah,
(B) Daerah nyaris tidak berdaya, melaporkan bahwa telah ditemukan
terutama dalam mengelola situs Bukit Karang di daerah Langsa.
keuangan daerah. (C) Akhirnya, Diana mendapatkan izin
(C) Ketika acara seminar berlangsung, belajar ke luar negeri setelah
Anda bertugas sebagai notulis. berjuang bertahun-tahun.
(D) Kita sekarang memerlukan perlatan (D) Hari Jum’at yang lalu, hujan deras
sekolah: sepatu, tas, dan alat tulis. mengguyur Jakarta seharian.
(E) Hak Asasi manusia adalah hak yang (E) Kami tidak bisa menerima surat
dimiliki oleh setiap manusia sejak tersebut, karena faksimile di kantor
lahir yang tidak boleh dilanggar, kami sedang rusak.
dibatasi, ataupun dikurangi oleh
siapa pun. 21. Penulisan kata serapan dalam kalimat-
kalimat berikut benar, KECUALI yang
19. Bentuk kata serapan yang benar terdapat dalam kalimat
terdapat dalam kalimat (A) Setiap pagi kumandang azan itu
(A) Radio pada mobil merupakan membangunkan saya.
asesori yang digemari oleh banyak (B) Muna bekerja di laboratorium itu
konsumen. sebagai bakteriolog.
(B) Tarian itu dianggap akseptebel (C) Orang yang mengusahakan bank
untuk ditampilkan di depan tamu. disebut bankir.
(C) Pada zaman penjajahan di kota (D) Keamanan di ibu kota merupakan
Malang terbit banyak koran. Koran- barometer keamanan di Indonesia.
koran tersebut tidak dapat terbit (E) Sebelum meninggal, N sempat
secara kontinyu. menyelesaikan otobiografinya.
(D) Pematuhan terhadap prinsip kerja
sama dan prinsip kesopanan akan 22. Kalimat berikut yang seluruh ejaannya
melahirkan komunikasi yang ditulis sesuai dengan aturan EYD adalah
bonafide sekaligus melahirkan pula (A) Tiga per empat bagian diberikan
wacana yang wajar. kepada orang lain.
(E) Komunikasi merupakan syarat (B) Sekalipun dia belum pernah ke Bali.
esensial bagi terciptanya prasyarat (C) Kemana pun perginya, ia tidak lupa
tumbuhnya kegiatan yang terbuka, membawa kamera.
yang tidak elit. (D) Kalaupun hari itu turun hujan lebat,
saya tetap bersedia datang pada
acara Anda.
(E) Mereka satu per satu datangnya
tetapi tidak satu pun yang absen.

Sony Sugema College 111


23. Penulisan bilangan yang tidak sesuai (C) asumsi esensial
dengan yang digariskan dalam EYD (D) apriori
dijumpai dalam kalimat (E) generalisasi abstrak
(A) Dari hasil penelitian yang dilakukan
Yayasan Kusuma Buana di sekolah- 25. Murid-murid memperbaiki kesalahan
sekolah di Jakarta, prevalensi pekerjaan rumah matematika. Perbaikan
anemia di setiap sekolah antara 20 itu sudah diperiksa guru.
persen dan 35 persen, bahkan ada Pilihlah nomor dengan pasangan
yang mencapai 60 persen. bentukan kata yang betul seperti contoh
(B) Sekolah Menengah Kejuruan Negeri kalimat di atas.
(SMKN) X, dua bulan yang lalu, (A) Warga kota memilih calon yang
dikunjungi oleh 20 guru dan 5 populer. Pilihan dilaksanakan
pejabat Pemerintah Malaysia. beberapa minggu yang lalu.
(C) Penghargaan Festival Teater (B) Sulit sekali menentukan masa depan
diperebutkan oleh 18 finalis yang keadaan negara kita. Para peramal
tampil di Teater Luwes Institut saja tidak mampu memprediksi
Kesenian Jakarta dan Teater Studio ketentuan seperti itu.
Taman Ismail Marzuki. (C) Para korban bencana luapan lumpur
(D) Dari 422 laporan yang masuk ke Sidoarjo tidak tahu lagi ke mana
komisi Kejaksaan, sebanyak 202 mereka harus mengadukan
laporan dilanjutkan ke Jaksa Agung. nasibnya. Pengaduannya ditanggapi
(E) Dari tiga ratus laporan masyarakat dingin di mana-mana.
yang masuk ke Komisi Kejaksaan (D) Para ahli hukum meniadakan
pada kurun April – Desember 2006, ketentuan yang penting dari
yang diteruskan ke Jaksa Agung rencana undang-undang itu.
sebanyak dua ratus laporan. Ketiadaan ketentuan itu
menimbulkan perdebatan seru di
24. Studi antropologi ekologi yang parlemen.
diharapkan dapat menghindari berbagai (E) Pelapor terjadinya kejahatan
persoalan asumsi-asumsi esensial, dilindungi undang-undang. Laporan
apriori, dan generalisasi abstrak adalah itu disampaikan secara rahasia.
studi yang bermaksud melakukan suatu
pembuktian secara empirik dengan
menempatkan praktik-praktik eksploitasi
sumber-sumber daya alam dan
konsekuensinya dalam konteks-konteks
pengaruh sosial budaya internal dan
eksternal yang historis.
Inti gabungan kata studi antropologi
ekologi yang diharpkan dapat
menghindari berbagai persoalan asumsi-
asumsi esensial, apriori, dan generalisasi
abstrak dalam kalimat di atas adalah
(A) antropologi ekologi
(B) studi
112 Sony Sugema College
Sony Sugema College
SNMPTN Tahun 2009 Kode Soal 183

1. (1) Setidaknya lima demonstran tewas di 2. Indonesia memiliki pulau yang


Mogadishu, Somalia. (2) Mereka menjadi jumlahnya puluhan ribu. Sebagian besar
korban hantaman peluru yang pulau tersebut telah diberi nama dan
ditembakkan aparat kepada demonstran berpenghuni. Pemberian nama pulau
yang memprotes kenaikan harga pangan. dilakukan agar mudah diidentifikasi.
(3) Di Burkina Faso, serikat-serikat buruh Pemerintah sekarang ini sedang
menyerukan pemogokan masal sebagai menggalakkan program pemberian nama
protes atas kenaikan harga pangan. (4) pulau terluar di Indonesia. Pemberian
Di Kamerun seratus orang tewas pada dan pemasangan nama pulau dilakukan
protes besar terkait kenaikan biaya oleh Departemen Kelautan, yang
hidup. (5) Negara-negara seperti Pantai melibatkan TNI Angkatan Laut.
Gading, Mozambik, Afrika Selatan, Haiti, Berdasarkan teks tersebut, pernyataan
Argentina, Peru, Afganistan, Bangladesh, dalam pilihan berikut yang merupakan
Vietnam, dan Rusia juga diharubirukan opini adalah ....
oleh krisis pangan. (6) Krisis pangan (A) Indonesia memiliki banyak pulau
benar-benar menusuk setajam pisau dan yang belum diberi nama.
memakan korban langsung. (B) pulau yang belum diberi nama
Kalimat topik paragraf tersebut adalah perlu dilakukan pemberian nama.
kalimat .... (C) TNI Angkatan Laut sedang
(A) ke-1 melakukan pemasangan nama
(B) ke-2 pulau.
(C) ke-4 (D) program pemberian nama pulau
(D) ke-5 sedang dilakukan pemerintah.
(E) ke-6 (E) Departemen Kelautan bekerja
sama dengan TNI Angkatan Laut.

Sony Sugema College 113


3. Dari 10 orang karyawan berpendidikan 4. Manakah pilihan di bawah ini yang
SD yang di-PHK, tidak seorang pun yang termasuk kalimat efektif?
tidak mengalami depresi, yakni 60% (A) Dalam musyawarah itu
mengalami depresi ringan dan 40% menghasilkan lima ketetapan yang
depresi sedang. Dari 15 orang ter-PHK harus dilaksanakan bersama.
yang berpendidikan SMP, 7% depresi (B) Dalam pertemuan itu memutuskan
berat, 66% mengalami depresi sedang, bahwa Andi yang ditunjuk menjadi
20% depresi ringan, dan 7% tidak ketua koperasi.
mengalami depresi. Sementara itu, dari (C) Kegagalan proyek itu karena
10 orang ter-PHK berpendidikan SMA, perancangan yang tidak baik dan
tidak seorang pun yang tidak mengalami pengawasan yang kurang cermat.
depresi, 40% mengalami depresi ringan, (D) Meskipun negara itu merupakan
30% depresi berat dan 30% depresi penghasil kapas nomor satu di
sedang. Dari 5 orang ter-PHK dunia, tetapi harga tekstilnya
berpendidikan perguruan tinggi (PT), 2 sangat tinggi.
orang yang tidak mengalami depresi, 2 (E) Budiman ingin menjadi juara
orang mengalami depresi ringan, dan 1 umum di sekolahnya, tetapi ia
orang mengalami depresi berat. hanya berhasil menjadi juara
Pernyataan berikut yang sesuai dengan ketiga.
isi teks adalah ....
(A) persentase karyawan di-PHK yang 5. Ada pernyataan menarik yang patut
mengalami depresi untuk dicermati dari pernyataan Zahidul
karyawan berpendidikan SMP Huque, perwakilan United Nation Fund
lebih tinggi daripada karyawan for Population (UNFPA) di Indonesia
berpendidikan SMA. ketika peluncuran laporan “Epidemi AIDS
(B) persentase karyawan di-PHK yang Global 2008” di Jakarta 13 Agustus 2008.
mengalami depresi pada jenjang Ia menyatakan bahwa meskipun
SD lebih tinggi daripada jenjang Indonesia termasuk kategori “kluster”
SMA. kedua dalam penyebaran HIV/AIDS di
(C) persentase karyawan di-PHK yang dunia bersama dengan Cina, Malaysia,
mengalami depresi pada jenjang India, Nepal, dan Vietnam, tetapi
SMP lebih tinggi daripada jenjang Indonesia merupakan negara dengan
PT. kasus HIV/AIDS yang terus meningkat
(D) persentase karyawan di-PHK yang dari tahun ke tahun. Bahkan, saat ini
mengalami depresi ringan dan Indonesia tercatat sebagai negara
berat pada jenjang SMP lebih dengan laju epidemi HIV/AIDS tercepat
tinggi daripada jenjang PT. di Asia. Pengertian kluster kedua adalah
(E) persentase karyawan di-PHK yang negara-negara yang sebenarnya rendah
mengalami depresi ringan dan dalam penyebaran HIV, tetapi negara
sedang pada jenjang SMA lebih bersangkutan justru gagal dalam
tinggi daripada jenjang SMP. menanggulanginya.

114 Sony Sugema College


Sony Sugema College
Berdasarkan teks di atas, simpulan di Kanal model SMART dinilai cerdas karena ...
bawah ini benar, KECUALI .... (A) cara kerja kanal sungguh-sungguh
(A) Indonesia paling cepat dalam modern.
penyebaran HIV/AIDS saat ini. (B) pengaplikasian teori-teori yang
(B) Indonesia termasuk rendah dalam memang canggih.
penyebaran HIV/AIDS. (C) perkembangan ilmu dan teknologi
(C) Indonesia tidak berhasil dalam terowongan.
menangani kasus HIV/AIDS. (D) dapat berfungsi ganda dalam
(D) Penyebaran HIV/AIDS di Indonesia sistem kerjanya.
termasuk kelompok kedua. (E) cara membuka dan menutup pintu
(E) Penyebaran HIV/AIDS di Indonesia kanal otomatis.
lebih cepat daripada Cina.
7. Seseorang akan mengembangkan
6. Selintas, kanal itu terlihat seperti karangan yang bertema upaya
terowongan di jalan tol biasa. Namun, mengatasi kekurangan gizi pada balita.
jika diperhatikan ada yang istimewa dari Berdasarkan tema tersebut, topik untuk
kanal yang berdiameter 11,8 meter itu. kerangka karangan yang runtut adalah ....
Saluran itu berada di bawah tanah, (A) masalah yang timbul akibat
dibangun tepat di bawah jalan tol kekurangan gizi pada balita,
Kualalumpur – Seremban. Kanal tersebut pentingnya mengatasi kekurangan
terdiri atas tiga lapis, mirip kue lapis. gizi, langkah mengatasi
Lapis pertama dan kedua untuk jalan kekurangan gizi, saran yang dapat
raya bebas dari kemacetan, dan lapis dilakukan untuk mengatasi
ketiga untuk saluran air bawah tanah. kekurangan gizi.
Pemerintah Malaysia menamakan (B) pentingnya mengatasi kekurangan
lubang panjang itu SMART, kependekan gizi pada balita, masalah yang
dari Storwater Management and Road timbul akibat kekurangan gizi,
Tunnel. langkah mengatasi kekurangan
Terowongan fungsi ganda itu gizi, saran yang dapat dilakukan
dirancang untuk mengantisipasi banjir, untuk mengatasi kekurangan gizi.
sekaligus mengurangi kemacetan di jalan (C) langkah mengatasi kekurangan gizi
tol. Jika debit air bertambah besar, pintu pada balita, pentingnya mengatasi
kedap air akan menutup akses masuk kekurangan gizi, masalah yang
kendaraan. Setelah tidak ada lagi timbul akibat kekurangan gizi,
kendaraan di dalam terowongan, jalan saran yang dapat dilakukan untuk
raya berubah menjadi saluran air. Harus mengatasi kekurangan gizi.
diakui, terowongan itu memang cerdas. (D) latar belakang timbulnya
Saat mobil lewat, kanal tetap dapat kekurangan gizi pada balita,
mengalirkan air dari hulu ke hilir, jika ada langkah mengatasi kekurangan
kiriman air yang tidak dapat ditampung gizi, masalah yang timbul akibat
oleh daerah resapan. kekurangan gizi, saran yang dapat
(Majalah Tempo, Oktober 2008) dilakukan untuk mengatasi
kekurangan gizi.

Sony Sugema College 115


(E) pentingnya saran yang dapat Paragraf tersebut dikembangkan dengan
dilakukan untuk mengatasi menggunakan pola sebab-akibat karena ....
kekurangan gizi pada balita, latar (A) kalimat utamanya menyatakan
belakang timbulnya kekurangan sebab dan akibat, sedangkan
gizi, masalah yang timbul akibat kalimat lainnya menyatakan
kekurangan gizi, langkah akibat.
mengatasi kekurangan gizi. (B) kalimat utama menyebabkan
sebab dan kalimat penjelas
8. .... Salah satu penyakit kulit adalah acne menyatakan akibat.
vulgaris. Tidak ada seorang pun di dunia (C) kalimat pertama menyatakan
ini yang tidak pernah menderita penyakit sebab dan kalimat lainnya
ini. Acne vulgaris adalah penyakit menyatakan akibat.
peradangan folikel sebasea yang (D) kalimat pertama dan keempat
umumnya terjadi pada masa remaja dan menyatakan sebab, sedangkan
dapat sembuh sendiri. Dalam lainnya menyatakan akibat.
masyarakat umum, acne vulgaris biasa (E) kalimat ketiga menyatakan akibat,
dikenal dengan istilah jerawat. sedangkan lainnya menyatakan
Kalimat manakah yang tepat untuk sebab.
mengisi titik-titik teks di atas?
(A) Acne vulgaris adalah penyakit kulit 10. Seorang pejabat pemerintah India
biasa. menyimpulkan bahwa di India banyak
(B) Acne vulgaris dialami semua orang miskin, terlalu padat
manusia. penduduknya, dan umumnya
(C) Penyakit kulit banyak jenis dan kekurangan gizi sebab agama mereka
macamnya. tidak mengizinkan untuk membunuh dan
(D) Jerawat banyak jenis dan mengonsumsi sapi yang berkeliaran di
macamnya. jalan. Ketika pejabat menganjurkan
(E) Penyakit yang diderita remaja penyembelihan secara besar-besaran,
bermacam-macam. ternyata pendapat dan tindakan ini
sangat tidak diinginkan karena jumlah
9. Siaran televisi sehari penuh hewan pembajak akan sangat berkurang,
menyebabkan anak-anak malas dan orang India tidak lagi mendapatkan
melakukan aktivitas. Mereka lebih suka kotoran sapi untuk pupuk dan sumber
menghabiskan waktu dengan duduk utama bahan bakar dalam memasak.
lama di depan layar kaca sehingga Agaknya, jika orang Hindu tidak lagi
melupakan berbagai kewajiban. Akibat dilarang untuk membunuh sapi, mungkin
yang lebih parah adalah dari segi masalah lain akan timbul, yaitu belum
kesehatan, yakni menumpuknya kadar cukupnya fasilitas di India untuk
lemak dalam tubuh. Olah raga yang mendirikan industri daging dalam skala
kurang dan kebiasaan menonton sambil besar. (dari Pokok-pokok Antropologi,
menikmati makanan atau cemilan 1990).
merupakan penyebab utama. Bahkan
yang lebih parah, mereka tidak sempat
atau tidak suka bersosialisasi dengan
teman.
116 Sony Sugema College
Sony Sugema College
Pernyataan yang tidak relevan dengan fisiknya dan kecerdasannya. (Kompas, 26
teks di atas adalah .... September 2007).
(A) keputusan seorang pemimpin Pernyataan dalam pilihan berikut ini
sering tidak cocok dengan kondisi yang tidak sesuai dengan teks di atas
yang ada. adalah ....
(B) mengubah sesuatu dalam (A) pemerintah perlu
masyarakat bisa menimbulkan memberagamkan bahan makanan
dampak yang besar. bagi rakyat Indonesia.
(C) pemerintah India belum siap (B) kemiskinan dan kebodohan di
menangani pemproduksian daging Indonesia merupakan salah satu
secara nasional. akibat rendahnya konsumsi
(D) pembunuhan sapi secara besar- protein.
besaran dapat melumpuhkan (C) konsumsi protein rakyat Indonesia
perekonomian India. paling rendah dibandingkan
(E) daging sapi diharapkan dapat dengan Malaysia, Thailand, dan
meningkatkan gizi masyarakat di Jepang.
India. (D) konsumsi karbohidrat rakyat
Indonesia lebih rendah daripada
11. Meskipun hampir 50 tahun kampanye konsumsi karbohidrat orang
“empat sehat lima sempurna”, tingkat Malaysia, Thailand, dan Jepang.
konsumsi protein rakyat masih rendah (E) gizi memiliki hubungan yang
karena terbatasnya pengetahuan dan berbanding lurus dengan kualitas
ketidakberdayaan ekonomi. Menurut sumber daya manusia di
Siswono Yudo Husodo, Indonesia Indonesia.
memiliki sumber karbohidrat dan protein
yang sangat beragam dan banyak yang
dapat dimanfaatkan untuk pangan
rakyat. Diversifikasi pangan sesuai
dengan kekayaan alam perlu menjadi
kebijakan pemerintah.
Ia khawatir karena di Indonesia pada
akhir-akhir ini masih muncul kasus
kelaparan dan penyakit kekurangan gizi.
Konsumsi protein per kapita rakyat
Indonesia setiap tahun terhitung masih
sangat kurang, misalnya konsumsi ikan
bagi rakyat Indonesia hanya 26 kg,
sedangkan Malaysia sudah sebanyak 45
kg, dan Jepang 60 kg. Rakyat Indonesia
mengonsumsi daging ayam sejumlah 3,8
kg, sedangkan Filipina 4 kg. Thailand 16,8
kg dan Malaysia 23 kg. Kualitas pangan
yang dikonsumsi rakyat akan
menentukan tingkat pertumbuhan

Sony Sugema College 117


12. (1) Kemiskinan hampir melanda setiap 13. Dalam krisis 1997 jumlah ringgit di
lapisan masyarakat di tengah berbagai Malaysia berkurang tajam. Untuk itu,
kemajuan ekonomi yang kita raih dan Malaysia berkomitmen tetap akan ...
rasakan sekarang ini. (2) Sejauh ini ada transaksi ringgit di luar negeri hingga
sekitar 16 persen warga kita yang hidup sepuluh tahun ke depan. Kebijakan itu
di bawah kemiskinan. (3) Dalam diambil untuk ... terulangnya krisis nilai
perhitungan angka, jumlahnya berkisar tukar tersebut. “Para pedagang valas
33,5 juta orang. (4) Dengan tidak diizinkan ... ringgit,” kata Wakil
pertumbuhan ekonomi yang tinggi, kita Menkeu Malaysia, Noor Muhammad
akan bisa membuka lapangan kerja. (5) Yackop, di depan parlemen.
Semua itu sangat mudah diucapkan dan Untuk mengisi titik-titik pada teks di
tidak mudah dilaksanakan. (6) Tingkat atas, kata-kata yang tepat adalah ....
pertumbuhan ekonomi yang bisa kita (A) menganjurkan, mencegah,
lakukan adalah pada tingkat sangat menghemat.
moderat, sekitar 4 persen, dan itu tidak (B) mengizinkan, menghindari,
cukup memadai untuk menyerap menjual.
penganggur dan menghapuskan (C) mempersilakan, melarang,
kemiskinan. menghemat.
Pernyataan berikut yang dapat dipakai (D) mencabut, menjaga, membeli.
untuk menyempurnakan teks di atas (E) melarang, mengantisipasi,
adalah .... meminjam.
(A) kata setiap (kalimat 3) dihilangkan
dan kata juta (kalimat 3)
seharusnya ditulis dengan angka.
(B) kata tapi (kalimat 5) diganti
dengan akan tetapi dan kata
rasakan (kalimat 1) diganti dengan
kata kita rasakan.
(C) kata ada (kalimat 2) dihilangkan
dan kata penganggur (kalimat 6)
diganti dengan kata
pengangguran.
(D) kata dan (kalimat 5) diganti
dengan tetapi dan kata adalah
(kalimat 6) diganti berada.
(E) kata ada pada kalimat 2
dihilangkan dan kata rasakan pada
kalimat 6 diganti dengan kata kita
rasakan.

118 Sony Sugema College


Sony Sugema College
14. Berikut ini adalah identitas buku yang dimanfaatkan untuk menulis daftar pustaka.

Judul Buku Penulis Penerbit Tahun Kota


1. Teknologi Rahma Farida Bima 2006 Surabaya
Komunikasi
2. Komunikasi Aris Tata Sapta 2007 Jakarta
Massa

3. Komunikasi Dian Rana Septina 2009 Medan


Sosial
Manakah penulisan daftar pustaka yang tepat untuk buku-buku di atas?
(A) Rahma, Farida. 2006. Teknologi Komunikasi. Surabaya : Bima.
Aris, Tata. 2007. Komunikasi Massa. Jakarta : Sapta.
Dian, Rana. 2009. Komunikasi Sosial. Medan : Septina.
(B) Farida, Rahma. 2006. Teknologi Komunikasi. Surabaya : Bima.
Tata, Aris. 2007. Komunikasi Massa. Jakarta : Sapta.
Rana, Dian. 2009. Komunikasi Sosial. Medan : Septina.
(C) Farida, Rahma. 2006. Teknologi Komunikasi. Surabaya :Bima.
Rana, Dian. 2009. Komunikasi Sosial. Medan : Septina.
Tata, Aris. 2007. Komunikasi Massa. Jakarta : Sapta.
(D) Farida, Rahma. 2006. Teknologi Komunikasi. Surabaya : Bima.
Rana, Dian. 2009. Komunikasi Sosial. Medan : Septina.
Tata, Aris. 2007. Komunikasi Massa. Jakarta : Sapta.
(E) Farida, Rahma. 2006. Teknologi Komunikasi. Surabaya : Bima.
Rana, Dian. 2009. Komunikasi Sosial. Medan : Septina.
Tata, Aris. 2007. Komunikasi Massa. Jakarta : Sapta.

Sony Sugema College 119


15. Pada halaman 25 buku Pengantar Sastra (E) Mahayama mengatakan bahwa
Modern yang ditulis oleh Maman dunia sastra dengan berbagai
Mahayama pada tahun 1998 tertulis kerumitannya mencoba pula
kalimat berikut. menyediakan pemakaian dan
Dunia sastra dengan berbagai kesadaran mengenai situasi dan
kerumitannya mencoba pula berbagai masalah yang dihadapi
menyediakan pemakaian dan kesadaran umat manusia (1998 : 25).
mengenai situasi dan berbagai masalah
yang dihadapi umat manusia.
Manakah penulisan kutipan yang tepat
untuk teks di atas?
(A) “Dunia sastra dengan berbagai
kerumitannya mencoba pula
menyediakan pemakaian dan
kesadaran mengenai situasi dan
berbagai masalah yang dihadapi
umat manusia” (Mahayama, 1998 :
25).
(B) Mahayama (1998 : 25), dunia
sastra dengan berbagai
kerumitannya mencoba pula
menyediakan pemakaian dan
kesadaran mengenai situasi dan
berbagai masalah yang dihadapi
umat manusia.
(C) Menurut Mahayama dalam
bukunya yang diterbitkan tahun
1998, dunia sastra adalah berbagai
kerumitan yang mencoba
menyediakan pemakaian dan
kesadaran mengenai situasi dan
berbagai masalah yang dihadapi
umat manusia.
(D) Dunia sastra dengan berbagai
kerumitannya mencoba pula
menyediakan pemakaian dan
kesadaran mengenai situasi dan
berbagai masalah yang dihadapi
umat manusia (dalam Mahayama,
1998 : 25).

120 Sony Sugema College


Sony Sugema College
SNMPTN Tahun 2009 Kode Soal 383

1. (1) Sekelompok anjing laut utara di Madinah. Penderitaan mereka diduga


(Mirounga angustirostris) sedang akibat makanan yang basi atau tidak
berganti kulit di sebuah pantai di dekat memenuhi standar kesehatan. Insiden
San Simeon, California. (2) Pantai ini seperti ini tidak perlu terjadi seandainya
sudah biasa dijadikan tempat perusahaan katering menjaga kualitas
berkumpulnya sekelompok anjing laut makanan yang disajikan. Kualitas layanan
utara. (3) Di tempat ini pada musim akan terjamin lagi jika tim kesehatan
panas, anjing laut mengalami pergantian selalu mengecek dan mengawasi
kulit besar-besaran yang berlangsung makanan untuk jemaah calon haji.
sekitar satu bulan. (4) Selama sebulan, (Koran Tempo, 21 November 2008)
mereka kehilangan bulu dan kulit. (5) Pernyataan berikut ini merupakan opini
Anjing laut utara menghangatkan badan dari teks di atas, KECUALI ...
mereka sewaktu musim berganti kulit (A) jemaah calon haji Indonesia tidak
dengan cara berkumpul di pantai sambil memperoleh layanan maksimal dari
menunggu tumbuhnya kulit dan bulu Departemen Agama.
yang baru. (B) Departemen Agama tidak bekerja
Kalimat topik paragraf di atas terletak secara profesional dalam
pada nomor ... penyelenggaraan ibadah haji.
(A) (1) (C) manajemen Departemen Agama
(B) (2) lemah sehingga tidak mampu
(C) (3) meningkatkan pelayanan
(D) (4) penyelenggaraan ibadah haji.
(E) (5) (D) tim kesehatan tidak mengecek dan
mengawasi makanan sehingga
2. Departemen Agama seharusnya sanggup jemaah calon haji makan nasi basi.
menyelenggarakan ibadah haji secara (E) koordinasi tidak berjalan baik di
lebih baik. Sesuai dengan undang- antara komponen panitia
undang terbaru, departemen ini tetap penyelenggara ibadah haji di
menjadi regulator sekaligus operator, Departemen Agama.
bahkan ikut menentukan anggota komisi
pengawas. Peningkatan kualitas
pelayanan diharapkan terjadi lewat
pembenahan manajemen. Keluhan mulai
muncul ketika belum ada perbaikan yang
berarti. Lihatlah, baru-baru ini 89
anggota jemaah calon haji dari Jakarta
dilaporkan mengalami diare saat berada

Sony Sugema College 121


3. Saat ini diperlukan realisasi janji dari (C) Kegagalan proyek itu karena
para pemimpin negeri ini, yakni perancangan yang tidak baik dan
peningkatan kualitas pendidikan dengan pengwasan yang kurang cermat.
anggaran hingga 20% dari total APBN (D) Meskipun negara itu merupakan
dan APBD. Sayangnya, baru sekitar 44 penghasil kapas nomor satu dunia,
kabupaten di Indonesia yang tetapi harga tekstilnya sangat tinggi.
mengalokasikan anggaran pendidikan (E) Budiman ingin menjadi juara umum
sesuai dengan ketentuan tersebut. di sekolahnya, tetapi ia hanya
Padahal, peningkatan kualitas berhasil menjadi juara ketiga.
pendidikan ini telah masuk dalam
delapan prioritas pembangunan dalam 5. Exxon Mobil memiliki komitmen untuk
program Presiden Susilo Bambang selalu menangani risiko-risiko yang
Yudhoyono. Di dalam salah satu dari berkaitan dengan keselamatan saat
delapan prioritas itu, disebutkan dengan bekerja. Langkah pertama untuk
jelas bahwa peningkatan akses dan mencapai prestasi kerja tanpa cacat
kualitas pendidikan dan kesehatan adalah dengan memastikan tingkat
menjadi prioritas pembangunan motivasi dan partisipasi karyawan yang
pemerintah Indonesia. Namun, tinggi dalam mencegah terjadinya
realisasinya masih sangat kecelakaan. Di samping itu, karyawan
mengecewakan (Koran Sindo, 21 harus mengecek dan membenahi adanya
November 2008) kerusakan. Exxon Mobil yakin bahwa
Pernyataan berikut yang tidak relevan penciptakan lingkungan kerja yang aman
dengan isi teks di atas adalah ... dapat memberikan kinerja yang baik.
(A) pemerintah menjanjikan anggaran Dengan bantuan teknologi baru, Exxon
pendidikan sebesar 20% dari APBN Mobil terus berupaya membuat kegiatan
dan APBD. operasionalnya lebih aman, sehat, dan
(B) alokasi anggaran pendidikan 20% ramah lingkungan. Di Exxon Mobil,
baru dilakukan oleh 44 kabupaten. kesempurnaan dalam masalah
(C) janji pemerintah menetapkan keselamatan, kesehatan, dan
anggaran pendidikan 20% belum perlindungan lingkungan merupakan
ditepati. unsur inti yang diutamakan. (Majalah
(D) pendidikan dan kesehatan Tempo, 17-23 November 2008)
merupakan prioritas utama program Simpulan teks tersebut yang paling tepat
pemerintah. adalah ...
(E) realisasi dari delapan prioritas (A) Exxon Mobil sangat peduli terhadap
pembangunan sangat keselamatan kerja karyawan.
mengecewakan. (B) Exxon Mobil menetapkan dua
4. Di bawah ini, manakah yang termasuk langkah untuk mencapai prestasi
kalimat efektif? kerja.
(A) Dalam musyawarah itu (C) lingkungan kerja yang aman akan
menghasilkan lima ketetapan yang menghasilkan kinerja yang baik.
harus dilaksanakan bersama. (D) teknologi baru dapat membuat
(B) Dalam pertemuan itu memutuskan kegiatan di Exxon Mobil lebih aman.
bahwa Andi yang ditunjuk menjadi (E) motivasi dan partisipasi karyawan
ketua Koperasi. Exxon Mobil harus tinggi.
122 Sony Sugema College
Sony Sugema College
6. Blog awalnya adalah catatan harian yang 7. Seseorang akan mengembangkan
ditempelkan (posting) ke sebuah situs karangan yang bertemu upaya
yang dapat diakses siapa saja. Blog mengatasi pergaulan bebas pada kaum
menjadi semakin populer ketika remaja di perkotaan.
kemudian tersedia situs yang Berdasarkan tema tersebut, topik-topik
menyediakan diri sebagai rumah gratis untuk kerangka karangan yang runtut
bagi blog, seperti blogspot dan adalah ...
wordpress. Apabila di dunia international (A) masalah yang timbul akibat
blog mulai dikenal tahun 1998, di pergaulan bebas pada kaum remaja
Indonesia, kata Wicaksono, muncul di perkotaan, pentingnya mengatasi
tahun 2004 dan meningkat tajam tahun pergaulan bebas, langkah mengatasi
2007. “Uniknya, di Indonesia paling- pergaulan bebas, saran yang dapat
paling hanya 5 persen yang bicara dilakukan untuk mengatasi
politik. Mengherankan juga karena pergaulan bebas.
sekarang ruang untuk berpendapat jauh (B) pentingnya mengatasi pergaulan
lebih bebas,” kata Wicaksono. Lainnya bebas pada kaum remaja di
berisi mulai dari kuliner, wisata, atau perkotaan, masalah yang timbul
perjalanan, jual-beli, hingga prosa dan akibat pergaulan bebas, langkah
puisi. Beberapa blog begitu populernya mengatasi pergaulan bebas, saran
sehingga dapat membangkitkan nilai yang dapat dilakukan untuk
ekonomi. (Kompas, 23 November 2008) mengatasi pergaulan bebas.
Blog di tanah air menjadi salah satu cara (C) langkah mengatasi pergaulan bebas
berkomunikasi di dunia maya yang pada kaum remaja di perkotaan,
populer karena ... pentingnya mengatasi pergaulan
(A) ada banyak blog dibuat orang bebas, masalah yang timbul akibat
Indonesai sehingga tidak harus pergaulan bebas, saran yang dapat
berkomunikasi dalam bahasa dilakukan untuk mengatasi
Inggris. pergaulan bebas.
(B) ada situs yang menyediakan diri (D) pentingnya mengatasi pergaulan
sebagai rumah gratis untuk blog bebas pada kaum remaja di
sehingga pengguna tidak perlu perkotaan, langkah mengatasi
mengeluarkan biaya. pergaulan bebas, masalah yang
(C) pengguna blog akan dipandang timbul akibat pergaulan bebas,
sebagai orang modern karena dapat saran yang dapat dilakukan untuk
berkomunikasi dengan peralatan mengatasi pergaulan bebas.
komunikasi modern. (E) pentingnya saran yang dapat
(D) pengguna blog dapat berbicara dilakukan untuk mengatasi
politik dan lain-lain secara bebas pergaulan bebas di perkotaan,
karena sulit diidentifikasi jati dirinya. masalah yang timbul akibat
(E) siapa saja dapat bergabung di sana pergaulan bebas, langkah mengatasi
dan apa saja dapat disampaikan pergaulan bebas, saran yang dapat
meskipun ada berbagai masalah. dilakukan untuk mengatasi
pergaulan bebas.

Sony Sugema College 123


8. ... Salah satu penyakit kulit adalah acne Paragraf di atas dikembangkan dengan
vulgaris. Tidak ada seorang pun di dunia menggunakan pola sebab-akibat karena ...
ini yang tidak pernah menderita penyakit (A) kalimat utamanya menyatakan
ini. Acne vulgaris adalah penyakit hubungan sebab dan akibat.
peradangan folikel sebasea yang (B) kalimat pertama menyatakan sebab
umumnya terjadi pada masa remaja dan dan kalimat lainnya menyatakan
dapat sembuh sendiri. Dalam akibat.
masyarakat umum, acne vulgaris biasa (C) kalimat ketiga menyatakan sebab,
dikenal dengan istilah jerawat. sedangkan kalimat lainnya
menyatakan akibat.
Kalimat manakah yang tepat untuk (D) kalimat pertama dan ketujuh
mengisi titik-titik di atas? menyatakan akibat, sedangkan
(A) Acne vulgaris adalah penyakit kulit lainnya menyatakan sebab.
biasa. (E) kalimat ketujuh menyatakan akibat,
(B) Acne vulgaris pernah dialami semua sedangkan kalimat lainnya
manusia. menyatakan sebab.
(C) Penyakit kulit banyak jenis dan
macamnya. 10. Tokoh-tokoh ternama, seperti Charlie
(D) Jerawat banyak jenis dan Chaplin, Jawaharlal Nehru, Gamal Abdul
macamnya. Naser, dan Bung Karno, pernah
(E) Penyakit yang diderita remaja menginap di sini. Gaya seni Art Deconya
bermacam-macam. yang unik seolah menjadi trademark
hotel yang satu ini. Cobalah menyusuri
9. Pasar uang dan pasar modal Indonesia Jalan Asia Afrika di Bandung. Di sana ada
belum pulih. Rupiah masih terus gedung yang memiliki lengkung
mengalami tekanan mengikuti bangunan unik berwarna keabuan.
penurunan sebagian besar mata uang Keunikan tersebut seolah-olah tak
Asia. Masalah minimnya pasokan dolar terpisahkan dari sejarah Kota Kembang
AS di pasar valas semakin mempersulit yang pernah kental sekali aura Eropanya.
mata uang lokal ini. Pada penutupan Hotel yang kini disebut Savoy Homann
perdagangan valas tanggal 24 Bidakara Hotel ini selalu menjadi
Novembner 2008, rupiah melemah kebanggaan ‘urang Bandung’.
hingga 320 poin ke posisi 12.320 per Selanjutnya, di bawah pengelolaan Van
dolar AS. Rupiah bahkan sempat Es, bangunan hotel Homann yang semula
menembus 12.325 per dolar AS. Pelaku sempit dan sederhana diperluas serta
pasar cemas karena belum ada sentimen dimodernisasi menjadi salah satu hotel
positif dari dalam negeri. Oleh karena terkemuka di Asia Tenggara. Gaya seni
itu, pasar lebih memilih memegang dolar Art Deco yang melanda daratan Eropa
AS dalam kondisi pasar global yang pada tahun 1920-an ikut mewarnai
sedang rentan ini. bangunan Homann lama, dan hiasan
interior, jendela kaca patri, ornamen
dinding, model meubel, sampai kap
lampu, semuanya berbau Art Deco.
(Koran Tempo, 21 November 2008)

124 Sony Sugema College


Sony Sugema College
Makna tersirat yang terdapat pada (B) jaringan internet antarplanet secara
kutipan teks tersebut adalah ... otomatis menyebarkan informasi.
(A) setiap kota memiliki bangunan hotel (C) jaringan internet antarplanet tidak
kebanggaan warganya sebagaimana memerlukan bantuan tenaga
warga Kota Bandung memiliki hotel manusia.
Savoy Homann. (D) wahana antariksa selalu melakukan
(B) Hotel Savoy Homann menjadi komunikasi langsung dengan bumi.
kebanggaan warga Kota Bandung (E) sistem baru internet antarplanet
karena megah, serta pernah NASA bernama Spirit dan
ditempati pemimpin dan seniman Opportunity.
besar dunia.
(C) bangunan hotel menjadi indah jika 12. Baru-baru ini sebuah lembaga daripada
mengadopsi model bangunan Eropa survei pendidikan tinggi di seluruh dunia
sebagaimana model bangunan Hotel tidak berapa lama ini telah
Savoy Homann di Kota Bandung. mengeluarkan daftar peringkat tentang
(D) hotel menjadi kebanggaan warga perguruan tinggi yang berkualitas di
jika memiliki sejarah panjang seluruh dunia.
sebagaimana Hotel Savoy Homann
yang ditempati tokoh terkenal. Perbaikan terhadap kalimat di atas
(E) Hotel Savoy Homann Bidakara yang adalah ...
berada di Kota Bandung bergaya Art (A) Tidak berapa lama ini sebuah
Deco Eropa karena berbentuk lembaga survei pendidikan tinggi
lengkung. telah mengeluarkan daftar peringkat
tentang perguruan tinggi yang
11. NASA selesai menguji coba sebuah berkualitas di seluruh dunia.
sistem yang baru pertama kali (B) Baru-baru ini sebuah lembaga dari
diluncurkan, yaitu sebuah sistem survei pendidikan tinggi telah
‘internet antarplanet’. Jaringan ini akan mengeluarkan daftar peringkat
dapat dipergunakan secara otomatis perguruan tinggi berkualitas di
untuk menyebarluaskan informasi ke seluruh dunia.
bumi, wahana antariksa, dan para (C) Tidak berapa lama ini sebuah
astronot. Wahana antariksa biasanya lembaga daripada survei pendidikan
melakukan komunikasi langsung dengan tinggi baru saja mengeluarkan daftar
bumi untuk memberikan instruksi peringkat perguruan tinggi yang
kepada rover-rover Mars dalam misi berkualitas di seluruh dunia.
eksplorasinya. Spirit dan opportunity (D) Baru-baru ini sebuah lembaga survei
mentrasmisikan data ke orbiter lalu pendidikan tinggi mengeluarkan
diteruskan kembali ke bumi. (Media daftar peringkat perguruan tinggi
Indonesia, 23 November 2004) berkualitas di seluruh dunia.
Gagasan utama paragraf di atas adalah (E) Baru-baru ini sebuah lembaga survei
... pendidikan tinggi telah
(A) NASA selesai menguji coba sistem mengeluarkan daftar peringkat
komunikasi baru internet tentang perguruan tinggi berkualitas
antarplanet. di seluruh dunia.

Sony Sugema College 125


13. Keanekaragaman hayati dan
pemandangan ... bawah laut di perairan
Pulau Lemukutan, yang menjadi bagian
dari Kawasan Konservasi Laut Daerah
(KKLD) Bengkayang, dalam setengah
tahun terakhir makin ... wisatawan. KKLD
Bengkayang berada di sekitar 35
kilometer sebelah barat Pulau
Kalimantan di pesisir pantai Bengkayang.
Di situ terdapat ... pulau seperti Pulau
Lemukutan, Randayan, Penata Besar,
Penata Kecil, Baru, dan Kabung.
Untuk mengisi titik-titik pada teks di
atas, kata-kata yang tepat adalah ...
(A) pesona, disenangi, kelompok.
(B) indah, dikenal, bentangan.
(C) elok, diketahui, banyak.
(D) cantik, dikunjungi, deretan.
(E) alam, diminati, gugusan.

14. Berikut ini adalah identitas buku yang digunakan sebagai sumber pustaka.
Judul Buku Penulis Penerbit Kota Tahun
1. Horizon Yusuf Anas Al-Huda Jakarta 2006
Manusia
2. Kalimat Ida Bagus Rapika Bandung 2007
Efektif Putrayasa Aditama
3. Retorika Jalaludin Rosdakarya Bandung 2006
Modern Rakhmat
Manakah penulisan daftar pustaka yang tepat untuk buku-buku di atas?
(A) Anas, Yusuf, 2006, Horizon Manusia, Jakarta; Al-Huda.
Putrayasa, Ida Bagus, 2007, Kalimat Efektif, Bandung; Rapika Aditama.
Rakhmat, Jalaludin, 2006, Retorika Modern, Bandung; Rosdakarya.
(B) Anas, Yusuf. 2006. Horizon Manusia. Jakarta, Al-Huda.
Putrayasa, Ida Bagus. 2007. Kalimat Efektif. Bandung, Rapika Aditama.
Rakhmat, Jalaludin. 2006. Retorika Modern. Bandung, Rosdakarya.
(C) Anas, Yusuf. 2006. Horizon Manusia. Jakarta: Al-Huda.
Putrayasa, Ida Bagus. 2007. Kalimat Efektif. Bandung: Rapika Aditama.
Rakhmat, Jalaludin. 2006. Retorika Modern. Bandung: Rosdakarya.
(D) Anas, Yusuf. 2006. Horizon Manusia. Jakarta: Al-Huda.
Putrayasa, Ida Bagus. 2007. Kalimat Efektif. Bandung: Rapika Aditama.
Rakhmat, Jalaludin. 2006. Retorika Modern. Bandung: Rosdakarya.
(E) Anas, Yusuf. 2006. “Horizon Manusia”. Jakarta: Al-Huda.
Putrayasa, Ida Bagus. 2007. “Kalimat Efektif”. Bandung: Rapika Aditama.
Rakhmat, Jalaludin. 2006. “Retorika Modern”. Bandung: Rosdakarya.

126 Sony Sugema College


Sony Sugema College
15. Pembelajaran berbasis teknologi
komputer perlu segara disosialisasikan
sampai ke sekolah-sekolah di pelosok
tanah air.
Pernyataan tersebut terdapat dalam
buku “Pembelajaran Berbasis TIK”
halaman 32 karya Rahman Arif yang
diterbitkan oleh Penerbit Fantasi pada
tahun 2009.
Teknik pengutipan teks di atas yang
betul adalah ...
(A) Menurut Arif (2009:32) adalah
“Pembelajaran berbasis teknologi
komputer perlu segera
disosialisasikan sampai dengan
sekolah-sekolah pelosok tanah air.”
(B) “Pembelajaran berbasis teknologi
komputer perlu segera
disosialisasikan sampai dengan
sekolah-sekolah pelosok tanah air”
(Arif, 2009:32).
(C) Arif (2009:32) “pembelajaran
berbasis teknologi komputer perlu
segera disosialisasikan ke sekolah-
sekolah di pelosok tanah air.”
(D) Menurut Arif (2009:32) bahwa
pembelajatan berbasis teknologi
komputer perlu segera
disosialisasikan ke sekolah-sekolah
di pelosok tanah air.
(E) “PembelaJaran berbasis teknologi
komputer perlu segera
disosialisasikan ke sekolah-sekolah
di pelosok tanah air” (dalam Arif,
2009:32).

Sony Sugema College 127


SNMPTN Tahun 2010 Kode Soal 326

1. Sebuah panel beranggotakan 15 pakar (E) Dirjen WHO menetapkan bahwa


influenza dari Komite Darurat Organisasi tidak tepat untuk membuat
Kesehatan Dunia menetapkan bahwa berbagai perubahan pada saat
terlalu dini untuk mengatakan pandemi situasi pandemi virus H1N1.
virus H1N1 melewati puncaknya. Setelah
berdiskusi, panel melaporkan kepada 2. Konferensi perubahan iklim internasional
Direktur Jenderal WHO, Margaret Chan, (COP) ke-15 di Denmark berbeda dengan
bahwa virus H1N1 tidak beredar luas di COP sebelumnya. Pada kali ini, COP
berbagai belahan dunia. Akan tetapi, dihiasi oleh wajah-wajah muda. Pada
ditemukan transmisi baru di Afrika Barat COP 15 ini, generasi muda duduk sejajar
dan belum dapat diprediksi cara virus dengan para pemimpin dunia. Mereka
tersebut akan menyebar ketika belahan mendapat kesempatan berbicara di
bumi selatan memasuki musim dingin. depan podium. Pada COP ini, Indonesia
“Berdasarkan bukti-bukti yang memberi kesempatan emas kepada anak
dipresentasikan, Direktur Jenderal muda untuk menjadi delegasi bayangan,
memutuskan bahwa tidak tepat untuk mengikuti perundingan delegasi
membuat perubahan apapun dalam fase Indonesia. Akan tetapi, delegasi muda itu
pandemi saat ini,” ujar Keiji Fukada, ahli belum diperbolehkan berbicara di depan
influenza WHO. forum dan waktu bicara mereka hanya
Pernyataan berikut yang tidak sesuai sedikit. Sebab, tugas utama delegasi
dengan isi teks di atas adalah ... muda adalah menjadi observer selama
(A) Pakar influenza masih belum jelas konferensi berlangsung.
bagaimana virus H1N1 akan Pernyataan berikut yang sesuai dengan
menyebar ketika musim dingin di isi teks di atas adalah ...
belahan bumi selatan. (A) Delegasi dari beberapa negara
(B) Keiji Fukuda memutuskan bahwa pada pertemuan COP 15 di
tidak tepat untuk membuat Denmark diikuti oleh tim bayangan
perubahan apapun dalam fase yang terdiri atas anak-anak muda.
pandemi saat ini. (B) Tim bayangan diperlakukan sama
(C) Ketika virus H1N1 tidak beredar dengan delegasi resmi untuk
secara luas di berbagai belahan mendapat kesempatan berbicara
dunia, ada transmisi baru di Afrika di podium.
Barat. (C) Forum COP 15 di Denmark
(D) Komite Darurat Organisasi memberi kesempatan kepada
Kesehatan Dunia belum dapat kaum muda untuk berbuat dan
memastikan berlalunya puncak menjadi teladan dalam
masa pandemi virus H1N1. menghadapi perubahan iklim
dunia.

128 Sony Sugema College


Sony Sugema College
(D) Tim bayangan memiliki tugas 4. Kegiatan mendonorkan darah sebaiknya
utama sebagai pengamat dan dijadikan gaya hidup. Para mahasiswa
memiliki kesempatan berbicara jangan hanya bangga dengan aksi
yang sama di depan forum. demonstrasi mereka. Akan lebih
(E) Indonesia berencana mengirimkan bijaksana jika aksi demonstrasi itu
tim bayangan dalam delegasi diganti dengan aksi donor darah.
pemerintah di dalam forum COP “Bersedekah dengan darah adalah amal
15 di Denmark. ibadah,” kata H.M. Jusuf Kalla. Jika donor
darah menjadi gaya hidup, ada peluang
3. Di antara sebagian kecil bintang olahraga peningkatan persediaan darah.
Indonesia yang berkelas dunia, Taufik Penyediaan darah oleh PMI baru
Hidayat termasuk yang paling bersinar. mencapai 0,7% dari jumlah penduduk
Ia ... banyak prestasi tingkat dunia pada (1,7 juta kantung pada tahun 2008). Saat
olahraga bulutangkis. Saat Taufik masih ini, persediaan darah nasional baru
berusia 15 tahun, ia ... Pusat Pelatihan mencukupi kebutuhan untuk dua hari.
Nasional (Pelatnas) Bulutangkis di Persediaan itu masih jauh dari target
Cipayung. Sementara itu, ia sedang ... WHO, yaitu 2% dari jumlah penduduk
masa remaja bersama teman-temannya. atau 4 juta kantung per tahun. Jumlah
Setiap hari kegiatannya ... dengan itu (2%), membuat persediaan darah
berlatih dan terus berlatih. Di Pelatnas, nasional akan mencukupi kebutuhan
dia ... untuk mengikuti kejuaraan untuk 4 hari. Jika ada 10.000 orang yang
bulutangkis tingkat dunia. Setahun mendonorkan darah 2 kali setahun, akan
kemudian, Taufik baru memperlihatkan didapat 20.000 kantung darah per tahun.
prestasinya. Ia menjadi juara Brunai Jumlah itu amat berarti dan akan banyak
Terbuka tahun 1998 untuk kategori nyawa orang yang terselamatkan.
tunggal putra. Ide pokok paragraf di atas adalah ...
Urutan kata yang tepat untuk (A) Persediaan darah PMI saat ini
melengkapi teks di atas adalah ... hanya mencukupi untuk
(A) menerima, dipanggil, menikmati, kebutuhan dua hari.
dilakukan, diminta. (B) Aksi donor darah dapat
(B) menorehkan, dipanggil, berdampak pada peningkatan
menikmati, dilakukan, persediaan darah PMI.
dipersiapkan. (C) Untuk meningkatkan persediaan
(C) menerima, diutus, mengalami, darah, donor darah harus menjadi
diisi, disiapkan. gaya hidup.
(D) menorehkan, dipanggil, (D) Saat ini persediaan darah di Palang
menikmati, didominasi, Merah Indonesia masih jauh dari
dipersiapkan. target WHO.
(E) memperoleh, diutus, mengalami, (E) Para mahasiswa diharapkan
didominasi, disiapkan. mengganti aksi demonstrasi
mereka dengan aksi donor darah.

Sony Sugema College 129


5. Kehadian puluhan museum di 6. (1) Padi merupakan tanaman yang dapat
Yogyakarta menjadi magnet pariwisata. tumbuh hampir di seluruh pelosok
Museum-museum itu mengukuhkan Indonesia. (2) Meskipun memerlukan air
gelar Yogyakarta sebagai Kota Bersejarah yang cukup banyak, tanaman padi tidak
di Indonesia. Yogyakarta merupakan bergantung pada musim. (3) Padi dapat
salah satu dari 88 kota bersejarah dunia, tumbuh subur di daerah yang bercurah
seperti Kyoto, Paris dan London. hujan rendah dan tinggi. (4) Kita dapat
Setidaknya, ada 37 museum di menemukan tanaman padi mulai dari
Yogyakarta atau 15% dari jumlah provinsi paling barat, Nangru Aceh
museum di Indonesia (272 buah). Darussalam, sampai dengan provinsi
Sementara itu, luas Yogyakarta hanya paling timur. (5) Produsen padi
sekitar 0,5 persen dari luas Nusantara. terbanyak terdapat di Kalimantan. (6)
Rangkuman teks di atas yang tepat Meskipun Nusa Tenggara Timur memiliki
adalah ... curah hujan rendah, tanaman padi masih
(A) Museum-museum di Yogyakarta ditemukan di sana.
menyimpan unsur-unsur sejarah Kalimat utama paragraf di atas adalah ...
karena itu menarik perhatian (A) kalimat 1.
wisatawan. (B) kalimat 2.
(B) Museum-museum di Yogyakarta (C) kalimat 3.
menyimpan sejarah peradaban (D) kalimat 4.
suatu bangsa Indonesia sehingga (E) kalimat 6.
dapat dilihat dan dipelajari.
(C) Yogyakarta merupakan kota 7. Bacalah tabel berikut dengan cermat!
bersejarah yang memiliki jumlah Tabel Produksi Pisang Provinsi Andalan
museum terbanyak di Indonesia. Tahun Daerah A Daerah B Daerah C Daerah D
(D) Yogyakarta merupakan salah satu 2006 125.000 125.000 135.000 90.000
2007 130.000 125.000 145.000 99.000
di antara 88 kota besar di dunia 2008 135.000 136.000 155.000 110.000
yang bergelar Kota Bersejarah.
(E) Lima belas persen dari jumlah Pernyataan berikut yang sesuai dengan
museum di Indonesia yang isi tabel di atas adalah ...
berjumlah 272 buah terdapat di (A) persentase kenaikan produksi
Yogyakarta. pisang daerah D paling tinggi.
(B) persentase kenaikan produksi
pisang daerah B paling rendah.
(C) persentase kenaikan produksi
pisang daerah D paling rendah.
(D) persentase kenaikan produksi
pisang daerah C paling tinggi.
(E) persentase pisang setiap daerah
selalu meningkat setiap tahun.

130 Sony Sugema College


Sony Sugema College
8. Bacalah tabel berikut dengan cermat! Perbaikan ejaan kalimat di atas yang
Diagram Perkembangan Jumlah Produksi tepat adalah ...
Rempah-Rempah di Kecamatan Sapta (A) Di dalam memecahkan masalah
Pesona (dalam Ton) kependudukan, para pakar
40 kependudukan tidak cukup
35 memberi solution dengan cara
30 memberi saran ke pada
25 Lada Pemerintah, tetapi mereka perlu
20 Pala terjun ke lapangan menangani
15
Cengkeh masalah secara langsung.
10 (B) Di dalam memecahkan masalah
5
kependudukan, para pakar
0
2006 2007 2008 2009 kependudukan tidak cukup
memberi solution dengan cara
Pernyataan berikut yang sesuai dengan memberi saran kepada
isi diagram di atas adalah dari tahun ke Pemerintah, tetapi mereka perlu
tahun ... terjun ke lapangan menangani
(A) petani cengkeh hendaknya dibina masalah secara langsung.
lebih intensif karena jumlah (C) Didalam memecahkan masalah
produksi cengkeh paling rendah. kependudukan, para pakar
(B) perlu pembinaan kepada para kependudukan tidak cukup
petani pala karena jumlah memberi “solution” dengan cara
produksi pala semakin merosot. memberi saran kepada
(C) perlu peningkatan penyuluhan pemerintah, tetapi mereka perlu
pertanian bagi petani lada karena terjun ke lapangan menangani
jumlah produksi lada merosot. masalah secara langsung.
(D) perlu pengintensifan kembali (D) Di dalam memecahkan masalah
program pertanian karena jumlah kependudukan, para pakar
produksi rempah-rempah kependudukan tidak cukup
terbanyak hanya terjadi pada memberi “solution” dengan cara
2006. memberi saran kepada
(E) perlu pembinaan kepada para Pemerintah, tetapi mereka perlu
petani pala karena jumlah terjun ke lapangan menangani
produksi pala cenderung menurun. masalah secara langsung.
(E) Di dalam memecahkan masalah
9. Didalam memecahkan masalah kependudukan, para pakar
kependudukan, para pakar kependudukan tidak cukup
kependudukan tidak cukup memberi memberi solution dengan cara
solution dengan cara memberi saran ke memberi saran kepada
pada Pemerintah tetapi mereka perlu pemerintah tetapi mereka perlu
terjun ke lapangan menangani masalah terjun ke lapangan menangani
secara langsung. masalah secara langsung.

Sony Sugema College 131


10. Dalam buku Mendidik Anak Cerdas karya 11. Professor Alan Maryon-Davis, presiden
Sihadono yang diterbitkan tahun 2009 dari Fakultas Kesehatan Masyarakat
pada halaman 37 terdapat teks sebagai Universitas Oxford, mengatakan
berikut. kebanyakan orang pada tahap awal
Mendidik anak sebenarnya dapat obesitas tidak menyadari bahaya
dilakukan orang tua sejak anak masih kelebihan berat badan yang dapat terus
dalam kandungan. Anak yang masih mengalami peningkatan.
dalam kandungan pada hakikatnya Kalimat tidak efektif di atas dapat diperbaiki
sudah dapat merespon segala sesuatu dengan cara berikut, kecuali ...
yang berada di sekitar ibunya (Ahmad, (A) kata presiden dituliskan dengan p
2008:234). kapital.
Jika Lilis mengutip pendapat Ahmad yang (B) kata dari dihilangkan.
dikutip dalam buku Sihadono di atas, (C) setelah Oxford ditambahkan tanda
penulisan kutipan yang benar adalah ... koma.
(A) Menurut Sihadono dalam bukunya (D) tanda koma setelah kata Davis
yang berjudul Mendidik Anak dihilangkan.
Cerdas menyatakan bahwa (E) kata bahwa ditambahkan setelah
mendidik anak sebenarnya dapat kata mengatakan.
dilakukan sejak dalam kandungan
karena anak sudah dapat 12. Teknologi dibuat atas dasar Ilmu
merespon yang berada di sekitar Pengetahuan dengan tujuan untuk
ibunya (Ahmad, 2008:234). memudahkan pekerjaan manusia. Jika
(B) Sihadono (2008:234) menyatakan pada kenyataannya tekhnologi justru
bahwa mendidik anak dapat bikin sulit, layakkah tekhnologi disebut
dilakukan orang tua sejak anak Ilmu Pengetahuan? Kata tekhnologi
masih dalam kandungan. sering menggambarkan penemuan dan
(C) Menurut Ahmad (dalam Sihadono, alat yang menggunakan prinsip dan
2009), mendidik anak dapat proses penemuan sain yang baru
dilakukan orang tua sejak anak ditemukan. Meskipun demikian
masih dalam kandungan. penemuan yang sangat lama seperti
(D) Dalam bukunya Ahmad, Sihadono roda juga disebut sebuah tekhnologi.
(2009) menyatakan bahwa anak Perbaikan paragraf di atas dapat
yang masih dalam kandungan pada dilakukan dengan cara berikut, kecuali ...
hakikatnya sudah dapat merespon (A) kata tekhnologi setelah kata
segala sesuatu yang berada di layakkah dihilangkan.
sekitar ibunya. (B) kata tekhnologi ditulis teknologi.
(E) Sihadono (dalam Ahmad, 2008) (C) kata bikin diganti membuat.
menyatakan bahwa anak yang (D) Ilmu Pengetahuan ditulis ilmu
masih dalam kandungan pada pengetahuan.
hakikatnya sudah dapat merespon (E) tanda koma diletakkan setelah
segala sesuatu yang berada di kata meskipun demikian.
sekitar ibunya.

132 Sony Sugema College


Sony Sugema College
13. Seseorang akan mengembangkan 14. Ibu-ibu hamil merupakan salah satu
karangan ilmiah yang bertema kelompok masyarakat yang sangat
pembinaan generasi muda pedesaan. rawan terhadap berbagai masalah gizi,
Berdasarkan tema tersebut, kerangka isi terutama masalah kekurangan energi
karangan yang runtut adalah ... dan protein (KEP). Bayi yang dilahirkan
(A) pentingnya pembinaan generasi oleh ibu dengan kondisi KEP akan
muda pedesaan, bentuk mempunyai berat badan lahir rendah
pembinaan generasi muda (BBLR), yaitu kurang dari 2,5 kg. Kondisi
pedesaan, sasaran pembinaan BBLR sangat berpengaruh terhadap
generasi muda pedesaan, langkah perkembangan kesehatan anak
pembinaan generasi muda selanjutnya. Bayi BBLR mempunyai
pedesaan. kemungkinan meninggal sebelum usia
(B) sasaran pembinaan generasi muda satu tahun 17 kali lebih besar
pedesaan, langkah pembinaan dibandingkan anak normal.
generasi muda pedesaan, bentuk Agar menjadi paragraf yang baik, kalimat
pembinaan generasi muda penutup yang sesuai adalah ...
pedesaan, pentingnya pembinaan (A) Dengan demikian, kondisi BBLR
generasi muda pedesaan. mengakibatkan terjadinya
(C) sasaran pembinaan generasi muda penurunan kemampuan belajar
pedesaan, pentingnya pembinaan anak selanjutnya.
generasi muda pedesaan, langkah (B) Di samping itu, kondisi BBLR dapat
pembinaan generasi muda berakibat pada penurunan
pedesaan, bentuk pembinaan kemampuan belajar anak.
generasi muda pedesaan. (C) Jadi, kondisi BBLR mengakibatkan
(D) pentingnya pembinaan generasi penurunan kemampuan belajar.
muda pedesaan, sasaran (D) Sebab, kondisi BBLR
pembinaan generasi muda mengakibatkan terjadinya
pedesaan, bentuk pembinaan penurunan kemampuan belajar
generasi muda pedesaan, langkah selanjutnya.
pembinaan generasi muda (E) Oleh sebab itu, kondisi BBLR
pedesaan. mengakibatkan terjadinya
(E) bentuk pembinaan generasi muda penurunan kemampuan belajar
pedesaan, sasaran pembinaan selanjutnya.
generasi muda pedesaan, langkah
pembinaan generasi muda
pedesaan, pentingnya pembinaan
generasi muda pedesaan.

Sony Sugema College 133


15. Berikut ini adalah identitas buku yang digunakan sebagai sumber untuk menulis karangan.
Judul Penulis Kota Penerbit Tahun
Mengentas Anak Putus Rahman P. Jayapura Bintang 2008
Sekolah
Berbagai Teknologi P. Puspitasari Medan Cahaya 2007
Tepat Guna
Perkembangan Mental Andy Sugentar Ambon Cemerlang 2008
Masa Remaja
Membina Keterampilan Dr. Suaendah Jayapura Gemerlap 2009
Remaja Rohe
Berdasarkan data pustaka di atas, penulisan daftar pustaka yang relevan dan tepat untuk
karangan yang berjudul Pembinaan Remaja Putus Sekolah adalah ...
(A) Rahman, P. 2008. Mengentas Anak Putus Sekolah. Jayapura: Bintang.
Rohe, S. 2009. Membina Keterampilan Remaja. Jayapura: Gemerlap.
Sugentar, A. 2008. Perkembangan Mental Masa Remaja. Ambon: Cemerlang.
(B) Puspitasari, P. 2007. Berbagai Teknologi Tepat Guna. Medan: Cahaya.
Rahman P. 2008. Mengentas Anak Putus Sekolah. Jayapura: Bintang.
Rohe, S. 2009. Membina Keterampilan Remaja. Jayapura: Gemerlap.
Sugentar, Andy. 2008. Perkembangan Mental Masa Remaja. Ambon: Cemerlang.
(C) Puspitasari, P. 2007. Berbagai Teknologi Tepat Guna. Medan: Cahaya.
Rahman P. 2008. Mengentas Anak Putus Sekolah. Jayapura: Bintang.
Rohe, Suaendah. 2009. Membina Keterampilan Remaja. Jayapura: Gemerlap.
(D) Andy Sugentar. 2008. Perkembangan Mental Masa Remaja. Ambon: Cemerlang.
P.Puspitasari. 2007. Berbagai Teknologi Tepat Guna. Medan: Cahaya.
Rahman P. 2008. Mengentas Anak Putus Sekolah. Jayapura: Bintang.
Suaendah Rohe. 2009. Membina Keterampilan Remaja. Jayapura: Gemerlap.
(E) Rahman P. 2008. Mengentas Anak Putus Sekolah. Jayapura: Bintang.
Rohe, Suaendah. 2009. Membina Keterampilan Remaja. Jayapura: Gemerlap.
Sugentar, Andy. 2008. Perkembangan Mental Masa Remaja. Ambon: Cemerlang.

134 Sony Sugema College


Sony Sugema College
SNMPTN Tahun 2010 Kode Soal 336

1. Fisika adalah sains atau ilmu tentang Humanity. “Sertifikat itu mengakui
alam dalam arti luas. Fisika ... gejala alam wayang Indonesia, batik Indonesia, dan
yang tidak hidup atau materi dalam keris Indonesia sebagai warisan budaya
lingkup ruang dan waktu. Fisikawan dunia dan kita harus bangga karenanya,”
mempelajari ... dan sifat materi dalam ujar Menko Kesra Agung Laksono dalam
bidang yang sangat beragam, mulai dari serah terima sertifikat yang dilakukan di
partikel submikroskopis yang ... segala kantornya, Jalan Merdeka Barat Jakarta,
materi hingga perilaku materi alam tanggal 5 Februari 2010.
semesta sebagai satu kesatuan kosmos. Pernyataan berikut yang sesuai dengan
Beberapa sifat yang ... dalam fisika isi teks di atas adalah ...
merupakan sifat yang ada dalam semua (A) Wayang, batik, dan keris yang
sistem materi yang ada, seperti hukum telah diakui UNESCO sebagai
kekekalan energi. Sifat semacam ini ... warisan kebudayaan dunia hanya
hukum fisika. ada di Indonesia.
Urutan kata yang tepat untuk (B) Sertifikat UNESCO atas warisan
melengkapi teks di atas adalah ... kebudayaan Indonesia merupakan
(A) mempelajari, perilaku, pengakuan atas peradaban bangsa
membentuk, dipelajari, disebut. Indonesia dalam kancah
(B) menguraikan, kegiatan, kebudayaan dunia.
membentuk, dinyatakan, diulas. (C) Sertifikat UNESCO tentang
(C) mempelajari, aktivitas, pengakuan warisan budaya
menjadikan, diulas, dipelajari. Indonesia sebagai warisan
(D) membahas, sikap, membuat, kebudayaan dunia diserahkan oleh
dibahas, disebut. Menko Kesra.
(E) menjelaskan, perilaku, mengubah, (D) Masyarakat Indonesia bangga
dikatakan, dipelajari. pada wayang, batik, dan keris
sebagai warisan kebudayaannya.
2. Pengakuan dunia internasional terhadap (E) Sertifikat UNESCO diharapkan
warisan budaya nasional semakin menjadi simbol pengakuan
mantap. Pemerintah secara simbolis, warisan budaya Indonesia sebagai
telah menerima tiga sertifikat dari bagian warisan kebudayaan
United Nations Educational, Scientific, Indonesia.
and Cultural Organization (UNESCO).
Sertifikat itu merupakan simbol
pengakuan warisan budaya Indonesia
dalam daftar The Representative List of
the Intangible Culture Heritage of

Sony Sugema College 135


3. Komite Darurat Organisasi Kesehatan 4. (1) Salah satu akibat aturan pasar bebas
Dunia akan meninjau kembali status yang paling mencolok adalah dibatasinya
pandemi H1N1 untuk memutuskan jumlah order (pesanan) oleh grosir dan
apakah dunia telah berpindah ke fase pelanggan di beberapa daerah, seperti
pascapuncak atau belum. Hilangnya Surabaya, Malang, Bali, Kalimantan, Jawa
H1N1 di Kanada disebabkan sekitar 45% Tengah, dan Jakarta. (2) “Pembatasan
warga Kanada telah divaksinasi, 30% order itu kini mencapai 50 persen. (3)
didiagnosis kebal, dan hanya sedikit Mereka (pelanggan dan grosir)
orang kemungkinan tertulari. Hal menyatakan tidak ingin gegabah
tersebut membuat penyebaran virus memesan barang dalam jumlah besar
semakin sulit terjadi. Akan tetapi, virus seperti dulu,” ungkap Emru, perajin
masih menyebar di beberapa bagian sepatu fashion dan mayoret. (4)
Eropa Timur dan Asia Tengah. WHO Menurut Amru, pembatasan jumlah
mengatakan bahwa ada aktivitas virus di order tersebut, sebelumnya, tidak
negara-negara Afrika, seperti Senegal diketahui alasan pastinya. (5) Sebab,
dan Mauritania. Untuk membantu badan sebagian perajin banyak yang belum
internasional yang berbasis di Jenewa mengetahui adanya perdagangan bebas
itu, Kanada menyumbangkan lima juta itu. (6) Jadi, produksi perajin sepatu
dosis vaksin untuk didistribusikan ke menurun drastis, dari 50 kodi menjadi
negara-negara berkembang yang tidak hanya 25 kodi.
mampu menyediakannya. Kalimat utama paragraf di atas adalah ...
Pernyataan berikut yang tidak sesuai (A) kalimat 1.
dengan isi paragraf di atas adalah ... (B) kalimat 2.
(A) Penyebaran virus H1N1 menjadi (C) kalimat 3.
sulit terjadi di Kanada antara lain (D) kalimat 4.
disebabkan 30% penduduk diduga (E) kalimat 6.
kebar virus tersebut.
(B) Hilangnya virus H1N1 di Kanada 5. Belakangan ini berbagai situs jejaring
salah satunya disebabkan vaksinasi sosial, misalnya facebook, twitter, my
yang dilakukan kepada sekitar 45% space, multiply, plurk, Hi5, dan tagged
warga. mewabah di Indonesia. Keberadaan
(C) Kanada menyumbang lima juta situs-situs jejaring sosial itu sempat
dosis vaksin kepada WHO untuk menuai kritik. Situs-situs jejaring sosial
membantu negara-negara tersebut dianggap kurang mendidik dan
berkembang. menyebabkan kecanduan penggunanya.
(D) Penyebaran virus H1N1 saat ini Seringkali karena berbagai hal, orang
masih terjadi di seluruh negara menjadi lupa waktu setelah mengakses
bagian Eropa Timur dan Asia situs jejaring sosial. Hal negatif lainnya
Tengah. adalah pengguna situs jejaring sosial itu
(E) Di negara-negara Afrika, seperti dinilai dapat terasing dari kehidupan
Senegal dan Mauritania, masih ada nyata kesehariannya. Oleh sebab itu,
aktivitas virus H1N1. diperlukan kesadaran dan kontrol diri
yang baik sehingga pengguna dapat
terhindar dari berbagai efek negatif
situs-situs jejaring sosial tersebut.

136 Sony Sugema College


Sony Sugema College
Ide pokok paragraf di atas adalah ... Rangkuman yang tepat untuk paragraf di
(A) Situs jejaring sosial yang sedang atas adalah ...
mewabah di Indonesia sempat (A) Permasalahan e-waste ini
menuai banyak kritik. mendapat perhatian serius dari
(B) Situs jejaring sosial dapat Deputi Kementrian Lingkungan
menyebabkan kecanduan bagi Hidup Bidang Pengelolaan Bahan
para penggunanya. Berbahaya Beracun dan Limbah
(C) Facebook, Twitter, My Space, Bahan Berbahaya dan Beracun.
Multiply, Plurk, dan lainnya (B) Indonesia sebagai negara
mewabah di Indonesia. berkembang sangat rentan
(D) Para pengguna situs jejaring sosial terhadap impor limbah B-3 (bahan
dapat terasing dari kehidupan berbahaya dan beracun),
nyata kesehariannya. khususnya e-waste (electronic
(E) Situs jejaring sosial berdampak waste) dari negara-negara maju.
negatif sehingga diperlukan (C) Negara-negara berkembang,
kontrol diri yang baik. termasuk Indonesia menjadi salah
satu importir terbesar e-waste
6. Sebagai negara berkembang, Indonesia karena tidak mampu mengatur
sangat rentan terhadap impor limbah B- regulasi penanganan pengelolaan
3 (bahan berbahaya dan beracun), limbah B-3.
khususnya e-waste (electronic waste), (D) Indonesia sangat rentan terhadap
seperti telepon seluler dan komputer limbah B-3 karena masih minimnya
dari negara-negara maju. Minimnya pengawasan serta belum adanya
pengawasan di pintu masuk dan belum regulasi khusus terhadap
adanya regulasi khusus tentang penanganan e-waste sehingga
penangan e-waste ini menjadi kendala diperlukan regulasi.
bagi pemerintah dalam menanggulangi (E) Permasalahan pengelolaan bahan
masalah ini. Selama ini, negara-negara berbahaya beracun dan limbah
berkembang, termasuk Indonesia, bahan berbahaya dan beracun di
menjadi salah satu importir terbesar e- Indonesia semakin berkembang
waste karena tidak mampu mengatur sehingga harus ada regulasi khusus
regulasi tersebut. Deputi Kementerian soal itu.
Lingkungan Hidup Bidang Pengelolaan
Bahan Berbahaya Beracun, Imam
Hendargo Abu Ismoyo mengatakan
bahwa permasalahan e-waste semakin
lama semakin rumit. Oleh sebab itu, kita
harus memiliki regulasi khusus soal itu.

Sony Sugema College 137


7. Bacalah tabel berikut dengan cermat!
Tahun Daerah A Daerah B Daerah C Daerah D
2006 275.000 275.000 285.000 240.000
2007 280.000 275.000 295.000 249.000
2008 285.000 286.000 305.000 260.000

Pernyataan berikut yang sesuai dengan


isi tabel di atas adalah ...
(A) poduksi tebu setiap daerah selalu
meningkat setiap tahun.
(B) persentase kenaikan produksi tebu
daerah B paling rendah.
(C) persentase kenaikan produksi tebu
daerah D paling tinggi.
(D) persentase kenaikan produksi tebu
daerah C paling tinggi.
(E) persentase kenaikan produksi tebu
daerah D paling rendah.

8. Menurut para ahli mengatakan


pengambilan data penelitian yang
dilakukan berulang-ulang kali dapat
mempermudah peneliti dalam
melakukan analisis data pasca
pengumpulan data.
Kalimat tidak efektif di atas dapat diperbaiki
dengan cara berikut, kecuali ...
(A) kata menurut dihilangkan.
(B) kata berulang kali diganti dengan
berulang-ulang.
(C) setelah kata ahli ditambahkan
tanda koma.
(D) kata bahwa ditambahkan setelah
kata mengatakan.
(E) pasca pengumpulan dituliskan
serangkai.

138 Sony Sugema College


Sony Sugema College
9. Berikut adalah identitas pustaka yang dapat digunakan sebagai sumber untuk menulis
karangan.
Judul Penulis Kota Penerbit Tahun
Pendidikan Anak Rahmini P. Surabaya Bintang 2008
Jalanan
Memanusiakan Anak P. Puspitasari Medan Cahaya 2007
Manusia
Perdagangan Anak Ahmad Ambon Cemerlang 2008
Sugentar
Bimbing Anak Dr. Siana Rohe Surabaya Gemerlap 2009
Berakhlak Mulia
Azhari menulis daftar pustaka untuk karangannya yang berjudul Pembinaan Anak Jalanan.
Berdasarkan data pustaka di atas, penulisan daftar pustaka yang relevan dan tepat untuk
karangan Azhari tersebut adalah ...
(A) Ahmad Sugentar. 2008. Perdagangan Anak. Ambon: Cemerlang.
P.Puspitasari. 2007. Memanusiakan Anak Manusia. Medan: Cahaya.
Rahmini P. 2008. Pendidikan Anak Jalanan. Surabaya: Bintang.
Siana Rohe. 2009. Bimbing Anak Berakhlak Mulia. Surabaya: Gemerlap.
(B) Puspitasari, P. 2007. Memanusiakan Anak Manusia. Medan: Cahaya.
Rahmini P. 2008. Pendidikan Anak Jalanan. Surabaya: Bintang.
Rohe, S. 2009. Bimbing Anak Berakhlak Mulia. Surabaya: Gemerlap.
Sugentar, Ahmad. 2008. Perdagangan Anak. Ambon: Cemerlang.
(C) Puspitasari, P. 2007. Memanusiakan Anak Manusia. Medan: Cahaya.
Rahmini P. 2008. Pendidikan Anak Jalanan. Surabaya: Bintang.
Rohe, Siana. 2009. Bimbing Anak Berakhlak Mulia. Surabaya: Gemerlap.
(D) Puspitasari, P. 2007. Memanusiakan Anak Manusia. Medan: Cahaya.
Rahmini P. 2008. Pendidikan Anak Jalanan. Surabaya: Bintang.
Rohe, S. 2009. Bimbing Anak Berakhlak Mulia. Surabaya: Gemerlap.
(E) Rahmini P. 2008. Pendidikan Anak Jalanan. Surabaya: Bintang.
Puspitasari, P. 2007. Memanusiakan Anak Manusia. Medan: Cahaya.
Sugentar, Ahmad. 2008. Perdagangan Anak. Ambon: Cemerlang.
Rohe, Siana. 2009. Bimbing Anak Berakhlak Mulia. Surabaya: Gemerlap

Sony Sugema College 139


10. Ujian nasional menjadi perhatian banyak 11. Untuk mengembangkan topik
pihak, baik pemerintah, orang tua, pembinaan keamanan lingkungan RT
maupun masyarakat. Tingginya menjadi tulisan yang runtut, kerangka
perhatian tersebut menunjukkan bahwa karangan yang baik adalah ...
semua pihak berharap agar pelaksanaan (A) sasaran pembinaan keamanan
ujian nasional tahun ini berkualitas. lingkungan RT, pentingnya
Orang tua berharap agar pelaksanaan pembinaan keamanan lingkungan
ujian nasional tahun ini berjalan lancar RT, langkah pembinaan keamanan
sehingga anak-anak mereka dapat lulus. lingkungan RT, bentuk pembinaan
Pemerintah menghendaki agar keamanan lingkungan RT.
pelaksanaan ujian nasional tahun ini (B) pentingnya pembinaan keamanan
didukung penuh oleh semua pihak. lingkungan RT, sasaran pembinaan
Agar menjadi paragraf yang baik, kalimat keamanan lingkungan RT, bentuk
penutup yang sesuai adalah ... pembinaan keamanan lingkungan
(A) Dengan demikian, pemerintah RT, langkah pembinaan keamanan
berharap agar ujian nasional tahun lingkungan RT.
ini jauh lebih berkualitas (C) pentingnya pembinaan keamanan
dibandingkan dengan ujian lingkungan RT, langkah pembinaan
nasional tahun lalu yang penuh keamanan lingkungan RT, bentuk
kecurangan. pembinaan keamanan lingkungan
(B) Oleh karena itu, pemerintah juga RT, sasaran pembinaan keamanan
berharap agar ujian nasional tahun lingkungan RT.
ini jauh lebih berkualitas (D) bentuk pembinaan keamanan
dibandingkan dengan ujian lingkungan RT, sasaran pembinaan
nasional tahun lalu yang penuh keamanan lingkungan RT,
kecurangan. pentingnya pembinaan keamanan
(C) Oleh pemerintah diharapkan agar lingkungan RT, langkah pembinaan
ujian nasional tahun ini jauh lebih keamanan lingkungan RT.
berkualitas dibandingkan dengan (E) Pentingnya pembinaan keamanan
ujian nasional tahun lalu yang lingkungan RT, langkah pembinaan
penuh kecurangan. keamanan lingkungan RT, sasaran
(D) Pemerintah berharap agar ujian pembinaan keamanan lingkungan
nasional tahun ini jauh lebih RT, bentuk pembinaan keamanan
berkualitas dibandingkan dengan lingkungan RT.
ujian nasional tahun lalu karena
penuh kecurangan.
(E) Pemerintah juga berharap agar
ujian nasional tahun ini jauh lebih
berkualitas dibandingkan dengan
ujian nasional tahun lalu yang
penuh kecurangan.

140 Sony Sugema College


Sony Sugema College
12. Baru-baru ini, tim peneliti dari University (D) petani anggur hendaknya dibina
School of Medicine, Washington, berhasil lebih intensif karena jumlah
mengidentifikasi protein plasmodium sp, produksi anggur cenderung
yaitu parasit nyamuk yang menyebabkan menurun.
penyakit malaria. Penemuan ini penting (E) perlu pengintensifan kembali
karena malaria kini sebagai pembunuh program pertanian karena jumlah
nomor tiga di dunia. Penyakit ini tercatat produksi buah-buahan terbanyak
membunuh satu hingga tiga juta orang hanya terjadi pada 2006.
setiap tahun.
Paragraf di atas dapat diperbaiki dengan 14. Dalam buku Jangan Serahkan Anak
cara berikut, kecuali ... kepada Pembantu halaman 11 karya
(A) kata ini diganti itu. Imron Zamani yang terbit tahun 2008
(B) kata tim ditulis Tim. terdapat kutipan berikut.
(C) kata dari dihilangkan. “Anak, pada hakikatnya, titipan Tuhan.
(D) kata plasmodium sp dicetak Sebagai pemegang titipan, tidak
miring. selayaknya orang tua menitipkan lagi
(E) tanda koma setelah Washington sang anak keapda pembantu”
dihilangkan. (Nurudin, 2007)
Jika Nunung mengutip pendapat Nurudin
13. Bacalah diagram berikut dengan teliti! yang dikutip dalam buku Imron Zamani,
Diagram Perkembangan Jumlah Produksi penulisan kutipan yang benar adalah ...
Buah-buahan di Kecamatan Panca Indah (A) Nurudin (dalam Zamani, 2008:11)
(dalam Ton) menyatakan bahwa sebagai
pemegang titipan, tidak
selayaknya orang tua menitipkan
lagi sang anak kepada pembantu.
(B) Nurudin (Imron Zamani, 2008:11)
menyatakan bahwa sebagai
pemegang titipan, tidak
selayaknya orang tua menitipkan
lagi sang anak kepada pembantu.
Pernyataan berikut yang sesuai dengan (C) Nurudin (2007) menyatakan
isi diagram di atas adalah dari tahun ke bahwa sebagai pemegang titipan,
tahun ... tidak selayaknya orang tua
(A) perlu peningkatan penyuluhan menitipkan lagi sang anak kepada
pertanian untuk petani semangka pembantu (dalam Zamani, 2008)
karena jumlah produksi semangka (D) Sebagai pemegang titipan, tidak
merosot. selayaknya orang tua menitipkan
(B) perlu pembinaan kepada para lagi sang anak kepada pembantu
petani mangga karena jumlah (Nurudin, 2007).
produksi mangga semakin (E) Menurut Nurudin dalam bukunya
merosot. Zamani (2008) menyatakan bahwa
(C) petani anggur hendaknya dibina sebagai pemegang titipan, tidak
lebih intensif karena jumlah selayaknya orang tua menitipkan
produksi anggur paling rendah. lagi sang anak kepada pembantu.
Sony Sugema College 141
15. Banyak orang setuju motifasi itu bagai
misteri karena motifasi itu tidak tampak
tetapi berpengaruh pada performance
kerja, sikap, dan perilaku seseorang
dalam kehidupan sehari-hari.
Perbaikan ejaan kalimat di atas yang
tepat adalah ...
(A) Banyak orang setuju motifasi itu
bagai misteri, karena motifasi itu
tidak tampak tetapi berpengaruh
pada performance kerja, sikap dan
perilaku seseorang dalam
kehidupan sehari-hari.
(B) Banyak orang setuju motifasi itu
bagai misteri karena motifasi itu
tidak tampak tetapi berpengaruh
pada performance kerja, sikap dan
perilaku seseorang dalam
kehidupan sehari-hari.
(C) Banyak orang setuju motivasi itu
bagai misteri karena motivasi itu
tidak tampak tetapi berpengaruh
pada performance kerja, sikap, dan
perilaku seseorang dalam
kehidupan sehari-hari.
(D) Banyak orang setuju motifasi itu
bagai misteri, karena motifasi itu
tidak tampak tetapi berpengaruh
pada performance kerja, sikap dan
perilaku seseorang dalam
kehidupan sehari-hari.
(E) Banyak orang setuju motivasi itu
bagai misteri karena motivasi itu
tidak tampak, tetapi berpengaruh
pada performance kerja, sikap,
dan perilaku seseorang dalam
kehidupan sehari-hari.

142 Sony Sugema College


Sony Sugema College
SNMPTN Tahun 2011 Kode Soal 123

Kemunculan pola-pola lingkaran geometris di Walaupun wajib belajar pendidikan dasar 9


ladang pertanian atau crop circle selalu … tahun sudah dicanangkan dalam kurun waktu
dengan kehadiran alien di tempat itu dengan yang relatif lama, penuntasannya masih
mengendarai UFO. Dugaan tersebut belum tercapai. Banyak
didasarkan keyakinan bahwa manusia tidak masalah yang timbul dalam pelaksanaan
mampu ... pola serumit itu dalam waktu wajib belajar 9 tahun, terutama di daerah
singkat dengan hasil yang hampir sempurna. pedesaan dan daerah pegunungan atau
Sesuai namanya, UFO atau unidentified flying terpencil. Penyebab ketidaktuntasan wajib
object adalah objek terbang yang tidak …. belajar dapat diidentifikasi sesuai dengan
Bentuknya bermacam-macam, ada yang kondisi wilayah dan masyarakatnya. Dari
seperti titik cahaya yang diam sejenak dan sejumlah hasil penelitian ditemukan bahwa
menghilang cepat, ada yang seperti piring penyebabnya adalah (1) masyarakat memiliki
terbang. UFO selalu dikaitkan dengan alien, kondisi ekonomi yang lemah, (2) sosial
sang makhluk luar angkasa yang dalam film- budaya masyarakat yang kurang mendukung,
film fiksi ilmiah ... sebagai sosok makhluk (3) kurangnya sarana pendidikan, (4)
kecil, berkepala gundul, telinga lebar dan rendahnya kualitas dan dedikasi guru, (5)
berwarna hijau. la digambarkan memiliki letak geografis yang sulit dijangkau, (6)
kekuatan super, termasuk ... dirinya sama keterbatasan informasi, dan (7) persepsi
persis dengan makhluk bumi. masyarakat yang menganggap kurang
pentingnya pendidikan bagi dirinya sendiri.
1. Urutan kata yang paling tepat untuk Kenyataan itu diperkuat oleh hasil penelitian
melengkapi teks di atas adalah ... pada awal dicanangkannya wajib belajar 6
(A) dihubungkan, membentuk, dikenal, tahun.
ditampilkan, membuat.
(B) dihubungkan, membentuk, dikenali, 2. Kata itu yang tercetak miring pada teks
digambarkan, mencipta. di atas merujuk pada ...
(C) dikaitkan, menghasilkan, dikenal, (A) banyaknya masalah di daerah
dimunculkan, membentuk. terpencil.
(D) dikaitkan, membuat, dikenali, (B) penuntasan wajib belajar belum
digambarkan, tercapai.
mengubah. (C) banyaknya masalah pelaksanaan
(E) disertai, membuat, dikenali, wajib belajar.
ditampilkan, memunculkan. (D) ketidaktuntasan program wajib
belajar.
(E) penyebab ketidaktuntasan wajib
belajar.

Sony Sugema College 143


Teks 1 4. Seseorang akan mengembangkan
Sungguhpun sudah ada program yang karangan ilmiah yang bertema
namanya Bantuan Operasional Sekolah pembiasaan hidup sederhana.
(BOS), masih banyak anak Indonesia yang Berdasarkan tema tersebut, kerangka isi
kesulitan mengakses pendidikan di sekolah karangan yang paling runtut berdasarkan
pola umum-khusus adalah ...
dasar dan menengah. Berdasarkan angka
(A) pentingnya pembiasaan hidup
resmi yang dikeluarkan pemerintah, angka
sederhana, manfaat pembiasaan
putus sekolah untuk tingkat SD dan SMP
hidup sederhana, bentuk
sekitar 768.960 orang yang terdiri atas
pembiasaan hidup sederhana,
527.850 siswa SD dan 241.110 siswa SMP.
sasaran pembiasaan hidup
Belum lagi, masih ada sedikitnya 8,3 juta
sederhana, langkah pembiasaan
orang Indonesia yang masih buta huruf.
hidup sederhana.
Teks 2 (B) manfaat pembiasaan hidup
Salah satu penyebab lancarnya penuntasan sederhana, pentingnya pembiasaan
wajib belajar 9 tahun di Kabupaten Bandung hidup sederhana, sasaran
adalah adanya dukungan BOS yang pembiasaan hidup sederhana,
dikucurkan sejak tahun 2005. Memasuki langkah pembiasaan hidup
tahun 2009 program BOS mengalami sederhana, bentuk pembiasaan
perubahan tujuan, pendekatan, dan orientasi hidup sederhana.
dari yang semula untuk perluasan akses (C) pentingnya pembiasaan hidup
menjadi peningkatan kualitas. Jumlah siswa sederhana, sasaran pembiasaan
yang menerima dana BOS di Kabupaten hidup sederhana, manfaat
Bandung, tercatat 511.228 siswa. Masing- pembiasaan hidup sederhana,
bentuk pembiasaan hidup
masing SD Negeri sederajat 379.701 siswa,
sederhana, langkah pembiasaan
SD swasta sederajat 9.122 siswa, SMP Negeri
hidup sederhana.
sederajat 69.289 siswa, SMP swasta
(D) sasaran pembiasaan hidup
sederajat 53.116 siswa.
sederhana, manfaat pembiasaan
hidup sederhana, pentingnya
3. Pernyataan yang paling sesuai dengan
pembiasaan hidup sederhana,
kedua teks tersebut adalah ...
bentuk pembiasaan hidup
(A) kedua teks tersebut mengungkap
sederhana, langkah pembiasaan
kegagalan BOS. hidup sederhana.
(B) kedua teks tersebut mengungkap (E) bentuk pembiasaan hidup
keberhasilan BOS. sederhana, pentingnya pembiasaan
(C) teks 1 mengungkap kegagalan BOS, hidup sederhana, langkah
sedangkan teks 2 mengungkap pembiasaan hidup sederhana,
keberhasilan sasaran pembiasaan hidup
BOS. sederhana, manfaat pembiasaan
(D) teks 1 mengungkap keberhasilan hidup sederhana.
BOS, sedangkan teks 2 mengungkap
kegagalan BOS.
(E) teks 1 mengungkap akibat kegagalan
BOS, sedangkan teks 2 mengungkap
penyebab keberhasilan BOS.

144 Sony Sugema College


Sony Sugema College
Untuk menjawab pertanyaan nomor 5-8, 5. Pada kalimat manakah kesalahan
bacalah teks dari buku Rudy Hartono yang penggunaan ejaan (tanda baca,
berjudul Class Meeting: Pendidikan Life Skill penulisan huruf, dan penulisan kata)
yang diterbitkan tahun 2010 halaman 76 di ditemukan?
bawah ini dengan saksama! (A) 1 dan 6
(B) 2 dan 7
(1) Class meeting dapat memberikan (C) 3 dan 8
keterampilan kepada siswa dalam merancang (D) 4 dan 9
dan melaksanakan sebuah pertandingan (E) 5 dan 10
olahraga. (2) Tidak semua siswa setelah lulus
dapat meneruskan pendidikan ke jenjang 6. Jika Anda mengutip pendapat Charles
Perguruan Tinggi. (3) Untuk itu, mereka perlu dari teks di atas, tanpa membaca sumber
dibekali materi life skill untuk menghadapi aslinya, penulisan kutipan yang paling
kerasnya hidup. (4) Dengan class meeting benar adalah ...
siswa akan memiliki a) rasa percaya diri (A) "Dengan class meeting siswa akan
terhadap kemampuannya sebagai atlit atau memiliki
petugas pertandingan, b) rasa harga diri, a) rasa percaya diri terhadap
serta c) apresiasi nilai-nilai kesehatan dan kemampuannya sebagai atlit atau
sumbangan dari aktivitas fisik terhadap petugas pertandingan, b) rasa harga
kesehatan dirinya (Charles, 1995:410). (5) diri, serta c) apresiasi
Dengan demikian, implementasi pendidikan nilaknilai kesehatan dan sumbangan
jasmani melalui class meeting dapat dari aktivitas fisik terhadap
memberikan sarana preventif dan kuratif kesehatan dirinya" (Charles, 1995).
dalam mengurangi kenakalan remaja. (6) (B) Charles (dalam Rudy Hartono, 2010)
Tindakan kuratif dapat dilakukan dengan cara menyatakan "Dengan class meeting
lain, seperti: melalui bimbingan konseling siswa akan memiliki a) rasa percaya
atau melalui sarana keagamaan dalam diri terhadap kemampuannya
memberikan terapi pelajar yang sudah sebagai atlit atau petugas
terlanjur nakal. pertandingan, b) rasa harga diri,
(7) Jika class meeting sudah menjadi tradisi serta c) apresiasi nilai-nilai
setiap akhir semester yang dilaksanakan oleh kesehatan dan sumbangan dari
semua sekolah, niscaya akan membuahkan aktivitas fisik terhadap kesehatan
hasil, khususnya dalam mereduksi frekuensi dirinya".
terjadinya kenakalan pelajar. (8) Daya (C) Dalam buku Hartono, Charles
kreativitas dan energi lebih yang mereka (1995:410) menyatakan bahwa
miliki dapat tersalur melalui berbagai melalui class meeting siswa akan
aktivitas positif dalam class meeting. (9) memiliki a) rasa percaya diri
Selain itu, class meeting akan membina sikap terhadap kemampuannya sebagai
sportif, saling menghargai, mengakui atlit atau petugas pertandingan, b)
kelebihan orang lain, dan sikap positif lain rasa harga diri, serta c) apresiasi
yang sangat diperlukan siswa sebagai nilai-nitai kesehatan dan sumbangan
penerus masa depan, bangsa. (10) Untuk itu, dari aktivitas fisik terhadap
implementasi class meeting akan dapat kesehatan dirinya.
membentuk national character building yang
indikasinya mulai lapuk.
Sony Sugema College 145
(D) Charles (1995) menyatakan "Dengan (C) Class meeting memberikan
class meeting siswa akan memiliki a) keterampilan kepada siswa dalam
rasa percaya diri terhadap merancang dan melaksanakan
kemampuannya sebagai atlit atau pertandingan olahraga yang
petugas pertandingan, b) rasa harga bermanfaat bagi penyaluran
diri, serta c) apresiasi nilai-nilai aktivitas positif melalui bimbingan
kesehatan dan sumbangan dari konseling atau sarana keagamaan
aktivitas fisik terhadap kesehatan dalam memberikan terapi pelajar
dirinya" (Hartono, 2010). yang nakal.
(E) Menurut Charles (dalam Hartono, (D) Daya kreativitas dan energi lebih
2010:76), dengan class meeting para pelajar dapat tersalur melalui
siswa akan memiliki a) rasa percaya aktivitas preventif sehingga kegiatan
diri terhadap kemampuannya class meeting dapat memberikan
sebagai atlit atau petugas kesadaran bagi pelajar yang
pertandingan, b) rasa harga diri, terlanjur nakal, khususnya dalam
serta c) apresiasi nilai-nilai mereduksi frekuensi terjadinya
kesehatan dan sumbangan dari kenakalan pelajar.
aktivitas fisik terhadap kesehatan (E) Jika class meeting sudah menjadi
dirinya. tradisi setiap akhir semester yang
dilaksanakan oleh semua sekolah,
7. Rangkuman yang paling tepat teks di niscaya daya kreativitas dan energi
atas adalah lebih dapat tersalur melalui aktivitas
(A) Daya kreativitas dan energi lebih positif untuk mengurangi kenakalan
yang dimiliki siswa dapat disalurkan remaja dalam membentuk national
lewat aktivitas positif sebab kegiatan character building.
class meeting dapat berfungsi
sebagai sarana kuratif untuk 8. Pola paragraf pertama teks di atas
mengurangi kenakalan remaja dan adalah ...
dapat membentuk national (A) kalimat penjelas - kalimat utama -
character building yang mulai lapuk. kalimat penjelas - kalimat penjelas.
(B) Class meeting memberikan (B) kalimat penjelas - kalimat penjelas -
keterampilan dalam merancang dari kalimat penjelas - kalimat utama.
melaksanakan kegiatan sehingga (C) kalimat utama - kalimat penjelas -
berfungsi sebagai sarana preventif kalimat penjelas - kalimat utama.
dan kuratif dalam mengurangi (D) kalimat utama - kalimat utama -
kenakalan remaja, yang pada kalimat penjelas - kalimat penjelas.
akhimya dapat membentuk national (E) kalimat utama - kalimat penjelas -
character building. kalimat penjelas - kalimat penjelas.

146 Sony Sugema College


Sony Sugema College
Untuk monjawab perlanyaon nomor 9-12, 10. Suntingan yang paling tepat kalimat ke-3
bacalah teks berikut dengan cermat! teks di atas adalah ...
(A) Beberapa motif batik disengaja
Motif batik bukan sekadar lukisan yang untuk menunjukkan status
ditorehkan pada kain dengan canting. Motif pemakainya.
pada selembar kain batik memiliki makna (B) Beberapa motif batik disengaja agar
tersembunyi. Beberapa motif batik sengaja menunjukkan status si pemakainya.
untuk menunjukkan status si pemakainya. (C) Motif-motif batik disengaja
Bahkan, sampai saat ini di Yogyakarta dan menunjukkan status si pemakainya.
Surakarla ada sejumlah motif batik (D) Beberapa motif batik sengaja untuk
tradisional yang hanya boleh dipakai oleh menunjukkan status pemakainya.
keluarga keraton. (E) Pada beberapa motif batik
Setiap motif batik tradisional memiliki filosofi menunjukkan status si pemakainya.
tersendiri. Bahkan, pada motif tertentu ada yang
dianggap sakral dan hanya dapat dipakai pada 11. Kalimat penutup yang paling tepat untuk
kesempatan atau peristiwa tertentu. Motif Sida paragraf ke-2 teks tersebut adalah ...
Mukli, misalnya, yang secara harfiah berarti (A) Jadi, pada masa dahulu banyak
"menjadi berkecukupan/makmur" biasanya orang membatik hanya untuk mata
hanya digunakan oleh kalangan keluarga pencaharian.
keraton. Ada lagi motif Wahyu Tumurun, yang (B) Dengan demikian, setiap motif batik
berarti turunnya wahyu. Motif itu digunakan tradisional memiliki fungsi yang
pada upacara jumenengan atau naik tahta raja. berbeda.
Motif Perang yang bernuansa cukup ramai (C) Dengan demikian, membatik adalah
biasanya dipakai untuk menghadiri suatu pesta warisan budaya yang perlu
atau perayaan. Untuk melayat (menghadiri dilestarikan.
pemakaman), digunakan warna yang lebih (D) Oleh sebab itu, batik memiliki motif
lembut, yaitu motif Kawung. Keempat motif yang bermacam-macam ragamnya.
batik tersebut hanya diperuntukan bagi keluarga (E) Jadi, batik adalah warisan leluhur
keraton dan tidak boleh digunakan oleh rakyat yang memiliki nilai filosofis kekuatan
pada umumnya. magis.
... .
9. Simpulan yang paling tepat untuk teks di 12. Gagasan utama paragraf ke-2 teks di atas
atas adalah ... adalah ....
(A) Motif batik bukan sekadar lukisan (A) Nilai filosofis selalu ada dalam setiap
karena bermakna tersembunyi. motif batik tradisional.
(B) Setiap motif batik tradisional (B) Motif batik memiliki fungsi dan
memiliki filosofi tersendiri yang kegunaan tersendiri bagi
dianggap sakral. pemakainya.
(C) Membatik adalah kegiatan (C) Hanya keluarga dari keraton yang
menorehkan lukisan pada kain, dapat nnemakai batik dengan motif
menggunakan canting. tertentu.
(D) Motif batik memiliki makna (D) Terdapat empat motif khusus batik
tersembunyi yang bersifat filosofis. untuk keluarga keraton.
(E) Empat motif batik hanya (E) Semua motif batik tradisional hanya
diperuntukan bagi keluarga keraton. diperuntukan bagi ketuarga keraton
Sony Sugema College 147
Untuk menjawab pertanyaan nomor 13-15, bacalah teks berikut secara cermat!

(1) Pembangunan perdagangan di Kabupaten Bantul dilaksanakan dengan mengembangkan


pemasaran berpola kemitraan dan promosi produk melalui misi dagang yang berlangsung
setiap tahun. (2) Pola tersebut mampu mendukung peningkatan pangsa pasar dan secara
makro mendukung pembangunan ekonomi di Kabupaten Bantul.
(3) Kualitas perdagangan luar negeri Bantul terlihat dari kemampuannya untuk melakukan
ekspor. (4) Dari segi volume, kemampuan untuk mengeskpor selama lima tahun tampak
menurun, tetapi jika dilihat dari nilainya justru cenderung meningkat. (5) Ini berarti bahwa
harga produk yang dieskpor mengalami kenaikan. (6) Jika dilihat dari besarnya jumlah negara
yang dituju maka tampak jumlah negara-negara tujuan dan bermacam-macam bentuk
komoditas yang dijual semakin kecil. (7) Di satu sisi, ini dapat bermakna positif, artinya
kemampuan ekspor ke masing-masing negara menjadi lebih tinggi dan komoditas yang dijual
semakin terspesialisasi. (8) Aspek negatifnya, konsentrasi pada sedikit negara dan sedikit
komoditas seperti meletakkan seluruh telur dalam satu keranjang, jika ada kegoncangan dari
negara yang dituju atau komoditas yang dipilih, maka sulit untuk melakukan pemulihan.
(9) Perekonomian Bantul sampai saat ini masih didominasi kegiatan perdagangan skala kecil
dan ekonomi lokal. (10) Suburnya pasar tradisional dapat menjadi indikasi kuatnya peran
ekonomi lokal dalam perekonomian Bantul.

Tabel Perkernbangan Ekspor Kabupaten Bantul


Tahun 2005-2009
Volume Nilai Jumlah negara Macam
Tahun
(ton) (US$ 000) tujuan Komoditas

2005 8.800 21.100 69 51


2006 8.500 23.600 60 48
2007 8.600 20.100 57 56
2008 7.000 20.500 48 29
2009 6.200 24.000 52 32

13. Kalimat tidak efektif terdapat pada 14. Pernyataan berikut yang paling sesuai
kalimat bernomor... dengan isi tabel di atas adalah ...
(1) 6. (1) Pada tahun 2005 Kabupaten Bantul
(2) 7. memiliki negara tujuan ekspor lebih
(3) 8. banyak dibandingkan tahun 2007
(4) 9. sehingga nilai ekspor tahun 2005
menjadi lebih tingggi.
(2) Tujuan negara ekspor dan macam
komoditas ekspor mengalami
penurunan pada tahun 2008, tetapi
nilai ekspor bertambah daripada
tahun sebelumnya.

148 Sony Sugema College


Sony Sugema College
(3) Pada tahun 2009 jumlah volume
ekspor lebih rendah dibandingkan
tahun 2008, tetapi jumlah nilai,
negara tujuan, dan macam
komoditas mengalami kenaikan.
(4) Nilai ekspor pada tahun 2007 turun
dibandingkan tahun 2006, tetapi
jumlah negara tujuan dan komoditas
lebih tinggi.

15. Komentar yang paling tepat dengan isi


teks di atas adalah ...
(1) Kebijakan pemerintahan Kabupaten
Bantul melindungi dan melestarikan
pasar tradisional untuk
perekonomian lokal perlu menjadi
contoh untuk kabupaten lain di
Indonesia.
(2) Peningkatan nilai ekspor di
Kabupaten Bantul yang tidak
bergantung pada banyaknya negara
tujuan danienis komoditas yang
dijual perlu ditiru oleh kabupaten
lain.
(3) Terspesialisasinya komoditas ekspor
dan negara tujuan yang telah
menaikkan nilai ekspor Kabupaten
Bantul tetap harus diantisipasi agar
tidak berisiko lebih besar bagi para
pedagang.
(4) Fokus perdagangan yang didominasi
kegiatan perdagangan skala kecil
dan ekonomi lokal dapat
meningkatkan nilai ekspor di
Kabupaten Bantul.

Sony Sugema College 149


SNMPTN Tahun 2012 Kode Soal 321

1. Bacalah teks berikut! 2. Seseorang akan mengembangkan


Berita yang dilansir Antara bulan karangan yang bertema upaya
lalu dengan tajuk “Bahasa Indonesia meningkatkan kemampuan
Berpeluang Jadi Bahasa Kedua ASEAN”, berwirausaha bagi remaja dengan
memberikan satu harapan besar bagi kerangka karangan berikut:
Indonesia untuk berbicara di kancah 1) saran bagi remaja untuk
internasional. Sekarang ada kesadaran di berwirausaha
kalangan warga Philipina–teristimewa di 2) pentingnya meningkatkan
kawasan selatan negara ini–bahwa kemampuan berwirausaha
mereka perlu belajar bahasa Indonesia 3) langkah meningkatkan kemampuan
karena bahasa itu dapat dimengerti di berwirausaha
empat negara anggota ASEAN lainnya: 4) komponen yang mendukung
Brunei, Malaysia, Singapura, dan kemampuan berwirausaha
Thailand (Selatan). Selain keempat 5) ketidakmampuan remaja
negara ASEAN itu, sebagian warga suku berwirausaha
Champ di Kamboja, Laos, dan Vietnam Urutan kerangka yang paling tepat untuk
mengerti bahasa Melayu yang menjadi tema karangan di atas adalah ...
induk bahasa Indonesia. .... (A) 5, 1, 2, 4, 3
Manakah kalimat yang tepat untuk (B) 2, 4, 5, 1, 3
menutup paragraf di atas? (C) 2, 3, 4, 5, 1
(A) Untuk itu, kita perlu menggalakkan (D) 4, 5, 2, 1, 3
penyebaran kamus bahasa (E) 5, 2, 4, 3, 1
Indonesia agar memudahkan orang
asing.
(B) Dengan demikian, jelaslah bahwa
bahasa Indonesia menjadi bahasa
yang diidamkan di kawasan ASEAN.
(C) Oleh karena itu, bangsa Indonesia
harus dapat menggunakan bahasa
Indonesia dengan baik dan benar.
(D) Untuk itu, perlu diupayakan agar
bahasa Indonesia menjadi bahasa
utama ASEAN.
(E) Sehubungan dengan hal itu,
menjunjung tinggi bahasa Indonesia
menjadi kewajiban seluruh bangsa
Indonesia.

150 Sony Sugema College


Sony Sugema College
3. Bacalah kedua teks berikut! Teks 2
Teks 1 Pada umumnya, orang memandang
Obat telah menjadi bagian yang obat sebagai sesuatu yang ajaib yang
tidak terpisahkan dari kehidupan dapat menyembuhkan segala penyakit
manusia masa kini. Bahkan, karena dengan cepat tanpa efek samping.
begitu akrabnya, orang justru semakin Semua obat adalah racun, kecuali
tergantung pada obat, semakin terbiasa diminum sesuai dengan dosis yang
menggunakan obat. Penggunaan obat- diizinkan. Hal itu menyadarkan kita
obatan kini justru terlihat sebagai gaya bahwa dalam mengonsumsi obat, kita
hidup modern. Mereka perlu tampil harus memperhatikan dosis yang tepat
seksi, bukan sehat, perlu obat kuat, untuk kesembuhan yang diharapkan.
bukan hidup harmonis, dan lain-lain. Masyarakat menaruh harapan besar
Faktanya zat-zat yang terkandung dalam akan obat, namun hal tersebut tidak
obat dan efeknya bagi tubuh kita sering sebanding dengan pengetahuan
tidak kita perhatikan karena masyarakat akan obat. Informasi tentang
menganggap obat itu menyembuhkan pentingnya obat, penggunaan, dan
tanpa memperhatikan apa yang bahayanya sangat penting bagi
sebenarnya terkandung dalam obat masyarakat.
tersebut. Dalam pemilihan obat harus
diperhatikan adanya kandungan bahan- Kedua teks di atas menjelaskan tentang ...
bahan kimia yang justru menimbulkan (A) kebutuhan manusia akan obat.
dampak buruk terhadap tubuh kita. (B) cara mengonsumsi obat.
(C) anggapan masyarakat akan obat.
(D) pengetahuan masyarakat akan obat.
(E) harapan manusia akan obat.

Bacalah teks berikut untuk menjawab soal nomor 4 sampai dengan nomor 6!

Sampai saat ini narkoba masih mengancam masyarakat Indonesia meski Indonesia telah
berkomitmen untuk bebas dari narkoba dan HIV AIDS pada tahun 2015. Hal itu dapat dilihat
dari jumlah pengguna narkoba yang terus meningkat setiap tahunnya. Pada tahun 1970
diperkirakan hanya 130.000 orang yang menggunakan narkoba dan pada tahun 2009
terdeteksi 2% penduduk Indonesia pernah bersentuhan dengan narkoba atau meningkat 0,5%
dibandingkan tahun sebelumnya. Hal tersebut sangat mengkhawatirkan semua pihak,
khususnya Badan Narkotika Nasional. Dari 2% penduduk yang pernah bersentuhan dengan
narkoba tersebut, 60% berusia produktif dan 40% pelajar.
Awalnya, pengguna narkoba adalah orang dewasa, berusia sekitar 25 tahun dan dari
kalangan ekonomi kelas menengah ke atas. Dalam perkembangannya, pengguna narkoba
sudah merambah para remaja dan masyarakat kelas menengah ke bawah. Bahkan,
gelandangan pun ada yang kecanduan narkoba. Keadaan tersebut sungguh sangat ironis.
Kondisi pengguna narkoba di Indonesia pada tahun 2005 – 2007 dipaparkan sebagai berikut.

Sony Sugema College 151


Tabel Pengguna Narkoba di Indonesia
Tahun 2005 – 2007
Usia 2005 2006 2007
< 16 127 175 180
16 – 19 1.668 2.447 2.617
20 – 24 5.503 8.383 8.275
25 – 29 6.442 8.105 9.278
>29 9.040 12.525 15.889
4. Pernyataan manakah yang paling tidak (B) Kondisi di atas menunjukkan bahwa
sesuai dengan isi tabel di atas? pengguna narkoba selalu meningkat.
(A) Semakin rendah kelompok usia Pengguna narkoba sudah merambah
seseorang, semakin kecil juga pada remaja berusia muda.
tingkat penggunaan narkoba. Pengguna narkoba juga merambah
(B) Pengguna narkoba pada usia 25 – 29 pada masyarakat dari kalangan
selalu lebih banyak daripada ekonomi kelas menengah ke bawah.
kelompok-kelompok usia lain yang (C) Meskipun narkoba membahayakan,
berusia < 24 tahun. para pengguna pada umumnya tidak
(C) Dari tahun ke tahun jumlah menyadarinya. Upaya menyadarkan
pengguna narkoba setiap kelompok pengguna narkoba harus dimulai dari
usia meningkat. pihak yang paling dekat dengan
(D) Semakin tinggi kelompok usia pengguna. Kalau tidak, mustahil upaya
seseorang, semakin besar tingkat tersebut dapat berhasil.
penggunaan narkobanya. (D) Dari tabel tersebut tampak bahwa
(E) Kelompok usia selalu menentukan jumlah pengguna narkoba selalu
jumlah atau banyaknya pengguna meningkat. Pada tahun 1970
narkoba. diperkirakan hanya 130.000 orang
5. Kata tersebut pada kalimat ke-4 paragraf yang menggunakan narkoba, tetapi
ke-1 di atas merujuk pada ... pada tahun 2009 terdeteksi 2%
(A) narkoba masih mengancam. penduduk pernah bersentuhan
(B) jumah pengguna narkoba. dengan narkoba.
(C) peningkatan pengguna narkoba. (E) Dari tabel tersebut tampak bahwa
(D) komitmen Indonesia. jumah pengguna narkoba mulai
(E) persentase pengguna narkoba. bervariasi. Mulai dari remaja sampai
dengan orang tua banyak yang
6. Paragraf manakah yang paling tepat menjadi pengguna narkoba.
melengkapi teks di atas? Masyarakat kelas menengah ke
(A) Tabel tersebut menunjukkan bahwa bawah dan bahkan gelandangan pun
pengguna narkoba semakin meningkat. ada yang kecanduan narkoba.
Untuk mengatasinya diperlukan upaya
sinergis dari semua pihak, khususnya
Badan Narkotika Nasional dengan
masyarakat. Tanpa sinergi tersebut
tidak mungkin bahaya narkoba dapat
diatasi.

152 Sony Sugema College


Sony Sugema College
Bacalah teks berikut untuk menjawab soal (11) Pada bidang informatika Tim
nomor 7 sampai dengan nomor 11! Olimpiade Komputer Indonesia (TOKI), akan
berkompetisi di International Olympiad in
(1) Indonesia kembali mengirimkan Informatics (IOI) ke-23 di Pattaya. (12) Pada
para siswa SMA ke olimpiade sains penyelenggaraan IOI ke-22 di Kanada, TOKI
internasional tahun 2011 untuk bidang fisika, meraih 2 medali perak dan 1 perunggu. (13)
matematika, biologi, kimia, dan informatika Kini TOKI Indonesia mengirimkan empat
ke sejumlah negara penyelenggara. (2) Ketika siswa dengan harapan kembali dapat
menerima tim-tim tersebut, Fasli Jalal memenangkan medali. (14) Seluruh bangsa
menyampaikan bahwa para peserta Indonesia turut serta mendoakan agar para
olimpiade diseleksi dengan ketat mulai dari siswa yang pahlawan itu berhasil
sekolah masing-masing sampai ke tingkat berkompetisi di ajang internasional. (15)
regional. (3) Jadi, mereka yang terpilih benar- Keberhasilan itu akan mendongkrak harga
benar memiliki kemampuan yang dapat diri bangsa yang ternyata tidak kalah
diandalkan dan berpeluang untuk meraih berprestasi dari bangsa-bangsa lain di dunia.
medali. (4) Cakupan asal daerah peserta
semakin merata, mulai dari kawasan barat 7. Kesalahan penggunaan ejaan ditemukan
sampai timur nusantara, yang menunjukkan pada kalimat ...
semakin meratanya prestasi peserta didik di (A) 6 dan 9
Indonesia. (5) “Indonesia tidak henti- (B) 5 dan 10
hentinya menunjukkan kebolehan siswa di (C) 4 dan 11
berbagai forum terhormat”, katanya. (D) 3 dan 12
(6) Pada bidang fisika Tim Olimpiade (E) 2 dan 15
Fisika Indonesia (TOFI) akan berlaga di
Olimpiade Fisika Internasional atau 8. Kalimat yang tidak efektif ditemukan
International Physics Olympiad (IPhO) ke-42 pada ...
di Bangkok. (7) Pada bidang matematika (A) 2 dan 6
Indonesia mengirimkan enam siswa ke (B) 3 dan 7
International Mathematical Olympiad (IMO) (C) 5 dan 9
ke-52 yang diselenggarakan di Amsterdam. (D) 8 dan 14
(8) Tim-tim ini ditargetkan meraih 2 medali (E) 10 dan 15
perak dan 4 medali perunggu, sedang tahun-
tahun lalu tim IMO Indonesia berada pada 9. Rangkuman manakah yang paling tepat
ranking ke-30 dari dari 96 negara-negara dan untuk teks di atas?
berhasil menyabet 1 medali perak, 4 medali (A) Tim Olimpiade Sains Indonesia yang
perunggu, dan 1 honorable mention. (9) Pada meliputi empat bidang akan kembali
bidang biologi tim akan berlaga di arena mengikuti olimpiade dengan
kompetisi International Biology Olympiad mengirimkan siswa-siswa SMA ke
(IBO) ke-22 di Taipei, Taiwan. (10) Indonesia berbagai negara di dunia.
kini mengirimkan 4 siswa yang ditargetkan (B) Tim Olimpiade Sains Indonesia akan
meraih medali, dan pada ajang tahun lalu, mengirimkan para siswa SMA untuk
tim IBO meraih 2 medali emas dan 2 mengikuti olimpiade dalam empat
perunggu. bidang ke berbagai negara dengan
harapan dapat memenangkan
medali.
Sony Sugema College 153
(C) Ada empat bidang dalam Tim Bacalah teks berikut untuk menjawab soal
Olimpiade Sains Indonesia yang nomor 12 sampai dengan nomor 15!
akan dikirimkan ke berbagai negara
untuk mengikuti olimpiade yaitu Tidak diragukan lagi bahwa yang
mencakup bidang fisika, merupakan sumber pencemaran sungai
matematika, biologi, dan komputer. adalah segala kegiatan hidup manusia yang
(D) Tim Olimpiade Sains Indonesia yang berkaitan erat dengan proses produksi dan
dikirimkan ke berbagai negara untuk reproduksi. Peningkatan jumlah penduduk
mengikuti olimpiade empat bidang serta turunnya standar hidup merupakan
adalah mencakup bidang fisika, parameter yang juga ikut menentukan laju
matematika, biologi, dan komputer. pencemaran sungai. Makin berjubelnya
(E) Pengiriman Tim Olimpiade Sains penduduk dengan standar hidup yang
Indonesia ke berbagai negara untuk memprihatinkan secara langsung
mengikuti olimpiade diharapkan memberikan dampak negatif terhadap
mampu memenangkan banyak lingkungan sungai. Jika diperhatikan, sungai-
medali agar dapat mendongkrak sungai yang mengalir di berbagai tempat,
harga diri bangsa Indonesia. kendati kondisinya sangat menjijikkan, sering
dipakai untuk mandi, mencuci pakaian,
10. Hubungan isi antarparagraf dalam teks di mencuci piring, bahkan menggosok gigi.
atas yang paling tepat adalah ... Sementara itu, sungai tersebut secara terus
(A) paragraf ke-2 menjadi akibat menerus dijejali dengan timbunan sampah,
paragraf ke-1. air limbah, dan tinja penduduk. Makin ke
(B) paragraf ke-3 merupakan penyebab hilir, warna air sungai makin hitam dan pekat.
paragraf ke-2. Apabila sungai tercemar, hal itu akan
(C) paragraf ke-2 memaparkan rincian memberikan keleluasaan bagi bibit-bibit
paragraf ke-1. penyakit untuk berkembang biak. Akhirnya,
(D) paragraf ke-3 merupakan kesehatan masyarakat akan terganggu.
pembanding paragraf ke-1. Gangguan kesehatan yang banyak muncul di
(E) paragraf ke-3 merupakan contoh antaranya adalah penyakit diare, pernapasan,
paragraf ke-2. demam berdarah, tipus, dan mag.
Untuk menghindari penumpukan
11. Penulis teks di atas berpihak kepada ... sampah yang sulit melapuk, perlu dibuat
(A) para siswa SMA. tempat penimbunan khusus di luar kota. Hal
(B) pemerintah Indonesia. yang baik adalah apabila sampah rumah
(C) bangsa Indonesia. tangga sejak dari rumah sudah dilakukan
(D) tim olimpiade sains Indonesia. pemisahan antara sampah yang mudah lapuk
(E) penyelenggara oilimpiade sains. dan yang sulit lapuk. Sudah waktunya,
sampah tidak dibuang ke sungai. Selain itu,
pembuatan septic tank di rumah-rumah akan
sangat membantu menanggulangi
pencemaran sungai. Khusus air buangan
industri, seharusnya, sebelum dibuang ke
sungai terlebih dahulu diolah melalui instalasi
penjernihan. Hal itu dimaksudkan untuk

154 Sony Sugema College


Sony Sugema College
mencegah adanya zat-zat yang bersifat 13. Apakah gagasan utama paragraf ke-4 di
racun. atas?
Usaha pencegahan harus ditunjang oleh (A) Kesadaran masyarakat untuk
adanya kesadaran masyarakat akan mencegah pencemaran sungai akan
pentingnya menjaga kelestarian sungai. membantu kelestarian sungai.
Kesadaran menjadi modal dasar yang positif (B) Kesadaran menjadi modal dasar bagi
bagi peningkatan perilaku yang baik dalam peningkatan perilaku yang baik
memperlakukan sungai. Sungai yang lestari dalam memperlakukan sungai.
mencerminkan keserasian lingkungan hidup (C) Perilaku tidak membuang sampah
manusia. Dampaknya adalah ekologi dan tidak membangun rumah di
lingkungan akan menjadi lebih baik, air bersih bantaran sungai membantu
tetap tersedia, dan kesehatan masyarakat keserasian lingkungan hidup
tetap terjaga. manusia.
(D) Kesadaran menciptakan sungai yang
12. Manakah suntingan yang paling tepat lestari mencerminkan keserasian
untuk kalimat ke-1 paragraf ke-1 di atas? lingkungan hidup manusia.
(A) Tidak diragukan lagi bila yang (E) Dampak kelestarian sungai adalah
merupakan sumber pencemaran air bersih tetap tersedia dan
sungai adalah segala kegiatan hidup kesehatan masyarakat tetap terjaga.
manusia yang berkaitan erat dengan
proses produksi dan reproduksi. 14. Manakah simpulan yang paling tepat
(B) Tidak diragukan lagi bahwa untuk teks di atas?
sesungguhnya sumber pencemaran (A) Sumber pencemaran sungai adalah
sungai ialah segala kegiatan hidup segala kegiatan hidup manusia yang
manusia yang berkaitan erat dengan berkaitan erat dengan proses
proses produksi dan reproduksi. produksi dan reproduksi.
(C) Tidak diragukan lagi bahwa sumber (B) Sumber pencemaran sungai
pencemaran sungai adalah segala berkaitan erat dengan kegiatan
kegiatan hidup manusia yang hidup manusia, peningkatan jumlah
berkaitan erat dengan proses penduduk, serta turunnya standar
produksi dan reproduksi. hidup.
(D) Tidak diragukan lagi apabila sumber (C) Peningkatan jumlah penduduk dan
pencemaran sungai adalah segala turunnya standar hidup merupakan
kegiatan hidup manusia yang parameter yang menentukan
berkaitan erat dengan proses pencemaran sungai.
produksi dan reproduksi. (D) Kepadatan penduduk dengan
(E) Tidak diragukan lagi, yang standar hidup yang memprihatinkan
merupakan sumber pencemaran memberikan dampak negatif
sungai itu segala kegiatan hidup terhadap lingkungan sungai.
manusia yang berkaitan erat dengan (E) Perilaku hidup tidak sehat
proses produksi dan reproduksi. masyarakat dipengaruhi oleh
kebiasaan negatif mereka dalam
memanfaatkan air sungai.

Sony Sugema College 155


15. Kelemahan paragraf ke-3 di atas adalah
...
(A) gagasan utamanya terletak di
tengah.
(B) tidak ada kalimat utamanya.
(C) terdapat kalimat sumbang di
tengah.
(D) istilah asing tidak dijelaskan artinya.
(E) gagasan utamanya lebih dari satu.

156 Sony Sugema College


Sony Sugema College
SBMPTN Tahun 2013 Kode Soal 123

Teks berikut digunakan untuk menjawab soal nomor 1 – 4.

(1) Semua orang pasti mengenal pendidikan. (2) Pendidikan adalah proses internalisasi
budaya ke dalam diri seseorang dan masyarakat sehingga membuat orang dan masyarakat jadi
beradab. (3) Pendidikan bukan hanya merupakan sarana transfer ilmu pengetahuan, tetapi
lebih luas lagi, yakni sebagai sarana pembudayaan dan penyaluran nilai (enkulturisasi dan
sosialisasi). (4) Anak harus mendapatkan pendidikan yang menyentuh dimensi dasar
kemanusiaan. (5) Dimensi kemanusiaan itu mencakup sekurang-kurangnya tiga hal paling
mendasar.
(6) Pendidikan karakter adalah pendidikan budi pekerti plus, yaitu yang melibatkan
aspek pengetahuan, perasaan, dan tindakan. (7) Menurut Lickona, tanpa ketiga aspek itu,
pendidikan karakter tidak akan efektif. (8) Dengan pendidikan karakter yang diterapkan secara
sistematis dan berkelanjutan, seorang anak akan menjadi cerdas emosinya. (9) Kecerdasan
emosi ini adalah bekal penting dalam mempersiapkan anak menyongsong masa depan. (10)
Terdapat sembilan pilar karakter yang berasal dari nilai-nilai luhur universal, yaitu karakter
cinta Tuhan dan segenap ciptaan-Nya; kemandirian dan tanggung jawab; kejujuran/amanah
dan diplomatis; hormat dan santun; dermawan, suka menolong, dan gotong royong/kerja
sama; percaya diri dan pekerja keras; kepemimpinan dan keadilan; baik dan rendah hati; serta
toleran dan cinta damai.

(Dikutip dengan pengubahan dari blogdetik.com)

1. Manakah gagasan utama paragraf ke-1? 2. Manakah pertanyaan yang jawabannya


(A) Pendidikan dikenal setiap orang. terdapat dalam paragraf ke-2?
(B) Pendidikan adalah internalisasi (A) Mengapa pendidikan karakter
budaya. merupakan pendidikan budi
(C) Pendidikan bukan sarana transfer pekerti?
ilmu. (B) Mengapa kecerdasan emosi penting
(D) Pendidikan merupakan sarana dalam mempersiapkan masa depan?
pembudayaan. (C) Bagaimana pendidikan karakter
(E) Pendidikan harus berdimensi yang sistematis dan berkelanjutan?
kemanusiaan. (D) Bagaimana pendapat Lickona
tentang aspek pendidikan karakter?
(E) Siapa yang mengemukakan sembilan
pilar karakter berasal dari nilai luhur
universal?

Sony Sugema College 157


3. Apa perbedaan gagasan antarparagraf 4. Apa kelemahan paragraf ke-1?
dalam teks tersebut? (A) Tidak ada hubungan antara
(A) Paragraf pertama memaparkan penjelasan kecerdasan emosi
pendidikan karakter secara umum, (kalimat 9) dengan sembilan pilar
sedangkan paragraf kedua karakter (kalimat 10).
memaparkan pendidikan karakter (B) Plus pada pendidikan budi pekerti
secara khusus. (kalimat 6) tidak dijelaskan secara
(B) Paragraf pertama memaparkan konsep rinci pada bagian berikutnya.
pendidikan karakter, sedangkan (C) Penjelasan tentang pendidikan
paragraf kedua memaparkan unsur sebagai sarana transfer ilmu
pendidikan karakter. (kalimat 3) bertentangan dengan
(C) Paragraf pertama memaparkan konsep pendidikan (kalimat 2).
pendidikan secara umum, sedangkan (D) Tidak ada hubungan antara ketiga
paragraf kedua memaparkan tujuan aspek yang dimaksud Lickona
pendidikan secara khusus. (kalimat 7) dengan nilai plus pada
(D) Paragraf pertama memaparkan pendidikan budi pekerti (kalimat 6).
konsep pendidikan secara umum, (E) Penjelasan tentang kecerdasan
sedangkan paragraf kedua emosi (kalimat 8) tidak berkaitan
memaparkan konsep pendidikan dengan pendidikan karakter (kalimat
karakter. 7).
(E) Paragraf pertama memaparkan
pendidikan secara umum,
sedangkan paragraf kedua
memaparkan manfaat pendidikan
karakter.

Teks berikut digunakan untuk menjawab soal nomor 5 – 9.

(1) Organisasi menggunakan berbagai penghargaan untuk mempertahankan dan


memotivasi karyawan dalam mencapai tujuan pribadi mereka dan tujuan organisasi. (2) Cara
dan waktu membagikan penghargaan merupakan masalah yang penting. (3) Jika hal itu
diberikan secara tidak tepat maka hasil yang diperoleh tidak akan memuaskan organisasi.
(4) Tujuan utama pemberian penghargaan adalah menarik orang yang berkualitas untuk
bergabung dalam organisasi, mempertahankan karyawan agar tetap dating bekerja dan
memotivasi karyawan untuk mencapai prestasi kerja yang lebih tinggi (Gibson, Ivancevich, dan
Donnely, 1991). Salah satu penghargaan yang digunakan dalam organisasi adalah penghargaan
ekstrinsik. (6) Menurut mereka penghargaan ekstrinsik meliputi gaji dan upah, jaminan sosial,
penghargaan antar pribadi serta promosi.
(7) Gaji dan upah adalah uang yang diberikan oleh pihak manajemen ketika menghargai
hasil kerja para karyawannya. (8) Sistem penilaian prestasi harus dirancang dengan baik agar
hubungan antara upah dan prestasi menjadi jelas bagi karyawan. (9) Jaminan sosial yang
diberikan organisasi dapat berupa tunjangan yang bersifat finansial maupun non finansial. (10)
Kebanyakan organisasi memberikan tunjangan yang bersifat finansial berupa program dana

158 Sony Sugema College


Sony Sugema College
pensiun. (11) Sedangkan tunjangan yang bersifat non finansial contohnya adalah program
rekreasi untuk para karyawan.
(12) Manajemen dalam suatu organisasi juga mempunyai kewenangan untuk memberikan
sanksi atau penghargaan antar pribadi. (13) Sanksi dapat diberikan antara lain dengan cara
memberikan pekerjaan yang dianggap organisasi tidak penting atau tidak berharga. (14) Sebaliknya,
untuk karyawan yang memiliki kinerja baik, manajer dapat mempromosikannya dengan memberikan
pekerjaan yang berwibawa sehingga statusnya meningkat.

(Dikutip dengan pengubahan dari www.materikuliah.com)

5. Penggunaan kata yang tidak tepat (D) Paragraf ke-4 merupakan


terdapat pada kata .... pembanding paragraf ke-3.
(A) memotivasi dalam kalimat 1 (E) Paragraf ke-4 merupakan akibat
(B) manajemen dalam kalimat 7 paragraf ke-3.
(C) sistem dalam kalimat 8
(D) manajemen dalam kalimat 12 9. Mengapa sebuah organisasi harus
(E) manajer dalam kalimat 14 menyusun sistem penilaian prestasi kerja
karyawan?
6. Kata itu pada kalimat 3 merujuk pada .... (A) Sistem penilaian terhadap prestasi
(A) mencapai tujuan organisasi kerja karyawan menjadikan sebuah
(B) cara dan waktu memberikan organisasi menyediakan penghargaan
penghargaan untuk karyawan.
(C) menggunakan berbagai (B) Melalui penilaian prestasi kerja, sebuah
penghargaan organisasi telah memperhatikan faktor
(D) mempertahankan dan memotivasi finansial yang harus diberikan pada
karyawan karyawan.
(E) mencapai tujuan pribadi (C) Penilaian prestasi akan berdampak
pada gaji, upah, dan jaminan sosial
7. Kesalahan penggunaan ejaan ditemukan yang akan diterima para karyawan
pada kalimat .... dalam organisasi tersebut.
(A) 1 dan 7 (D) Penghargaan dalam bentuk finansial
(B) 2 dan 5 yang diterima karyawan akan
(C) 4 dan 6 dijelaskan dalam sistem penilaian
(D) 9 dan 13 prestasi kerja.
(E) 11 dan 14 (E) Penilaian prestasi kerja karyawan
yang jelas pada sebuah organisasi
8. Bagaimana hubungan isi antarparagraf akan berdampak pada motivasi kerja
dalam teks tersebut? karyawannya.
(A) Paragraf ke-1 menjadi penyebab
paragraf ke-2.
(B) Paragraf ke-2 memaparkan contoh
paragraf ke-1.
(C) Paragraf ke-3 menjelaskan paragraf
ke-2.

Sony Sugema College 159


Teks berikut digunakan untuk menjawab soal nomor 10 – 15.
(1) Sejak Januari sampai dengan Oktober 2007, harga minyak tidak pernah mengalami
penurunan dalam pergerakan bulanan. (2) Bahkan, bila dibanding harga pada 2006 yang masih
US$27/barel, harga minyak dunia pada tahun 2008 lalu sudah naik tiga kali lipat. (3) Penyebab
kenaikan harga minyak karena beberapa faktor, seperti adanya ketidakseimbangan antara
permintaan dan penawaran serta perkembangan harga minyak dunia selalu mengalami
fluktuasi. (4) Pada tahun 2007 total suplai minyak dunia mencapai 84,65 juta barel per hari
(bph) dengan kontribusi produksi mentah negara anggota OPEC sebesar 30,97 bph atau sekitar
36%. (5) Sampai dengan kuartal ketiga 2008, produksi minyak mentah OPEC meningkat
menjadi 32,35 juta bph, sedangkan total produksi minyak non-OPEC tidak banyak berubah,
yakni sekitar 53,4 juta bph. (6) Pada tahun 2009 diproyeksikan pangsa pasar suplai minyak
non-OPEC akan tidak banyak berubah sekitar 55 – 56 juta bph.
(7) Pada tahun 2008 permintaan minyak terbesar berasal dari negara-negara maju di wilayah
Amerika Utara, Eropa Barat, dan Pasifik. (8) Total permintaan dari negara-negara tersebut adalah
sebesar 47,67 juta bph. (9) Dari jumlah tersebut, proporsi paling besar adalah berasal dari Amerika
Utara, yaitu sebesar 24,30 juta bph. (10) Akan tetapi, total permintaan dari negara-negara
berkembang adalah sebesar 25,06 juta bph. (11) Dengan proporsi terbesarnya adalah berasal dari
negara-negara berkembang di Asia, yaitu sebesar 9,34 juta bph. (12) Kemudian kawasan lainnya, yaitu
negara-negara bekas Uni Soviet, Eropa lainnya, dan Cina memiliki total permintaan sebanyak minyak
sebesar 13,10 juta bph. (13) Dengan proporsi terbesar adalah berasal dari Cina yaitu 8,01 juta bph.
Tabel Realisasi dan Prediksi Permintaan Minyak Dunia Tahun 2008 – 2009 (Juta Bph)

Kawasan 2008 2009 Selisih


Realisasi Total Negara Maju 47,67 46,70 −0,98
Total Negara Berkembang 25,06 25,52 0,46
Total “Kawasan Lainnya” 13,10 13,47 0,37
Total Dunia 85,83 85,68 −0,15
Prediksi 86,19 86,68 0,49
(Dikutip dengan pengubahan dari Ekonomika Indonesia, 2009)

10. Pernyataan manakah yang paling sesuai (D) Kebutuhan minyak negara maju
dengan isi tabel tersebut? tidak lebih banyak dibandingkan
(A) Realisasi permintaan minyak negara dengan total negara berkembang
maju cenderung menurun seperti dan “kawasan lainnya”.
halnya negara “kawasan lainnya”. (E) Kesesuaian prediksi permintaan
(B) Realisasi permintaan yang minyak dunia pada tahun 2009 lebih
meningkat dari tahun 2008 ke 2009 baik bila dibandingkan dengan tahun
paling banyak terjadi pada negara 2008.
berkembang.
(C) Realisasi permintaan minyak dunia
cenderung meningkat dari tahun ke
tahun, baik di kawasan negara maju
maupun lainnya.

160 Sony Sugema College


Sony Sugema College
11. Kalimat yang tidak efektif terdapat pada .... (C) menemukan cara-cara yang efektif
(A) 1 dan 7 untuk mengantisipasi kenaikan
(B) 2 dan 8 harga minyak dunia, terutama bagi
(C) 3 dan 11 negara-negara maju dan
(D) 4 dan 12 berkembang
(E) 5 dan 13 (D) memberikan informasi bahwa harga
minyak dunia cenderung meningkat
12. Simpulan manakah yang paling tepat karena permintaan terus betambah,
untuk teks tersebut? terutama negara berkembang
(A) Harga minyak dunia cenderung (E) mendapatkan gambaran bahwa
meningkat karena kontribusi produksi minyak negara
ketidakseimbangan permintaan- OPEC tidak lebih banyak
penawaran serta harganya selalu dibandingkan dengan negara non-
fluktuatif. OPEC
(B) Pada tahun 2007 total suplai minyak
dunia mencapai angka tertinggi 14. Apa kelemahan isi teks tersebut?
dalam sejarah yagn berasal dari (A) Isi tabel tidak sesuai dengan uraian
negara OPEC dan non-OPEC. sebelumnya.
(C) Diperkirakan permintaan minyak (B) Prediksi permintaan dalam tabel
dunia secara keseluruhan akan tidak dirinci per kawasan.
mengalami penurunan dari tahun (C) Terdapat kesalahan dalam
2008 ke 2009. penghitungan dalam tabel.
(D) Permintaan minyak terbanyak (D) Pernyataan kenaikan harga minyak
berasal dari negara maju di wilayah tidak disertai penyebabnya.
Eropa Barat, Pasifik, dan Amerika (E) Jumlah permintaan negara
Utara sebagai yang terbesar. berkembang yang meningkat tidak
(E) Permintaan minyak negara maju, disertai data.
negara berkembang, dan kawasan
lainnya tidak seimbang dari tahun
2008 ke 2009.

13. Penulis teks tersebut bertujuan agar


pembaca ....
(A) menyadari bahwa meningkatnya
harga minyak dunia disebabkan
ketidak-seimbangan antara
permintaan-penawaran serta
fluktuasi harga
(B) menyadari bahwa permintaan
minyak negara berkembang terus
meningkat karena itu diperlukan
upaya konkret untuk mengurangi
kebutuhan tersebut

Sony Sugema College 161


15. Skema manakah yang paling tepat menggambarkan teks tersebut?
(A)
kenaikan harga
minyak dunia

ketidakseimbangan permintaan &


fluktuasi harga minyak dunia
penawaran

(B) prediksi jumlah permintaan


yang meleset

permintaan dan penawaran


fluktuasi harga minyak dunia
tidak seimbang

kenaikan
harga
minyak
dunia

(C)
ketidak-
seimbangan
permintaan &
penawaran

kenaikan
harga minyak
dunia

fluktuasi
harga minyak
dunia

(D) ketidakseimbangan
kenaikan harga minyak
antara permintaan dan
dunia
penawaran

fluktuasi harga
minyak dunia

(E) ketidakseimbangan
fluktuasi harga
antara permintaan dan
minyak dunia
penawaran

kenaikan harga
minyak dunia

162 Sony Sugema College


Sony Sugema College

Anda mungkin juga menyukai